P1 1a.31fcd1d Notes

You might also like

Download as pdf or txt
Download as pdf or txt
You are on page 1of 220

PART 1

PART 1 UNIT 1

1
1A. External Financial
Reporting Decisions

Module

1 (Optional) Background: Accounting Cycle 3

2 A.1. Financial Statements: Part 1 19

3 A.1. Financial Statements: Part 2 39

4 A.2. Asset Valuation: Receivables 63

5 A.2. Asset Valuation: Inventory 75

6 A.2. Investments and Long-Term Assets 95

7 A.2. Liabilities and Equity 129

8 A.2. Special Valuation Considerations:


Taxes and Leases 145

9 A.2. Revenue Recognition and Income Measurement 167


NOTES

1–2 © Becker Professional Education Corporation. All rights reserved.


(Optional)

1
MODULE
PART 1 UNIT 1

Background: Part 1
Accounting Cycle Unit 1

This section is designed to provide an overview of the accounting cycle and provide a refresher
on fundamental concepts. Although specific questions related to the accounting cycle may not
be seen on the exam, the concepts here are foundational to accounting and are assumed to be
known and understood by exam candidates.

1 Accounting Cycle

The primary objective of the accounting cycle is to capture the economic activity of a business.
The accounting cycle can be broken down into the following steps:
1. Journalize transactions.
2. Post transaction to the general ledger.
3. Prepare an unadjusted trial balance.
4. Record adjusting entries.
5. Prepare an adjusted trial balance.
6. Prepare the financial statements.
7. Prepare closing entries.
8. Prepare the post-closing trial balance.
Each economic transaction captured by the accounting system has an effect on the accounting
equation. The accounting equation can be expressed as follows:

Assets = Liabilities + Owners' equity (or Stockholders' equity)

This equation and any transaction captured by the accounting system demonstrates the
equality of total economic resources (assets) and the claims against those resources (liabilities
and ownership).
Properly recorded journal entries maintain equilibrium in the accounting equation.
The equity portion of the accounting equation is further broken down to demonstrate the
relationship of an owner's equity to the income statement. Contributed capital and retained
earnings are the two primary categories of equity. Contributed capital represents investments
made by owners to the business. Retained earnings represent the utilization of assets in the
creation of wealth for owners and the ability to create returns/profits to owners. These retained
earnings may be reinvested back into the company or made available to pay out to shareholders
as a return through dividends.

© Becker Professional Education Corporation. All rights reserved. Module 1 1–3 (Optional) B
1 (Optional) Background: Accounting Cycle PART 1 UNIT 1

A = L + OE
+ Paid-in capital + Retained earnings

+ Revenues – Expenses
– Dividends
+ Gains – Losses

Every transaction of a business should be analyzed to determine the effect on the accounting
equation. In any transaction, a minimum of two accounts will be affected. When analyzing the
effects of each transaction, follow the steps indicated below:
1. Identify which accounts are involved in the transaction (remember there will be at least two).
2. Determine if the accounts are increasing or decreasing.
3. Ensure the accounting equation balances after the transaction is analyzed.

Illustration 1 Transaction Effects on Financial Statement Elements

For each of the following transactions, indicate the impact on the financial statements and
ensure the accounting equation balances.
1. February 1: The corporation issued an additional 10,000 shares of capital stock to
Harris Corp. in exchange for $15,000 cash.
2. February 1: Purchased a building from Sweeney Enterprise for $150,000. A cash
payment of $25,000 was made at the time of the purchase, and a note payable was
issued for the remaining balance.
3. February 1: Paid $7,800 for fire insurance on building for next 3 years.
4. February 9: Purchased additional supplies on account for $3,500.
5. February 11: Billed students $35,840 for tutoring services provided during the first half
of February.
6. February 16: Paid $12,300 in salaries earned by employees during the first half of
February.
7. February 18: Received $10,500 cash in advance from students for tutoring services to
be provided in the future.
8. February 22: Collected cash from accounts receivable of $40,000.
9. February 25: Paid $3,330 for utilities during period.
10. February 25: Paid $6,250 cash on accounts payable.
11. February 28: Declared and paid a $2,000 dividend.

(continued)

1–4 Module 1 (Optional)


© Becker Professional Education Background:
Corporation. Accounting
All rights reserved. Cycle
PART 1 UNIT
1 1 (Optional) Background: Accounting Cycle

(continued)

Statement of
Stockholders'
Balance Sheet Equity Income Statement
Stockholders'
Assets = Liabilities + Equity Dividends Revenues – Expenses = Net Income
February 1 +15,000 +15,000
February 1 +125,000 +125,000
February 1 Net effect $0
February 19 +3,500 +3,500
February 11 +35,840 +35,840 +35,840 +35,840
February 16 (12,300) (12,300) +12,300 (12,300)
February 18 +10,500 +10,500
February 22 Net effect $0
February 25 (3,330) (3,330) +3,330 (3,330)
February 25 (6,250) (6,250)
February 28 (2,000) (2,000) +2,000

1.1 Journal Entries


Accountants record transactions using the double-entry accounting system using debits (on the
left) and credits (on the right). A T-account is used to graphically represent each account used in
the general ledger. The side an account increases on (shown in bold, below) is considered the
normal balance.

Assets = Liabilities + Equity


+ - - + - +
Debit Credit Debit Credit Debit Credit

Common Stock Retained Earnings


- + - +
Debit Credit Debit Credit

Expenses, Losses, Revenues and


and Dividends Gains
+ - - +
Debit Credit Debit Credit

© Becker Professional Education Corporation. All rights reserved. Module 1 1–5 (Optional) B
1 (Optional) Background: Accounting Cycle PART 1 UNIT 1

Illustration 2 Journal Entries

Required: For each of the following transactions, record the journal entry.
1. February 1: The corporation issued an additional 10,000 shares of capital stock to
Harris Corp. in exchange for $15,000 cash.
2. February 1: Purchased a building from Sweeney Enterprise for $150,000. A cash
payment of $25,000 was made at the time of the purchase, and a note payable was
issued for the remaining balance.
3. February 1: Paid $7,800 for fire insurance on building for next 3 years.
4. February 9: Purchased additional supplies on account for $3,500.
5. February 11: Billed students $35,840 for tutoring services provided during the first half
of February.
6. February 16: Paid $12,300 in salaries earned by employees during the first half of February.
7. February 18: Received $10,500 cash in advance from students for tutoring services to
be provided in the future.
8. February 22: Collected cash from accounts receivable of $40,000.
9. February 25: Paid $3,330 for utilities during period.
10. February 25: Paid $6,250 cash on accounts payable.
11. February 28: Declared and paid a $2,000 dividend.
February 1 journal entry: Assets (cash) and equity (common stock) increased by $15,000.

DR Cash $15,000
CR Common stock $15,000

February 1 journal entry: Assets (building and cash) and liabilities (note payable) increased
by $125,000.

DR Building $150,000
CR Cash $25,000
CR Note payable 125,000

February 1 journal entry: Assets increased (prepaid insurance) and decreased (cash)
by $7,800.

DR Prepaid insurance $7,800


CR Cash $7,800

February 9 journal entry: Assets increased (supplies) and liabilities decreased


(accounts payable) by $3,500.

DR Supplies $3,500
CR Accounts payable $3,500

(continued)

1–6 Module 1 (Optional)


© Becker Professional Education Background:
Corporation. Accounting
All rights reserved. Cycle
PART 1 UNIT
1 1 (Optional) Background: Accounting Cycle

(continued)

February 11 journal entry: Assets (accounts receivable) and revenues (sales revenue)
increased as a result of the credit sale. The increase in revenue increases net income and
eventually increases retained earnings.

DR Accounts receivable $35,840


CR Sales revenue $35,840

February 16 journal entry: Assets (cash) decreased and expenses (wage expense) increased
as a result of wages paid to employees. The increase in expenses decreases net income
and eventually decreases retained earnings.

DR Wage expense $12,300


CR Cash $12,300

February 18 journal entry: Assets (cash) and liabilities (unearned revenue) increased as a
result of the advanced cash payment for tutoring services to be provided in the future by
the company.

DR Cash $10,500
CR Unearned revenue $10,500

February 22 journal entry: Assets (cash) increased and decreased (accounts receivable)
by $40,000.

DR Cash $40,000
CR Accounts receivable $40,000

February 25 journal entry: Assets (cash) decreased and expenses (utilities) increased as
a result of the utilities payment. The increase in expenses decreases net income and
eventually decreases retained earnings.

DR Utilities expense $3,330


CR Cash $3,330

February 25 journal entry: Assets (cash) decreased and liabilities (accounts payable)
decreased by $6,250.

DR Accounts payable $6,250


CR Cash $6,250

February 28 journal entry: Assets (cash) decreased and dividends increased as a result of
the dividend payment. The increase in dividends decreases retained earnings at the end
of the period.

DR Dividends $2,000
CR Cash $2,000

© Becker Professional Education Corporation. All rights reserved. Module 1 1–7 (Optional) B
1 (Optional) Background: Accounting Cycle PART 1 UNIT 1

1.2 General Ledger


Each transaction journalized requires an update to the general ledger. The general ledger shows the
debits and credits recorded for each account and shows the account balance at any point in time.

Illustration 3 Posting to the General Ledger

For each of the following transactions, post the activity recorded in the journal (in the
previous illustration) into the ledger.
1. February 1: The corporation issued an additional 10,000 shares of capital stock to
Harris Corp. in exchange for $15,000 cash.
2. February 1: Purchased a building from Sweeney Enterprise for $150,000. A cash
payment of $25,000 was made at the time of the purchase, and a note payable was
issued for the remaining balance.
3. February 1: Paid $7,800 for fire insurance on building for next 3 years.
4. February 9: Purchased additional supplies on account for $3,500.
5. February 11: Billed students $35,840 for tutoring services provided during the first half
of February.
6. February 16: Paid $12,300 in salaries earned by employees during the first half of February.
7. February 18: Received $10,500 cash in advance from students for tutoring services to
be provided in the future.
8. February 22: Collected cash from accounts receivable of $40,000.
9. February 25: Paid $3,330 for utilities during period.
10. February 25: Paid $6,250 cash on accounts payable.
11. February 28: Declared and paid a $2,000 dividend.

Assets Liabilities
Cash Accounts Payable
Date Debit Credit Date Debit Credit
Beg. Bal. $32,000 Beg. Bal. $4,200
Feb. 1 15,000 Feb. 9 3,500
Feb. 1 $25,000 Feb. 25 $6,250
Feb. 1 7,800 End. Bal. $1,450
Feb. 16 12,300
Feb. 18 10,500 Unearned Revenue
Feb. 22 40,000 Date Debit Credit
Feb. 25 3,330 Beg. Bal. $ 0
Feb. 28 6,250 Feb. 18 10,500
Feb. 28 2,000 End. Bal. $10,500
End. Bal. $40,820

(continued)

1–8 Module 1 (Optional)


© Becker Professional Education Background:
Corporation. Accounting
All rights reserved. Cycle
PART 1 UNIT
1 1 (Optional) Background: Accounting Cycle

(continued)

Assets Liabilities
Accounts Receivable Notes Payable
Date Debit Credit Date Debit Credit
Beg. Bal. $11,000 Beg. Bal. $ 0
Feb. 11 35,840 Feb. 1 125,000
Feb. 22 $40,000 End. Bal. $125,000
End. Bal. $ 6,840

Supplies
Date Debit Credit
Beg. Bal. $5,600
Feb. 9 3,500
End. Bal. $9,100
Equity
Prepaid Insurance Common Stock
Date Debit Credit Date Debit Credit
Beg. Bal. $ 0 Beg. Bal. $ 85,000
Feb. 1 7,800 Feb. 1 15,000
End. Bal. $7,800 End. Bal. $100,000

Land Retained Earnings


Date Debit Credit Date Debit Credit
Beg. Bal. $68,000 Beg. Bal. $27,400
End. Bal. $68,000 End. Bal. $27,400

Building Dividends
Date Debit Credit Date Debit Credit
Beg. Bal. $ 0 Feb. 28 $2,000
Feb. 1 150,000 End. Bal. $2,000
End. Bal. $150,000

Expenses Revenue
Wage Expense Revenue
Date Debit Credit Date Debit Credit
Feb. 16 $12,300 Feb. 11 $35,840
End. Bal. $12,300 End. Bal. $35,840

Utility Expense
Date Debit Credit
Feb. 25 $3,330
End. Bal. $3,330

© Becker Professional Education Corporation. All rights reserved. Module 1 1–9 (Optional) B
1 (Optional) Background: Accounting Cycle PART 1 UNIT 1

1.3 Unadjusted Trial Balance


The unadjusted trial balance reflects the balances in each financial statement account at a point
in time. This trial balance is prepared at the end of the period, prior to any adjusting entries
necessary to ensure total debits posted equal total credits posted and serve as a basis for
determining any adjusting entries needed.

Illustration 4 Unadjusted Trial Balance

The unadjusted trial balance below reflects activity journalized and posted during the
month of February.

Harris Enterprise
Unadjusted Trial Balance
At February 28, Year 2

Account Debit Credit


Cash $ 40,820
Accts. receivable 6,840
Supplies 9,100
Prepaid insurance 7,800
Land 68,000
Building 150,000
Accounts payable $ 1,450
Unearned revenue 10,500
Notes payable 125,000
Common stock 100,000
Retained earnings 27,400
Dividends 2,000
Tutoring service revenue 35,840
Wage expense 12,300
Utility expense 3,330
Total $300,190 $300,190

Note that total debits equal total credits.

1.4 Adjusting Entries


Adjusting journal entries are necessary under accrual accounting to ensure all revenues earned
during the period and expenses incurred are captured so that the correct balances are reflected
on the balance sheet and income statement. Accrual-based accounting does not delay the
recognition of revenues and expenses until cash is collected or cash is paid. The matching
principle requires companies to record expenses when incurred to match them properly against
the associated revenues. Because accrual accounting is based on the timing of revenue and
expense recognition and not when cash is received or paid, adjusting entries are necessary at
the end of each accounting period in order to ensure all revenues earned and expenses incurred
are captured in the accounting system.

1–10 Module 1 (Optional)


© Becker Professional Education Background:
Corporation. Accounting
All rights reserved. Cycle
PART 1 UNIT
1 1 (Optional) Background: Accounting Cycle

There are three broad categories of adjusting journal entries: prepayments/deferrals, accruals,
and estimates.

1.4.1 Prepayments/Deferrals
Prepayments, also referred to as deferrals, are transactions in which cash flows precede the
revenue or expense recognition. Examples of prepayments/deferrals include:
Prepaid/Deferred Expense: Represents assets associated with a cash distribution resulting
in benefits to be received or to be used beyond the current reporting period. Common
examples would include prepaid rent, prepaid insurance, and office supplies. As prepaid
expenses (which are classified as assets) are used, an expense is recognized and the asset is
reduced.
Unearned Revenue: Represents the receipt of cash prior to the providing of a good or
service to a customer. Advance collections of cash from customers result in liabilities on the
part of the company because there is a required future action or event that must take place
before the business earns the revenue.

Illustration 5 Initial Entries With Year-end Adjusting Entries: Rent

Assume that on November 1, Year 1, Swift Corp. prepaid four months of rent by
paying $8,000.
November 1 initial journal entry:

DR Prepaid rent $8,000


CR Cash $8,000

December 31 adjusting journal entry:

DR Rent expense $4,000


CR Prepaid rent $4,000

November and December rent expense of $2,000 per month must be recognized and the
prepaid rent account needs to reflect only two months of prepaid rent remaining.

Prepaid Rent Rent Expense

Balance Balance
Date Debit Credit Debit Credit Date Debit Credit Debit Credit
Nov. 1 $8,000 $8,000 Dec. 31 $4,000 $4,000
Dec. 31 $4,000 $4,000

Assume that on January 1, Year 1, Swift purchases $1,000 of supplies. A physical count of
office supplies available at the end of the month showed that only $200 in supplies remain.

(continued)

© Becker Professional Education Corporation. All rights reserved. Module 1 1–11 (Optional) B
1 (Optional) Background: Accounting Cycle PART 1 UNIT 1

(continued)

January 1 initial entry:

DR Office supplies $1,000


CR Cash $1,000

January 31 adjusting entry:

DR Supply expense $800


CR Office supplies $800

The supplies that have been used during the period no longer have a benefit to the
company (they are no longer an asset), therefore, an entry is necessary to record the
expense associated with the use of the supplies and to adjust the office supplies account
to reflect supplies still on hand at the end of the period.

Office Supplies Supply Expense

Balance Balance
Date Debit Credit Debit Credit Date Debit Credit Debit Credit
Jan. 1 $1,000 $1,000 Dec. 31 $800 $800
Dec. 31 $800 $ 200

When advance collections of cash create a liability (unearned revenue) on the part of a
company, adjusting entries are necessary as services are earned.
Assume on May 1, Year 1, Swift receives $600 in prepayments from customers for three
events that Swift will host for $200 per event. By the end of May, Swift has hosted two of
the three events. Prepare both the initial and adjusting entries necessary.
May 1 initial journal entry:

DR Cash $600
CR Unearned revenue $600

May 31 adjusting journal entry:

DR Unearned revenue $400


CR Service revenue $400

Swift has earned the revenue by hosting two events and at May 31, Swift is only responsible
for hosting one remaining event. Two of the three events hosted results in revenue of $400
earned; the $200 remaining balance in the liability account, unearned revenue, reflects the
one remaining event.

Unearned Revenue Service Expense

Balance Balance
Date Debit Credit Debit Credit Date Debit Credit Debit Credit
May 1 $600 $600 Dec. 31 $400 –
Dec. 31 $400 $200

1–12 Module 1 (Optional)


© Becker Professional Education Background:
Corporation. Accounting
All rights reserved. Cycle
PART 1 UNIT
1 1 (Optional) Background: Accounting Cycle

1.4.2 Accruals
Accruals involve transactions where the cash outflow or inflow takes place in the period
following the adjusting entry. The adjusting entry for accruals happens first followed by the cash
inflow in future periods.
Common examples of accruals include accruals for liabilities as well as for receivables at the end
of a period.
Accrued liabilities represent liabilities recorded when an expense has been incurred, but
no cash has been paid yet. Common examples of accrued liabilities would include wages
payable and interest payable. For example, when employees perform work associated with
wages, at the end of the accounting period, the company must accrue earned wages for
unpaid work. A corresponding liability is recognized at the time of the accrual. The liability is
reversed when payments occur in the following period.
Accrued receivables involve situations when revenue has been earned and is eligible for
recognition, but cash has not yet been received. For example, when investments in debt
made by a company result in the periodic receipt of interest payments, any interest earned
at the end of the period is accrued and a receivable representing the right to receive the
interest is reflected on the balance sheet.

Illustration 6 Initial Entries With Year-end Adjusting Entries: Interest

Assume Harris Investments has earned $1,500 in interest on investments in Dawson


Enterprises at December 31, Year 1. The interest payment will not be made until January 1,
Year 2.
December 31 adjusting journal entry:

DR Interest receivable $1,500


CR Interest revenue $1,500

January 1 subsequent journal entry:

DR Cash $1,500
CR Interest receivable $1,500

The interest revenue is recognized in the period earned, regardless of the collection of
cash. With revenue/receivable accruals, the adjusting entry precedes the collection of cash.

Interest Receivable Interest Revenue

Balance Balance
Date Debit Credit Debit Credit Date Debit Credit Debit Credit
Dec. 31 $1,500 $1,500 Dec. 31 $1,500 $1,500
Jan. 1 $1,500 —

Assume employees earn wages of $35,000 per week based on a five-day work week.
December 31 is a Tuesday, and payday is Friday.

(continued)

© Becker Professional Education Corporation. All rights reserved. Module 1 1–13 (Optional) B
1 (Optional) Background: Accounting Cycle PART 1 UNIT 1

(continued)

December 31 adjusting entry:

DR Wage expense $14,000


CR Wage payable $14,000

January 3 subsequent entry:

DR Wage expense $21,000


DR Wage payable 14,000
CR Cash $35,000

The wage expense incurred in December should be recognized in December regardless


of cash payment in January. $35,000 / 5 days = $7,000 per day × 2 days = 14,000 expense
associated with December. With expense/liability accruals, the adjusting entry precedes the
payment of cash.

Wages Payable Wage Expense

Balance Balance
Date Debit Credit Debit Credit Date Debit Credit Debit Credit
Dec. 31 $14,000 $14,000 Dec. 31 $14,000 $14,000

1.4.3 Estimates
Many adjusting entries involve either prepayments or accruals; however, estimates made in the
accounting process also result in adjusting entries. A common example of this is depreciation.
When long-term assets are acquired, an allocation of the cost of the asset over period of benefit
is used to match depreciation expense against revenues earned during the period.
Many estimates go into the calculation of the allocation process. For example, the purchase
of equipment at a cost of $10,000 on January 1, with an expected useful life of four years and
no salvage value would be depreciated on a straight-line basis $2,500 per year for four years
($10,000 / 4 years = $2,500 per year). Depreciation is not an attempt to reflect market value of
an asset, but merely to allocate cost over the period of expected benefit, matching the expense
of the asset to the revenues generated during its use.

1–14 Module 1 (Optional)


© Becker Professional Education Background:
Corporation. Accounting
All rights reserved. Cycle
PART 1 UNIT
1 1 (Optional) Background: Accounting Cycle

Illustration 7 Initial Entries With Year-end Adjusting Entries: Depreciation

Assume Jones Co. purchased a $50,000 building with a 25-year life on January 1, Year 1.
January 1 initial entry:

DR Building $50,000
CR Cash/note payable $50,000

December 31 adjusting entry:

DR Depreciation expense $2,000


CR Accumulated depreciation—building $2,000

The year-end adjusting entries would include $2,000 in depreciation expense for one year
of benefit received out of the estimated 25-year life.

Accumulated Depreciation—Building Depreciation Expense

Balance Balance
Date Debit Credit Debit Credit Date Debit Credit Debit Credit
Dec. 31 $2,000 $2,000 Dec. 31 $2,000 $2,000

1.5 Adjusted Trial Balance


After adjusting entries are made, the general ledger is updated to reflect the adjusted balances
and an adjusted trial balance is prepared. The adjusted trial balance is the starting point for
the preparation of the company's financial statements, because all transactions and financial
changes for the period reported have been captured by the accounting system. The format of
an adjusted trial balance is identical to the initial trial balance, but the title is updated to indicate
"adjusted" because new revenue and expense items may exist as well as increases/decreases to
related assets and liabilities after the adjustments are made.

© Becker Professional Education Corporation. All rights reserved. Module 1 1–15 (Optional) B
1 (Optional) Background: Accounting Cycle PART 1 UNIT 1

Illustration 8 Adjusted Trial Balance

Based on adjusting entries, the adjusted trial balance for Harris Enterprises is shown below.

Harris Enterprises
Adjusted Trial Balance
February 28, Year 2

Account Debit Credit


Cash $ 40,820
Accts. receivable 6,840
Supplies 1,500
Prepaid insurance 7,583
Land 68,000
Building 150,000
Accumulated depreciation $ 625
Accounts payable 1,450
Unearned revenue 5,750
Wage payable 11,600
Interest payable 1,250
Note payable 125,000
Common stock 100,000
Retained earnings 27,400
Dividends 2,000
Tutoring service revenue 40,590
Wage expense 23,900
Utility expense 3,330
Insurance expense 217
Supply expense 7,600
Depreciation expense 625
Interest expense 1,250
Total $313,665 $313,665

Note that total debits equal total credits.

1–16 Module 1 (Optional)


© Becker Professional Education Background:
Corporation. Accounting
All rights reserved. Cycle
PART 1 UNIT
1 1 (Optional) Background: Accounting Cycle

1.6 Closing Entries and the Post-Closing Trial Balance


Once the adjusted trial balance is prepared, the financial statements can be prepared. At
the end of a reporting period, all temporary accounts must be closed. Temporary accounts
include all revenues, expenses, and dividends during the period. Revenues less expenses (net
income) and dividends paid to shareholders during the period are then closed into the retained
earnings account.
Only permanent accounts (assets, liabilities, and equity) remain after closing. Once all closing
journal entries have been made, a post-closing trial balance is prepared to reflect assets,
liabilities, and equity. The only permanent account that will have a change from the adjusted to
post-closing trial balance is the retained earnings account.

Illustration 9 Preparing Closing Entries

Use the information from Harris Enterprises Income Statement to prepare the closing entries.
February 28 close revenue accounts to retained earnings:

DR Tutoring service revenue $40,590


CR Retained earnings $40,590

February 28 close expense accounts to retained earnings:

DR Retained earnings $36,922


CR Wage expense $23,900
CR Utility expense 3,330
CR Insurance expense 217
CR Supply expense 7,600
CR Depreciation expense 625
CR Interest expense 1,250

February 28 close dividends to retained earnings:

DR Retained earnings $2,000


CR Dividends $2,000

(continued)

© Becker Professional Education Corporation. All rights reserved. Module 1 1–17 (Optional) B
1 (Optional) Background: Accounting Cycle PART 1 UNIT 1

(continued)

Once the closing entries are prepared, the post-closing trial balance is checked to ensure
only balance sheet accounts remain:

Harris Enterprises
Post-Closing Trial Balance
February 28, Year 2

Account Debit Credit


Cash $ 40,820
Accts. receivable 6,840
Supplies 1,500
Prepaid insurance 7,583
Land 68,000
Building 150,000
Accumulated depreciation $ 625
Accounts payable 1,450
Unearned revenue 5,750
Wage payable 11,600
Interest payable 1,250
Note payable 125,000
Common stock 100,000
Retained earnings 29,068
Total $274,743 $274,743

Note that total debits equal total credits.

1–18 Module 1 (Optional)


© Becker Professional Education Background:
Corporation. Accounting
All rights reserved. Cycle
2
MODULE
PART 1 UNIT 1

A.1. Financial
Statements: Part 1
Part 1
Unit 1

This module covers the following content from the IMA Learning Outcome Statements.

CMA LOS Reference: Part 1—Section A.1. Financial Statements: Part 1

The candidate should be able to:


a. identify the users of these financial statements and their needs
b. demonstrate an understanding of the purposes and uses of each statement
c. identify the major components and classifications of each statement
d. identify the limitations of each financial statement
e. identify how various financial transactions affect the elements of each of the financial
statements and determine the proper classification of the transaction
f. demonstrate an understanding of the relationship among the financial statements
g. demonstrate an understanding of how a balance sheet, an income statement, a
statement of changes in equity, and a statement of cash flows (indirect method)
are prepared

1 Income Statement LOS 1A1a

The purpose of the income statement is to provide information about the revenues, expenses,
gains, and losses associated with the operations of the company during a specified period of time.

1.1 Uses of the Income Statement LOS 1A1b

The income statement is useful in determining profitability and value for investment purposes,
as well as creditworthiness for prospective lenders. Evaluating profitability provides insight into
the utilization of company assets in the creation of wealth for shareholders. The evaluation of
the income statement allows users to determine/estimate the value of the company as well as
make decisions regarding the long-term solvency. The multistep income statement categorizes
and lists the components of net income in order for users to determine profit (loss) from
operations and from ancillary activities.

1.2 Components of the Income Statement LOS 1A1c

The multistep income statement consists of two categories: operating items and nonoperating LOS 1A1e
items. Operating revenues and expenses are directly related to the primary revenue generating
activities of the company. Nonoperating revenues, expenses, gains, and losses (interest expense,
interest revenue, etc.) are associated with the company's peripheral or incidental activities and
included in income from continuing operations.

© Becker Professional Education Corporation. All rights reserved. Module 2 1–19 A.1
2 A.1. Financial Statements: Part 1 PART 1 UNIT 1

Classifying items as either operating or nonoperating allows users to differentiate between


recurring earnings and nonrecurring earnings. Recurring items are likely to occur on a regular
basis and have a more consistent effect on earnings. Nonrecurring items are not likely to recur
and can have an irregular effect on earnings.
Income tax expense is reported separately on an income statement as a major subtotal.

Example 1 Cost Classification

Facts: Dawson Corp.'s adjusted trial balances consist of the following income statement items:

Debit Credit
Service revenue $5,000,000
Interest income 150,000
Gain on the sale of investments 25,000
Cost of sales $2,700,000
Selling expenses 340,000
General and administrative expenses 660,000
Interest expense 15,000
Research and development expense 78,000
Required: Classify costs as operating and nonoperating. Calculate the company's operating
income for the year.
Solution:

Classification
Service revenue Operating
Interest income Nonoperating
Gain on the sale of investments Nonoperating
Cost of sales Operating
Selling expenses Operating
General and administrative expenses Operating
Interest expense Nonoperating
Research and development expense Operating
Operating Income:

Service revenue $5,000,000


Cost of sales (2,700,000)
Selling expenses (340,000)
General and administrative expenses (660,000)
Research and development expense (78,000)
Operating income $1,222,000

1–20 Module 2 A.1. Financial


© Becker Professional Education Corporation. Statements:
All rights reserved.Part 1
PART 1 UNIT
2 1 A.1. Financial Statements: Part 1

1.3 Income Statement Presentation LOS 1A1g

The income statement reports operating revenues and expenses separately from nonoperating
revenues and expenses and other gains and losses. The multiple-step income statement
presents major subtotals such as gross profit, operating income, and income before taxes.
The benefit of the multiple-step income statement is enhanced user information; line
items presented often provide the user with readily available data with which to calculate
analytical ratios.

Maxwell Gear Corp.


Income Statement
For the Year Ended December 31, 2021

Sales revenue $573,522


Cost of goods sold (302,371)
Gross profit $271,151
Operating expenses:
Selling expense $(47,341)
General and administrative expense (24,888)
Research and development expense (16,300)
Total operating expenses (88,529)
Operating income $182,622
Other income (expense):
Interest revenue $ 5,500
Gain on sale of investments 26,400
Interest expense (14,522)
Total other income, net 17,378
Income before income taxes $200,000
Income tax expense (50,000)
Net income $150,000

1.4 Single-Step Income Statement


In the single-step income statement, presentation of income from continuing operations and
total expenses (including income tax expense) are subtracted from total revenues in a single
step. The benefits of a single-step income statement are its simple design, and the various types
of revenues or expenses do not appear to be more important than any other type.

© Becker Professional Education Corporation. All rights reserved. Module 2 1–21 A.1
2 A.1. Financial Statements: Part 1 PART 1 UNIT 1

Maxwell Gear Corp.


Income Statement
For the Year Ended December 31, 2021

Revenue and gains:


Sales revenue $ 573,522
Interest revenue 5,500
Gain on sale of investments 26,400
Total revenues and gains $605,422
Expenses and losses:
Cost of goods sold $(302,371)
Selling expense (47,341)
General and administrative expense (24,888)
Research and development expense (16,300)
Interest expense (14,522)
Total expenses and losses (405,422)
Income before income taxes $200,000
Income tax expense (50,000)
Net income $150,000

LOS 1A1d 1.5 Limitations of the Income Statement


Management has the ability to change reported income through the use of assumptions and
estimates. Assumptions and estimates are utilized to account for items such as depreciation,
warranty costs, and allowances for bad debts. For example, depreciation involves the allocation
of cost over a period of benefit. The appropriate useful life, salvage value, and method of
deprecation are determined by management based on estimates regarding the use of the
asset. Estimates must be made and accounted for at the point of sale for future warranty
costs associated with inventory sold. Credit sales may result in bad debts if customers do
not ultimately pay. Allowances must be made by management based on an evaluation of
receivables for this risk. Each of the items indicated creates uncertainties and therefore requires
management to calculate an estimate. The subjective nature of items evaluated results in
increased risks of manipulation of the assumptions and estimates used.
Manipulation of assumptions and estimates is sometimes referred to as income smoothing.
Opposing views of the utilization of these techniques exist. Some argue that investors
and creditors are better able to analyze reported income and make predictions of future
performance when volatility of earnings is managed. However, many believe income smoothing
leads to income manipulation, resulting in altering the financing performance and causing
investors and creditors not to fully comprehend the risk of operations.

1–22 Module 2 A.1. Financial


© Becker Professional Education Corporation. Statements:
All rights reserved.Part 1
PART 1 UNIT
2 1 A.1. Financial Statements: Part 1

Another limitation to the income statement is management's ability to undertake classification


shifting. Classification shifting involves misclassifying operating expenses as nonoperating
expenses, which results in higher reported operating income. Operating income is used by
investors and creditors to evaluate the company's performance and to make predictions
of future performance. Net income is not altered, but the evaluation of profits based on
core operations (the quality of earnings) appears stronger than it should because of the
misclassification of expenses. As such, classification shifting leads to distorted views of the
profitability of the business.

Example 2 Multiple-Step Income Statement

Facts: The following adjusted trial balance contains a list of income statement accounts for
the year ended December 31, Year 1.
Adjusted Trial Balance: Income Statement Accounts Only
For the Year Ended December 31, Year 1 (in thousands)
4000 Sales revenue $380
4050 Sales returns $ 25
4060 Sales discounts 5
4100 Service revenue 200
4200 Rental revenue 100
5000 Cost of goods sold 200
5100 Cost of services sold 150
5200 Cost of rental income 60
5250 Salaries expense* 70
5300 Freight out 25
5400 Commissions 40
5500 Advertising 15
5600 Insurance expense 20
5800 Depreciation expense 80
5900 Income tax expense 100
6000 Interest revenue 170
6200 Other revenue 130
6500 Gain on sale 50
7000 Interest expense 50
7500 Loss on sale of assets 40
7550 Loss on sale of investments 100
*Salaries expense: $20 relates to salaries for sales representatives and $50 relates to
salaries for officers of the company.
Note: This adjusted trial balance does not balance because the balance sheet accounts are
excluded from this example.

(continued)

© Becker Professional Education Corporation. All rights reserved. Module 2 1–23 A.1
2 A.1. Financial Statements: Part 1 PART 1 UNIT 1

(continued)

Required: Prepare a multiple-step income statement based upon the adjusted trial balance
information above.
Solution:

Radon Industries Inc.


Income Statement
For the Year Ended December 31, Year 1
(in thousands)

Net sales (including goods, services, and rentals), less


discounts and returns $650
Cost of sales (including goods, services, and rentals): (410)
Gross margin $240
Selling expenses $(100)
General and administrative expenses (70)
Depreciation expense (80) (250)
Income (loss) from operations $ (10)
Other revenues and gains:
Interest revenue $ 170
Gain on sale of fixed assets 50
Other revenue 130 350
Other expenses and losses:
Interest expense (50)
Loss on sale of fixed assets (40) (90)
Income before income tax $250
Loss on sale of available-for-sale securities (100)
Income before income tax $150
Income tax expense (100)
Net income (or "income from continuing operations")* $ 50

*"Income from continuing operations" would be used if the income statement


includes "discontinued operations."

LOS 1A1a 2 Statement of Changes in Equity


LOS 1A1b
Stockholders' equity (also called shareholders' equity or owners' equity) is the owners' claim to the
LOS 1A1c net assets (i.e., assets minus liabilities) of a corporation. Stockholders' equity is generally presented
on the statement of financial position (balance sheet) as the last major section, following liabilities.
The accounts that make up stockholders' equity are clearly classified according to source.

1–24 Module 2 A.1. Financial


© Becker Professional Education Corporation. Statements:
All rights reserved.Part 1
PART 1 UNIT
2 1 A.1. Financial Statements: Part 1

The two primary sources of ownership interests are amounts invested by shareholders of
the corporation (paid-in capital) and amounts earned by the corporation on behalf of the
shareholders (retained earnings).

2.1 Equity Components and Classifications


Shareholders' equity consists of four main classifications: paid-in capital, retained earnings,
accumulated other comprehensive income, and treasury stock.
The balance sheet reports the shareholders' equity account balances at a specific point in time.
The purpose of the statement of shareholders' equity is to disclose the activity in the equity
accounts from the beginning of an accounting period to the end of the account period.
The illustration below shows the shareholders' equity portion of the balance sheet.

BGSE Capital Corp.


Partial Balance Sheet—Shareholders' Equity
December 31, Year 5

Capital Stock
Preferred stock, cumulative, $100 par value; 10,000 shares
authorized, 2,000 shares issued and outstanding $ 200,000
Common stock, $10 par value; 1,000,000 shares authorized,
100,000 shares issued, 97,000 outstanding 1,000,000
Total capital stock 1,200,000
Additional Paid-in Capital
Paid-in capital in excess of par—preferred stock 30,000
Paid-in capital in excess of par—common stock 2,500,000
Total Paid-in Capital $ 3,730,000
Retained earnings 16,500,000
Accumulated other comprehensive income 350,000
Treasury stock (120,000)
Total Shareholders' Equity $20,460,000

2.2 Capital Stock


Legal capital or capital stock is the amount of capital that must be retained by the corporation
for the protection of creditors.

2.2.1 Common Stock


Common stock is the basic ownership interest in a corporation. Common shareholders bear the
ultimate risk of loss and receive the ultimate benefits of success. Because common shareholders
bear risk of loss, they are not guaranteed dividends or a share of assets upon dissolution of the
company. Common shareholders generally control management through their right to vote.
Common shareholders also have the right to share in earnings of the corporation and the right
to share in assets upon liquidation after the claims of creditors and preferred shareholders
are satisfied.

© Becker Professional Education Corporation. All rights reserved. Module 2 1–25 A.1
2 A.1. Financial Statements: Part 1 PART 1 UNIT 1

2.2.2 Preferred Stock


Preferred stock is an equity security with preferences and features not associated with common
stock. Preferred stock may include a preference relating to dividends, which may be cumulative
or noncumulative, and participating or nonparticipating. Preferred stock may also include a
preference relating to liquidation. Usually, preferred stock does not have voting rights.

2.2.3 Par Value


Generally, preferred stock is issued with a par value, but common stock may be issued with or
without a par value. No-par common stock may be issued as either true no-par stock or as no-
par stock with a stated value. Any excess of the actual amount received over the par or stated
value of the stock is accounted for as additional paid-in capital.

2.2.4 Authorized, Issued, and Outstanding


A corporation's charter contains the amounts of each class of stock that it may legally issue, and
this is called authorized capital stock. When part or all of the authorized capital stock is issued,
it is called issued capital stock. Because a corporation may own issued capital stock in the form
of treasury stock, the amount of issued capital stock in the hands of shareholders is called
outstanding capital stock. In summary, capital stock may be:
1. authorized (in the corporate charter);
2. authorized and issued (previously issued to shareholders but no longer owned by a
shareholder); or
3. authorized, issued, and outstanding (previously issued to shareholders and still outstanding
in the market).
The number of shares of each class of stock and their status as authorized, issued, or
outstanding must be disclosed in the financial statements and the accompanying notes to the
financial statements.

2.3 Additional Paid-in Capital


Additional paid-in capital is generally contributed capital in excess of par or stated value. It can
also arise from other types of transactions. Additional paid-in capital may be aggregated and
shown as one amount on the balance sheet.

Example 3 Journal Entry: Stock Issuance for Cash

Facts: On January 1, Year 1, Harris Enterprises issued 1 million shares of its $10 par
common stock for $25 per share.
Required: Prepare the journal entry for the issuance of shares.
Solution:
January 1 journal entry:

DR Cash $25,000,000
CR Common stock $10,000,000
CR Additional paid-in capital—common stock 15,000,000

1–26 Module 2 A.1. Financial


© Becker Professional Education Corporation. Statements:
All rights reserved.Part 1
PART 1 UNIT
2 1 A.1. Financial Statements: Part 1

2.4 Retained Earnings


Retained earnings (or deficit) is accumulated earnings (or losses) during the life of the corporation
that have not been paid out as dividends. The amount of accumulated retained earnings is
reduced by distributions to stockholders and transfers to additional paid-in capital for stock
dividends. Retained earnings do not include treasury stock or accumulated other comprehensive
income. If the retained earnings account has a negative balance, it is called a deficit.

Net income/loss
– Dividends (cash, property, and stock) declared
± Prior period adjustments
± Accounting changes reported retrospectively
Retained earnings

2.5 Accumulated Other Comprehensive Income


Components of accumulated other comprehensive income include pension adjustments,
unrealized gains and losses on available-for-sale debt securities, foreign currency translation
adjustments, any portion of a gain or loss that is the result of a change in credit risk of debt
when the fair value option is elected, and deferred gains and losses on the effective portion of
cash flow hedges.
These components of other comprehensive income are not included in determining net
income and, therefore, are not included in retained earnings. Rather, these components are
recognized in the period in which they occur and are combined with net income to determine
comprehensive income. Total accumulated other comprehensive income must be shown in
the shareholders' equity section separate from capital stock, additional paid-in capital, and
retained earnings.

2.6 Treasury Stock


Treasury stock is a corporation's own stock that has been issued to shareholders and
subsequently reacquired (but not retired) by the corporation. Because treasury shares are held
(owned) by the issuing corporation, they are not entitled to any of the common shareholders'
ownership rights, such as the right to vote or to receive dividends.

© Becker Professional Education Corporation. All rights reserved. Module 2 1–27 A.1
2 A.1. Financial Statements: Part 1 PART 1 UNIT 1

BGSE Capital Corp.


Statement of Shareholders' Equity
For the Year Ended December 31, Year 5

Add'l Add'l Accumulated


Paid-in Paid-in Other Total
Preferred Common Capital: Capital: Retained Comprehensive Treasury Stockholders'
Stock Stock PS CS Earnings Income Stock Equity
Balances,
January 1, Year 5 $200,000 $ 990,000 $30,000 $2,475,000 $11,265,000 $362,000 $(120,000) $15,202,000
Net income 6,185,000 6,185,000
Other
comprehensive
income (12,000) (12,000)
Cash dividends (950,000) (950,000)
Issuance of
common stock 10,000 25,000 35,000
Balances,
December 31,
Year 5 $200,000 $1,000,000 $30,000 $2,500,000 $16,500,000 $350,000 $(120,000) $20,460,000

LOS 1A1d 2.7 Limitations of the Statement of Changes in Equity


The statement of changes in equity is a straightforward concept. This statement represents
the residual interest of assets minus creditors' claims to those assets. Differences within state
laws affect the account of shareholders' equity transactions and can cause variations in the
accounting and reporting of equity, making the statement of shareholders' equity difficult to
compare between companies.
Despite the relatively simple calculation of equity, this section is frequently misunderstood due
to terminology differences of reporting companies. For example, the term shareholders' equity,
shareowners' equity, stockholders' investments, contributed capital, and other similar titles can
be used to describe equity. Terminology surrounding stock varies from common stock to class A
shares or stock, preferred stock, or preference shares. Retained earnings may be referred to as
reinvested earnings or earned capital.
The statement often provides a three-year reconciliation of the changes in balances of
stockholders' equity accounts. Users of the financial statements should read management's
discussion and analysis along with the footnotes to the financial statements to help decipher the
components of equity.

1–28 Module 2 A.1. Financial


© Becker Professional Education Corporation. Statements:
All rights reserved.Part 1
PART 1 UNIT
2 1 A.1. Financial Statements: Part 1

3 Balance Sheet LOS 1A1a

The balance sheet reports the company's financial position (assets, liabilities, and equity) at
a specific point in time. The formal name for the balance sheet is the statement of financial
position. The formula for the balance sheet is:

Assets = Liabilities + Shareholders' equity

3.1 Purposes and Uses of the Balance Sheet LOS 1A1b

The balance sheet provides a list of assets and liabilities held by an entity, which are classified
into meaningful categories, and owners' equity. Classification of items on the balance sheet
allows users of the financial statements to determine not only the composition of assets and
liabilities, but also to evaluate the liquidity and long-term solvency.

3.2 Components of the Balance Sheet LOS 1A1c

The three main components of the balance sheet are assets, liabilities, and owners' equity. LOS 1A1e

3.2.1 Assets
Assets represent probable future economic benefits obtained or controlled by a particular entity
as a result of past transactions or events. Generally, assets are what a company "owns." There
are five categories of assets reported on the balance sheet: current assets; investments and
other funds; property, plant, and equipment; intangible assets; and other assets.
Current Assets: Cash and other assets expected to be converted to cash or consumed
either within one year of the balance sheet date or within the company's operating cycle,
whichever is longer. Current assets are listed in order of liquidity. Examples: cash and cash
equivalents, short-term investments (management ability and intent to sell within a year),
accounts receivable, inventories, and prepaid expenses (receive benefit within one year).
Investments and Funds: Assets that are not used directly in operations. These are
noncurrent because of management's intent to not convert the assets into cash within
the next year or within the company's operating cycle, (whichever is longer). Examples:
investments in equity and debt securities of other companies, land held for speculation,
noncurrent receivables, certificates of deposit having a maturity date greater than one year
from the date the certificate was issued, and cash set aside for special purposes.
Property, Plant, and Equipment: Assets that are tangible, long-lived, and used in business
operations. Reported at historical cost less accumulated depreciation to date (except land,
which is not depreciated). Examples: land, land improvements, buildings, equipment,
machinery, furniture, and natural resources.

© Becker Professional Education Corporation. All rights reserved. Module 2 1–29 A.1
2 A.1. Financial Statements: Part 1 PART 1 UNIT 1

Intangible Assets: Assets that have no physical substance, are long-lived, and used in the
operations of the business. These assets typically represent exclusive rights that a company
can use to generate future revenues. Reported at historical cost net of accumulated
amortization (usually shown net). Examples: patents, copyrights, trademarks, trade names,
and franchises.
Other Assets: A catch-all category of noncurrent assets. This category is reserved for assets
that will not fit into one of the first four categories discussed above. Examples: long-term
prepayments (sometimes called deferred charges) and deferred income taxes.
Each company varies in terms of asset composition; therefore, the types of assets reported on
each company's balance sheet may differ.

3.2.2 Liabilities
Liabilities represent probable future sacrifices of economic benefits arising from present
obligations of a particular entity to transfer assets or provide services to other entities in the
future as a result of past transactions or events. Liabilities represent what a company "owes."
Liabilities are reported as either current or non-current obligations.
Current Liabilities: Represent obligations that are expected to be satisfied with current
assets or the creation of other current liabilities. Includes all liabilities that are expected to
be satisfied within one year or within the company's operating cycle, whichever is longer
(with some exceptions outside the scope of the CMA Exam). Examples: accounts payable,
notes payable, unearned (or deferred) revenues, accrued liabilities, and current maturities
of long-term debt.
Long-Term Liabilities: Represent obligations that will not be satisfied within the next year
or within the company's operating cycle, whichever is longer. Examples: long-term notes,
bonds, lease obligations, and pension obligations.

LOS 1A1g 3.2.3 Equity (or Net Assets)


Equity represents the shareholders' residual claim in the entity's assets after deducting
liabilities. That is, the difference between what the company "owns" and what the company
"owes." Categories of equity include paid-in capital, retained earnings, accumulated other
comprehensive income, and treasury stock. Equity is presented in order of preference upon
liquidation.

Illustration 1 Example Balance Sheet Presentation

Below is the balance sheet for Emmons Corp. at December 31, Year 1.
Amounts reflected in the financial statements come from the adjusted trial balance. Note
the order of the presentation of total assets is consistent with the categories discussed
above. The balance sheet below does not contain an "other asset" category because it
did not apply to Emmons Corp. It is important to note the balance sheet must balance:
Accounting equation assets = Liabilities + Owners' equity must always be in equilibrium.
If the balance sheet does not balance, something has been incorrectly recorded.

1–30 Module 2 A.1. Financial


© Becker Professional Education Corporation. Statements:
All rights reserved.Part 1
PART 1 UNIT
2 1 A.1. Financial Statements: Part 1

Emmons Corp.
Balance Sheet
At December 31, Year 1
Assets
Current Assets:
Cash $ 80
Trading securities 75
Accounts receivable, net of allowance of $10 250
Inventories 120
Prepaid rent 25
Total current assets $ 550
Investments:
Land held for investment 90
Investment in stocks 75
Total investments 165
Property, Plant, and Equipment:
Land 200
Buildings 65
Equipment 100
Accumulated depreciation (75)
Total property, plant, and equipment 295
Intangible Assets:
Copyright 20
Total intangible assets 20
Total assets $1,025
Liabilities and Stockholders' Equity
Current Liabilities:
Dividends payable $ 10
Notes payable 50
Accounts payable 35
Total current liabilities $ 95
Long-term Liabilities:
Notes payable 75
Bonds payable 525
Total long-term liabilities 600
Total liabilities $ 695
Stockholders' Equity:
Common stock, $1 par 30
Additional paid-in capital—common stock 270
Total paid-in capital $ 300
Retained earnings (calculated) 30
Total stockholders' equity 330
Total liabilities and stockholders' equity $1,025

© Becker Professional Education Corporation. All rights reserved. Module 2 1–31 A.1
2 A.1. Financial Statements: Part 1 PART 1 UNIT 1

LOS 1A1d 3.3 Limitations of the Balance Sheet


The key limitation of the balance sheet is that the net assets (assets minus liabilities) does
not equal the market value of the entity. The reasons for the difference in value include the
use of historical cost for many assets reported on the balance sheet, the subjective nature of
asset recognition, and the use of estimates within the financial statements. Even though the
balance sheet does not directly measure the market value of the company, it provides valuable
information on which users of the balance sheet can base decisions regarding market valuation.

4 Preparation of Financial Statements

Once the adjusted trial balance is prepared, the financial statements can be compiled. The four
basic financial statements are the income statement, statement of changes in equity, balance
sheet, and statement of cash flows.

4.1 Income Statement


The income statement is prepared first and reflects the revenues earned and expenses incurred
for a stated period of time. In this example, the income statement summarizes all of the
revenues and expenses occurring in the month of February.

Harris Enterprises
Income Statement
For the Month Ended February 28
Tutoring service revenue $ 40,590
Wage expense $(23,900)
Utility expense (3,330)
Insurance expense (217)
Supply expense (7,600)
Depreciation expense (625)
Interest expense (1,250)
Total expenses (36,922)
Net income $ 3,668

4.2 Statement of Changes in Stockholders' Equity


The statement of changes in stockholders' equity follows the preparation of the income
statement and explains the changes in ownership for an entity by providing a reconciliation
of beginning of the year balances within equity and detailing changes within each ownership
category resulting in updated year-end balances. The income statement must be prepared first,
because net income from the income statement is added to retained earnings shown on the
statement of stockholders' equity.

1–32 Module 2 A.1. Financial


© Becker Professional Education Corporation. Statements:
All rights reserved.Part 1
PART 1 UNIT
2 1 A.1. Financial Statements: Part 1

Harris Enterprises
Statement of Changes in Stockholders' Equity
For the Month Ended February 28

Common Stock Retained Earnings Total


Beginning balances $ 85,000 $27,400 $112,400
Issuance of stock 15,000 15,000
Net income 3,668 3,668
Dividends (2,000) (2,000)
Ending balances $100,000 $29,068 $129,068

4.3 Balance Sheet


The balance sheet reflects the financial position of a company at a specific date. To prepare the
balance sheet, the ending balance of retained earnings must be calculated on the statement of
stockholders' equity.

Harris Enterprises
Balance Sheet
As of February 28

Cash $ 40,820
Accts. receivable 6,840
Supplies 1,500
Prepaid insurance 7,583
Total current assets $ 56,743
Land 68,000
Building 150,000
Accumulated depreciation (625)
Total assets $274,118

Accounts payable $1,450


Unearned revenue 5,750
Wage payable 11,600
Interest payable 1,250
Note payable 125,000
Total liabilities $145,050
Common stock 100,000
Retained earnings 29,068
Total Equity $129,068
Total liabilities and stockholders' equity $274,118

© Becker Professional Education Corporation. All rights reserved. Module 2 1–33 A.1
2 A.1. Financial Statements: Part 1 PART 1 UNIT 1

4.4 Statement of Cash Flows


The statement of cash flows, which is covered in detail in the next module, explains the change
in cash during the period. The cash flow statement is comprised of three main sections:
operating, investing, and financing. The statement of cash flows is prepared after the balance
sheet to reconcile the beginning and ending cash balance for the period.

Harris Enterprises
Statement of Cash Flows
For the Month Ended February 28

Operating Activities:
Net income $ 3,668
Add Back Noncash Items:
Depreciation expense 625
Changes in Current Assets & Liabilities:
Decrease in accounts receivable 4,160
Increase in prepaid insurance (7,583)
Decrease in supplies 4,100
Decrease in accounts payable (2,750)
Increase in wages payable 11,600
Increase in interest payable 1,250
Increase in unearned revenue 5,750
Cash flow from operating activities $20,820

Investing Activities:
Purchase of building (25,000)
Cash flow from investing activities $(25,000)

Financing Activities:
Issuance of stock 15,000
Payment of dividends (2,000)
Cash flows from financing activities $13,000

Increase/(Decrease) in Cash 8,820


Beginning cash balance, February 1 32,000
Ending cash balance, February 28 $ 40,820

1–34 Module 2 A.1. Financial


© Becker Professional Education Corporation. Statements:
All rights reserved.Part 1
PART 1 UNIT
2 1 A.1. Financial Statements: Part 1

5 Articulation of the Financial Statements LOS 1A1f

Each financial statement demonstrates a relationship with the balance sheet. The beginning
balances of the balance sheet from the previous period increase and decrease as a result of
the company's business transactions. These transactions are recorded and reported by the
accounting system and result in the recognition of assets, liabilities, owners' equity, revenues
and/or expenses. The board of directors will analyze reported income and assess whether any
dividends will be declared and paid by the company. Reported income less dividends results in
increase to retained earnings, which is reflected on the balance sheet.
The statement of cash flows reconciles the change in cash during the year and reverses the
effects of accrual accounting, which allows users of the financial statements to analyze the
sources and uses of cash.

Illustration 2 Relationship Between the Financial Statements

Income Statement
for the Year Ended December 31, Year 2
Net sales $1,530.0
Expenses $1,434.5
Net income $ 95.5

Statement of Retained Earnings


Balance Sheet for the Year Ended December 31, Year 2 Balance Sheet
as of December 31, Year 1 as of December 31, Year 2
Retained earnings, 01/01/Year 1 $439.2
Cash $ 30.0 Net income $ 95.5 Cash $ 40.0
All other assets $1,233.2 Dividends $ 36.7 All other assets $1,284.0
Total assets $1,263.2 Retained earnings, 12/31/Year 2 $498.0 Total assets $1,324.0
Total liabilities $ 652.0 Liabilities $ 632.0
Capital stock $ 172.0 Capital stock $ 194.0
Retained earnings $ 439.2 Statement of Cash Flows
for the Year Ended December 31, Year 2 Retained earnings $ 498.0
Total liabilities $1,263.2
Operating activities $ 11.5 Total liabilities $1,324.0
and equity
and equity
Investing activities $(92.8)
Financing activities $(14.7)
Net increase in cash $ 10.0
Beginning cash $ 30.0
Ending cash $ 40.0

© Becker Professional Education Corporation. All rights reserved. Module 2 1–35 A.1
2 A.1. Financial Statements: Part 1 PART 1 UNIT 1

Question 1 MCQ-12325

Lewis Industries reports the following at the end of the current year:
Service revenue $800,000
Administrative expenses 240,000
Loss on sale of investment (15,000)
Interest expense (7,000)
Interest income 8,000
Cost of services 310,000
Calculate the operating income for Lewis Industries.
a. $235,000
b. $236,000
c. $250,000
d. $490,000

Question 2 MCQ-12328

How are the primary revenue-generating activities of the reporting entity categorized on
the multistep income statement?
a. Included as other income and expense items.
b. Excluded from the calculation of net income.
c. Excluded on the income statement and shown as current assets on the balance sheet.
d. Included as operating revenues and expenses.

1–36 Module 2 A.1. Financial


© Becker Professional Education Corporation. Statements:
All rights reserved.Part 1
PART 1 UNIT
2 1 A.1. Financial Statements: Part 1

Question 3 MCQ-12333

Based on the account listing provided below, calculate the total dollar amount of current
assets to be reported on the balance sheet.
Sales revenue $157,000
Common stock 55,000
Dividends 12,000
Equipment 72,000
Cash 4,500
Salary expense 48,000
Inventory 12,000
Accounts receivable 8,000
Building 125,000
a. $36,500
b. $24,500
c. $96,500
d. $221,500

Question 4 MCQ-12340

Rainbow Writing Center provided Lewis Accountants with the following information related
to the current fiscal year-end:

Revenues $63,000
Stock issuances 10,000
Liabilities at end of fiscal year 43,000
Stockholders' equity at the beginning of the fiscal year 25,000
Stockholders' equity at the end of the fiscal year 57,000
Dividends 7,000

Calculate the net income of Rainbow Writing Center based on the information provided.
a. $32,000
b. $22,000
c. $29,000
d. Insufficient information to calculate

© Becker Professional Education Corporation. All rights reserved. Module 2 1–37 A.1
2 A.1. Financial Statements: Part 1 PART 1 UNIT 1

Question 5 MCQ-12338

Which of the following best articulates the relationship among the financial statements?
a. Each financial statement reports information unrelated to other financial statements.
b. Each financial statement presents financial information to dissect the company's
performance and establishes the relationship between the financial statements.
c. The balance sheet and income statement are the only two financial statements
that demonstrate a relationship.
d. The changes on the balance sheet are not represented on any other financial
statements prepared.

1–38 Module 2 A.1. Financial


© Becker Professional Education Corporation. Statements:
All rights reserved.Part 1
3
MODULE
PART 1 UNIT 1

A.1. Financial
Statements: Part 2
Part 1
Unit 1

This module covers the following content from the IMA Learning Outcome Statements.

CMA LOS Reference: Part 1—Section A.1. Financial Statements: Part 2

The candidate should be able to:


a. identify the users of these financial statements and their needs
b. demonstrate an understanding of the purposes and uses of each statement
c. identify the major components and classifications of each statement
d. identify the limitations of each financial statement
e. identify how various financial transactions affect the elements of each of the financial
statements and determine the proper classification of the transaction
f. demonstrate an understanding of the relationship among the financial statements
g. demonstrate an understanding of how a balance sheet, an income statement, a statement
of changes in equity, and a statement of cash flows (indirect method) are prepared
h. define integrated reporting (IR), integrated thinking, and the integrated report and
demonstrate an understanding of the relationship between them
i. identify the primary purpose of IR
j. explain the fundamental concepts of value creation, the six capitals, and the value
creation process
k. identify elements of an integrated report; i.e., organizational overview and external
environment, governance, business model, risks and opportunities, strategy and
resource allocation, performance, outlook, and basis of preparation and presentation
l. identify and explain the benefits and challenges of adopting IR

1 Statement of Cash Flows (SCF)

The statement of cash flows (SCF) is a required part of a full set of financial statements for
all business enterprises. The other three primary financial statements (income statement,
statement of stockholders' equity, and the balance sheet) are all prepared using the accrual
basis where revenues are recognized when performance obligations are satisfied, and expenses
are recognized when incurred. The SCF is prepared on a cash basis, reflecting an entity's cash
inflows and outflows for a period of time, and reconciling the change from beginning to ending
cash and cash equivalents reported on the balance sheet.

© Becker Professional Education Corporation. All rights reserved. Module 3 1–39 A.1
3 A.1. Financial Statements: Part 2 PART 1 UNIT 1

Illustration 1 Summarized Statement Format

X Company
Statement of Cash Flows
For the Year Ended December 31, Year 1

Net cash provided by (used in) operating activities $XXX


Net cash provided by (used in) investing activities XXX
Net cash provided by (used in) financing activities $XXX
Net increase (decrease) in cash $XXX
Cash and cash equivalents at beginning of year XXX
Cash and cash equivalents at end of year $XXX

LOS 1A1a 1.1 Purposes and Uses


LOS 1A1b The purpose of the SCF is to provide information about the sources of cash (i.e., cash receipts)
and cash equivalents and the uses (i.e., cash disbursements) of cash and cash equivalents. The
cash basis is used because investors, creditors, shareholders, vendors, regulators, taxing entites,
and other interested parties need information about the entity's available cash and cash needs
(i.e., ability to pay obligations, dividends, etc.). Information concerning an entity's cash inflows
and outflows during the period aids in the assessment of an entity's liquidity (how quickly items
are converted into cash) and solvency (ability to satisfy debt requirements). The statement can
also provide answers to questions such as:
How successful are the entity's operations at generating cash?
Why is there a difference between net income and operating cash flows?
Is the entity using investments to strengthen its operating capacity?
Will vendors receive payments in a timely manner?
Will the entity have the cash to make tax payments when they come due?
Is cash being generated to cover future dividend payments?
Why did the dividend payments not match the level of income earned?
Will the entity have enough cash available to cover its principal and interest debt obligations?
How much outside financing is it using to support operations?
How much cash was brought in because of operations as opposed to external financing by
using debt or equity, or both?
Was cash used to purchase treasury stock?
Is the entity building up its investment in long-term assets or liquidating them?
What is the possibility of generating future cash inflows?
Are those future cash flows able to satisfy the entity's obligations to its creditors?

1–40 Module 3 A.1. Financial


© Becker Professional Education Corporation. Statements:
All rights reserved.Part 2
PART 1 UNIT
3 1 A.1. Financial Statements: Part 2

1.2 Components and Classifications LOS 1A1c


The SCF includes three major sections: operating activities, investing activities, and financing LOS 1A1e
activities, and a separate schedule of significant noncash investing and financing activities.

1.2.1 Operating Activities


The operating activities section relates to transactions involved in the production of goods
and the delivery of services to customers. Operating activities consist of cash receipts and
disbursements from transactions reported on the income statement and related to current
assets and liabilities, except those classified as investing or financing activities. Examples include
sales of products/services, payments to employees, inventory purchases, payment and receipt of
interest, purchases and sales of trading securities (if classified as current), payments of account
payables, and receipts of accounts receivable.

1.2.2 Investing Activities


This section includes cash flows from the purchase or sale of non-current assets. Some
examples are:
making loans to other entities (cash outflow);
purchasing (cash outflow) or disposing of (cash inflow) trading securities (if classified as
non-current), available-for-sale securities, and held-to-maturity investment securities;
acquiring (cash outflow) or disposing of (cash inflow) property, plant, and equipment
(productive assets); and
acquiring another entity under the acquisition method using cash (cash outflow). The
payment for the acquisition is shown net of the cash acquired.

1.2.3 Financing Activities


Financing activities include cash flows from non-current liability (creditor-oriented) and equity
(owner-oriented) transactions. Examples include:
Non-current Liability (Creditor-Oriented) Activities
— Obtaining resources from creditors, such as issuing bonds, notes, and other borrowings
(cash inflow).
— Payments of principal (not interest, which is part of the operating activities section) on
amounts borrowed (cash outflow).
Equity (Owner-Oriented) Activities
— Obtaining resources from owners, such as issuing stock (cash inflow).
— Providing owners with a return on their investment, such as paying cash dividends or
repurchasing stock (cash outflow).

© Becker Professional Education Corporation. All rights reserved. Module 3 1–41 A.1
3 A.1. Financial Statements: Part 2 PART 1 UNIT 1

Pass Key

The three major sections of the SCF consist of information from the following financial
statements:
1. Operations: Income statement, balance sheet (current operating assets and current
operating liabilities)
2. Investing: Balance sheet (non-current assets)
3. Financing: Balance sheet (non-current liabilities, equity)

LOS 1A1e 1.2.4 Significant Noncash Investing and Financing Activities


Information about material noncash investing and financing activities (those that do not
result in cash receipts or payments) is provided separately in a supplemental disclosure. Any
part of the transaction that involves cash would be included in the SCF. Examples of noncash
activities include:
issuance of stock to purchase fixed assets;
conversion of bonds to equity instruments, which generally does not involve cash;
acquisition of assets through the incurrence of a finance lease obligation; and
exchange of one noncash asset for another noncash asset.

Example 1 Cash Flow Classifications

Facts: Dustbowl Corp. is preparing its statement of cash flows and is classifying several
transactions for the current year.
Required: Classify the following ten transactions into the appropriate categories on the
statement of cash flows. If the transaction will not appear on the statement, indicate
that fact.
1. Issued 500,000 shares of common stock at par value.
2. Sold land at cost for $475,000.
3. Accounts receivable increased by $120,000 during the year.
4. Purchased a machine for $750,000.
5. Sold $2,000,000 in bonds at face value.
6. Accounts payable decreased by $85,000 during the year.
7. Paid a short-term note payable of $325,000.
8. Purchased 50,000 shares of treasury stock.
9. Issued $500,000 in bonds at face value in exchange for a machine.
10. Declared a cash dividend of 25 cents per share.

(continued)

1–42 Module 3 A.1. Financial


© Becker Professional Education Corporation. Statements:
All rights reserved.Part 2
PART 1 UNIT
3 1 A.1. Financial Statements: Part 2

(continued)

Solution:

Statement of Cash Flows


Transaction Classification
1. Issued 500,000 shares of common stock at Financing activities
par value.
2. Sold land at cost for $475,000. Investing activities
3. Accounts receivable increased by $120,000 Operating activities
during the year.
4. Purchased a machine for $750,000. Investing activities
5. Sold $2,000,000 in bonds at face value. Financing activities
6. Accounts payable decreased by $85,000 Operating activities
during the year.
7. Paid a short-term note payable of $325,000. Financing activities
8. Purchased 50,000 shares of treasury stock. Financing activities
9. Issued $500,000 in bonds at face value in Significant noncash investing and
exchange for a machine. financing activities (no cash involved)
10. Declared a cash dividend of 25 cents Not included (the dividend will appear
per share. once it is paid)

1.3 Limitations LOS 1A1d

A limitation of the SCF is that it is prepared using the cash basis, which does not match revenues
with the expenses used to generate those revenues as effectively as an accrual basis accounting
system. To overcome this limitation, it is important to analyze the statement of cash flows along
with the balance sheet and income statement to present a clearer picture of operating results.
Another limitation is the extent to which management decisions are not fully reflected in the
SCF. For example, waiting to pay vendors for credit purchases instead of paying earlier and
potentially receiving a discount increases cash flow from operations in the short-term but may
be harmful in the future. The issuance of debt or equity provides an immediate infusion of cash,
but the SCF does not reflect the obligation for interest, principal repayments, and dividends
owed in the future. An investment in property, plant, and equipment should ultimately generate
positive cash flows, but in the short-term, it will just appear as a cash outflow from investing.

1.4 Preparation of the Statement of Cash Flows LOS 1A1f

The SCF may be prepared using either the direct method or the indirect method. The only LOS 1A1g
difference between these two methods is the format of the operating activities section. The
direct method separates the cash disbursements from the cash receipts and finds the difference
between the receipts and disbursements to determine the cash flow from operating activities.
The indirect method begins with accrual basis net income and adjusts it for various items to
convert to cash basis cash flows from operating activities. Both methods result in the amount for
cash flows from operating activities.

© Becker Professional Education Corporation. All rights reserved. Module 3 1–43 A.1
3 A.1. Financial Statements: Part 2 PART 1 UNIT 1

Pass Key

On the CMA Exam, only the indirect method of preparing the SCF is tested.

Preparing the SCF requires gathering information from the three other financial statements.
Income Statement: The indirect method begins with net income and removes the effects
of noncash items found on the income statement, such as depreciation and amortization
expenses, and gains and losses related to investing activities and then adjusts the reporting
from accrual basis to cash basis.
Statement of Shareholders' Equity: Provides important information as to dividends and
other equity transactions that may have occurred.
Comparative Balance Sheet: A comparative balance sheet has multiple periods of data.
This is crucial because it allows the changes that occurred in all of the balance sheet
accounts during the period to be calculated. The balance sheet also provides the cash and
cash equivalent balances to which the statement of cash flows must reconcile.
Footnotes to the Financial Statements: The footnotes provide information on material
noncash transactions. For example, the exchange of assets in a noncash transaction would
be important information disclosed in the footnotes but not detectable from examining the
changes in the account balances on the comparative balance sheet.

1.4.1 Determining Cash Flows From Operating Activities


The indirect method starts with net income from the income statement. The net income
reported by an entity is unlikely to equal cash flow from operations. The indirect method
operating section of the SCF shows the reconciliation of accrual basis net income to cash basis
cash flow from operating activities.
Step 1: Adjustments to Net Income
The indirect method operating section starts by adjusting net income for:
Noncash expenses reported on the income statement (add noncash expenses)
Some expenses have no effect on cash. For example, depreciation expense is a noncash
expense (i.e., it does not represent an outflow of cash). However, depreciation expense does
reduce the amount of net income reported by an entity. Therefore, to determine the cash
flows from operating activities, depreciation, amortization, and other noncash expenses are
added back to net income.
Gains and losses (subtract gains and add losses)
After adding to net income the amount of noncash expenses, the effects of gains and
losses are eliminated by subtracting all gains and adding losses. Gains and losses relate to
investing activities and should not be included in cash flows from operating activities.
For example, if land that cost $20,000 is sold for $23,000, a $3,000 gain on the sale of land
is recognized on the income statement. In the investing section of the SCF this transaction
is recorded as a $23,000 cash inflow. The net income on the income statement includes
the $3,000 gain, which must be reversed (subtracted) when calculating cash flows from
operations so the cash flows from the sale of land is reported only in the investing section.

1–44 Module 3 A.1. Financial


© Becker Professional Education Corporation. Statements:
All rights reserved.Part 2
PART 1 UNIT
3 1 A.1. Financial Statements: Part 2

Step 2: Changes in Current Operating Assets and Current Operating Liabilities


Net income is then adjusted for changes in current asset and current liability balances during
the period. The effect of these increases and decreases are determined as follows:
An increase to a current operating asset is treated as a decrease in cash and is subtracted
from net income. For example, if inventory increases during the period, costs incurred to
acquire or manufacture the inventory remain on the balance sheet and do not result in
revenue until the inventory is sold.
A decrease to a current operating asset is treated as an increase in cash and is added to
net income. For example, if inventory decreases during the period, the assumption is that
inventory was sold and the sales will produce cash inflows for the company.
An increase to a current operating liability is treated as an increase in cash and is added to
net income. For example, if accounts payable increases during the period, the cash paid to
vendors is less than the related expenses reported on the income statement.
A decrease to a current operating liability is treated as a decrease in cash and is subtracted
from net income. If accounts payable decreases during the period, the cash paid to vendors
is greater than the related expenses reported on the income statement because it includes
cash paid for expenses incurred in prior periods.
The operating activities adjustments for noncash expenses, gains/losses, and current operating
assets/current operating liabilities are totaled and added to or subtracted from net income
to determine the net cash provided by (positive amount) or used in (negative amount)
operating activities.

Pass Key

You will be able to easily remember the adjustments made to the operating activities
section under the indirect method by remembering the mnemonic CLAD.
——Current assets and liabilities
——Losses and gains
——Amortization and depreciation
——Deferred items

© Becker Professional Education Corporation. All rights reserved. Module 3 1–45 A.1
3 A.1. Financial Statements: Part 2 PART 1 UNIT 1

Example 2 Cash Flow From Operations

Facts: Fryar Corp., a wholesaler-retailer, is in the process of preparing its statement of cash
flows using the following balance sheet and income statement information for Year 2.

Balance Sheet Year 2 Year 1


Cash $ 165,520 $ 80,000
Accounts receivable, net 50,000 45,000
Inventory 78,000 92,000
Prepaid insurance 40,000 32,000
Marketable securities 62,000 30,000
Land 150,000 50,000
Buildings 365,000 485,000
ROU asset 125,000 –
Equipment 320,000 350,000
Accumulated depreciation (25,805) (65,000)
Patents 50,000 50,000
Total assets 1,379,715 1,149,000
Accounts payable 35,000 25,000
Wages payable 20,000 25,000
Notes payable 115,000 15,000
Lease liability 125,000 –
Long-term debt 325,000 365,000
Total liabilities 620,000 430,000
Common stock 500,000 500,000
Retained earnings 259,715 219,000
Total equity 759,715 719,000
Total liabilities and equity 1,379,715 1,149,000

Income Statement Year 2


Sales $ 242,000
Cost of goods sold 140,000
Operating expenses* 47,805
Loss on bond retirement (5,000)
Gain on equipment sale 15,000
Pretax income 64,195
Net income $ 50,715
* Salaries $35,000, Depreciation $10,805, Insurance $2,000

(continued)

1–46 Module 3 A.1. Financial


© Becker Professional Education Corporation. Statements:
All rights reserved.Part 2
PART 1 UNIT
3 1 A.1. Financial Statements: Part 2

(continued)

Required: Prepare the operating section of the statement of cash flows using the
indirect method.
Solution:

Cash Flow From Operating Activities


Net income $50,715
Adjustments to reconcile net income to net cash
provided by operating activities:
Depreciation and amortization 10,805
Gain on sale of equipment (15,000)
Loss on bond retirement 5,000
Change in current assets and current liabilities:
Increase in accounts receivable, net (5,000)
Decrease in inventory 14,000
Increase in prepaid insurance (8,000)
Increase in accounts payable 10,000
Decrease in wages payable (5,000)
Total adjustments 6,805
Net cash provided by operating activities $57,520

1.4.2 Determining Cash Flows From Investing Activities


Investing activities include cash flows from the purchase or sale of non-current assets, including
property, plant, and equipment, investments in trading securities classified as non-current, all
available-for-sale securities and held-to-maturity securities, and acquisitions of other entities.
Purchases are cash outflows and sales are cash inflows. The total of cash flows from investing
activities is labeled "net cash provided by investing activities" when positive and labeled "net
cash used by investing activities" when negative.

1.4.3 Determining Cash Flows From Financing Activities


Financing activities include cash flows from non-current liability and equity transactions.
Proceeds from issuing stock, bonds, notes, or other borrowings are cash inflows. Paying
dividends, repurchasing stock, and repaying principal on bonds or notes are cash outflows. The
total cash flows from financing activities is labeled "net cash provided by financing activities"
when positive and "net cash used by financing activities" when negative.

© Becker Professional Education Corporation. All rights reserved. Module 3 1–47 A.1
3 A.1. Financial Statements: Part 2 PART 1 UNIT 1

Example 3 Cash Flows From Investing and Financing

Facts: Fryar Corp. had the following investing and financing transactions during the period:
1. Equipment was sold that had a cost of $30,000 and accumulated depreciation of
$10,000. The equipment was sold for a $15,000 gain.
2. A building was sold at the beginning of the year at book value. The building had an
original cost of $120,000 with $40,000 of accumulated depreciation.
3. A note payable was issued in exchange for land in the amount of $100,000.
4. The company borrowed $60,000 by issuing a long-term note payable.
5. A $5,000 loss was recorded from the retirement of bonds, which the company had
previously issued. The net carrying value on the books at the time of retirement was
$100,000.
6. A purchase of held-to-maturity securities for $55,000 was made at the end of the
current year. The face value of these securities was $50,000.
7. A finance lease obligation was used to acquire equipment on the last day of the year.
The present value is equal to $125,000 and no down payment was required.
8. Sale of trading securities at $23,000 originally purchased at face value for $20,000.
9. Declaration and payment of a $10,000 cash dividend to common stockholders.
Required: Prepare the investing activities section and the financing activities section for the
year. Assume that investments in trading securities are classified as non-current assets.
Solution:

Cash Flow From Investing Activities


Proceeds from the sale of equipment 35,000 (1)
Proceeds from the sale of a building 80,000 (2)
Payment for the purchase of held-to-maturity securities (55,000) (6)
Proceeds from the sale of trading securities 23,000 (8)
Net cash provided by investing activities $ 83,000

Cash Flow From Financing Activities


Proceeds from issuance of long-term note 60,000 (4)
Payment for retirement of bonds (105,000) (5)
Payment of dividends to common stockholders (10,000) (9)
Net cash used by financing activities $(55,000)

(continued)

1–48 Module 3 A.1. Financial


© Becker Professional Education Corporation. Statements:
All rights reserved.Part 2
PART 1 UNIT
3 1 A.1. Financial Statements: Part 2

(continued)

(1)
 he equipment sold had a book value of $20,000 ($30,000 original cost − $10,000
T
accumulated depreciation). The $15,000 gain is the amount in excess of the equipment's
book value. The $35,000 sales price is the sum of the $20,000 book value + $15,000 gain.
(2)
 he book value (and the sales price) of the building was $80,000 ($120,000 original cost −
T
$40,000 accumulated depreciation).
(3)
 his transaction is a noncash transaction, which is reflected in its own section (see the
T
next example).
(4) 
The long-term note of $60,000 provided a cash inflow.
(5) 
The amount paid to retire the bonds was $105,000 ($100,000 face value + $5,000 loss
to retire).
(6) 
The purchase of $55,000 in held-to-maturity securities is treated as an investing outflow.
(7) 
This transaction is a noncash transaction, which is reflected in its own section (see the
next example).
(8) 
The proceeds from the sale of the trading securities (assumed to be non-current) are
treated as a cash inflow from investing.
(9)
 he payment of dividends to common stockholders is a $10,000 cash outflow from
T
financing. Note that this will also reflect as a reduction to retained earnings. The change
in retained earnings of $40,715 is equal to net income of $50,715 less the dividend paid
of $10,000.

1.4.4 Net Change in Cash


Once all three sections have been prepared, the totals are totaled to show either a net increase
in cash or a net decrease in cash. The net change is added (net increase) or subtracted (net
decrease) from the beginning cash balance reported on the previous period's balance sheet and
compared to the ending cash balance from the balance sheet. The net increase or decrease in
cash for the period must equal the change (beginning of year to end of year) in the cash and
cash equivalents line item on the balance sheet.

1.4.5 Significant Noncash Investing and Financing Activities


Information about material noncash investing and financing activities should be provided in a
supplemental disclosure to the statement of cash flows.

© Becker Professional Education Corporation. All rights reserved. Module 3 1–49 A.1
3 A.1. Financial Statements: Part 2 PART 1 UNIT 1

Example 4 Noncash Investing and Financing Activities

Facts: Fryar Corp. had the following investing and financing transactions during the period:
1. Equipment was sold that had a cost of $30,000 and accumulated depreciation of
$10,000. The equipment was sold for a $15,000 gain.
2. A building was sold at the beginning of the year at book value. The building had an
original cost of $120,000 with $40,000 of accumulated depreciation.
3. A note payable was issued in exchange for land in the amount of $100,000.
4. The company borrowed $60,000 by issuing a long-term note payable.
5. A $5,000 loss was recorded from the retirement of bonds, which the company had
previously issued. The net carrying value on the books at the time of retirement
was $100,000.
6. A purchase of held-to-maturity securities for $55,000 was made at the end of the
current year. The face value of these securities was $50,000.
7. A finance lease obligation was used to acquire equipment on the last day of the year.
The present value is equal to $125,000 and no down payment was required.
8. Sale of trading securities at $23,000 originally purchased at face value for $20,000.
9. Declaration and payment of a $10,000 cash dividend to common stockholders.
Required: Prepare the significant noncash investing and financing activities section.
Solution: The components of the significant noncash investing and financing activities
section include the following:
3. A note payable was issued in exchange for land in the amount of $100,000.
7. A finance lease obligation was used to acquire equipment on the last day of the year.
The present value is equal to $125,000 and no down payment was required.
Supplemental Disclosures of Cash Flow Information
Issuance of note payable in exchange for land $100,000
Finance lease obligation used to acquire equipment $125,000

1–50 Module 3 A.1. Financial


© Becker Professional Education Corporation. Statements:
All rights reserved.Part 2
PART 1 UNIT
3 1 A.1. Financial Statements: Part 2

The following table summarizes the statement of cash flow classifications of individual
transactions under U.S. GAAP:

Operating Investing Financing No Net


Cash Flow Cash Flow Cash Flow Cash
Transaction (CFO) (CFI) (CFF) Flow

Selling products / collecting receivables 


Purchasing inventory / paying vendors 
Purchasing supplies and services / paying vendors 
Paying taxes 
Purchasing / selling trading securities (general rule) 
Purchasing long-term assets or long‑term investments
for cash 
Recording depreciation, amortization, or depletion 
Collecting interest on an investment 
Collecting dividends on an investment 
Recording income of equity method affiliates 
Selling long-term assets or investments (noncash
equivalents and nontrading securities): cash proceeds 
Borrowing funds (e.g., bank loans, issuing debt) 
Paying interest on debt 
Paying principal on debt 
Issuing common or preferred stock 
Paying dividends on common or preferred stock 
Repurchasing stock (e.g., treasury shares) 
Prepaying debt or paying debt extinguishment costs 

© Becker Professional Education Corporation. All rights reserved. Module 3 1–51 A.1
3 A.1. Financial Statements: Part 2 PART 1 UNIT 1

Illustration 2 Statement of Cash Flows

Based on the previous examples, the following is the statement of cash flows for
Fryar Corp.

Cash Flow From Operating Activities


Net income $ 50,715
Adjustments to reconcile net income to net cash
provided by operating activities:
Depreciation and amortization 10,805
Gain on sale of equipment (15,000)
Loss on bond retirement 5,000
Change in current assets and current liabilities:
Increase in accounts receivable, net (5,000)
Decrease in inventory 14,000
Increase in prepaid insurance (8,000)
Increase in accounts payable 10,000
Decrease in wages payable (5,000)
Total adjustments 6,805
Net cash provided by operating activities $ 57,520

Cash Flow From Investing Activities


Proceeds from the sale of equipment 35,000
Proceeds from the sale of a building 80,000
Payment for the purchase of held-to-maturity securities (55,000)
Proceeds from the sale of trading securities 23,000
Net cash provided by investing activities $ 83,000

Cash Flow From Financing Activities


Proceeds from issuance of long-term note 60,000
Payment for retirement of bonds (105,000)
Payment of dividends to common stockholders (10,000)
Net cash used by financing activities $ (55,000)

Net Decrease in Cash and Cash Equivalents $ 85,520

Cash and cash equivalents at the beginning of the year 80,000


Cash and cash equivalents at the end of the year $165,520

Supplemental Disclosures of Cash Flow Information


Issuance of note payable in exchange for land $100,000
Finance lease obligation used to acquire equipment $125,000

1–52 Module 3 A.1. Financial


© Becker Professional Education Corporation. Statements:
All rights reserved.Part 2
PART 1 UNIT
3 1 A.1. Financial Statements: Part 2

2 Integrated Reporting LOS 1A1h

The International Integrated Reporting Council (IIRC) is a global coalition of regulators, investors,
companies, standard setters, the accounting profession, and nongovernmental organizations
("NGOs"). The IIRC first published the Integrated Reporting framework in 2013, which was
designed to clarify the effect of nonfinancial performance on financial performance over short-,
medium- and long-term time horizons. The framework states that Integrated Reporting (IR)
"promotes a more cohesive and efficient approach to corporate reporting and aims to improve
the quality of information available to providers of financial capital to enable a more efficient
and productive allocation of capital." The guidelines also stated that IR aims to do the following:
improve the quality of information available to providers of financial capital to enable a
more efficient and productive allocation of capital;
promote a more cohesive and efficient approach to corporate reporting that communicates
the full range of factors that materially affect the ability of an organization to create value
over time;
enhance accountability and stewardship for the broad base of capitals (financial,
manufactured, intellectual, human, social and relationship, and natural) and promote
understanding of their interdependencies; and
support integrated thinking, decision making, and actions that focus on the creation of value
over the short-, medium-, and long-term.

2.1 Integrated Thinking


Integrated thinking is defined in the International Integrated Reporting Council (IIRC) framework
of 2013 as “…the active consideration by an organization of the relationships between its various
operating and functional units and the capitals that the organization uses or affects. Integrated
thinking leads to integrated decision-making and actions that consider the creation of value over the
short, medium and long term.”
Management should take into account the interconnectivity and integration of the various
factors that together can help create value. These factors include the capitals that the
organization uses or influences, the capabilities of the organization in putting all of the factors to
work together, the strategies an organization sets to grasp opportunities in the market, and the
response to the various risks the organization confronts.

2.2 The Integrated Report


The integrated report is defined in the International Integrated Reporting Council (IIRC)
framework of 2013 as: “… a concise communication about how an organization's strategy,
governance, performance and prospects, in the context of its external environment, lead to the
creation of value over the short, medium and long term.”
When management uses integrated thinking, it is better equipped with data collection tools,
data analytics, and information systems to capture both financial and nonfinancial information
for integrated reporting. Integrated thinking allows managers to understand and properly utilize
the available information in creating shared value that is beneficial to all.

© Becker Professional Education Corporation. All rights reserved. Module 3 1–53 A.1
3 A.1. Financial Statements: Part 2 PART 1 UNIT 1

LOS 1A1i 2.3 Historical Context


Historically, many corporations focused on short-term financial performance measures. As such
corporate operations focused on maximizing shareholder wealth while overlooking negative
effects on the environment, communities, social aspects, and economic sustainability to the
detriment of their communities and of the world in general. Integrated reporting systems
shifted corporate focus from narrow, short-term financial performance to broader, long-term
performance measures.
Many corporations now focus on creating value to shareholders, society, and the communities in
which they operate. These corporations endeavor to create value for owners but do not ignore
the corporate role in benefiting the community. Creating shared value is a tool for sustainability
and long-term survival. Corporations should operate in a manner that achieves shareholder
goals as well as contributing to the society as a whole. While financial reports present historical
financial information, they provide an incomplete "picture" of the corporation's impact on
communities. Integrated reporting, on the other hand, includes important nonfinancial aspects
of the future potential to create value to the community.

With permission from the International Integrated Reporting Council © 2020.

The primary purpose of IR is to report to stakeholders how an organization creates value


over time. Corporations must create value for all stakeholders, not just for shareholders.
Stakeholders include the following groups:
Owners
Creditors
Employees
Suppliers
Society
Customers
Governments

LOS 1A1j 2.4 Value Creation


The three underlying concepts that serve as the foundation for integrated reporting include the
value creation concept, the six capitals, and the value creation process.

2.4.1 The Value Creation Concept


Value creation is the process of transforming inputs into outputs with higher values than the
inputs used to create them. Through an organization's activities and the use of the six capitals, an
organization can create value, thereby increasing and improving the financial capital of the owners.
The value created by the organization includes the following:
Value created for the owners or financial providers in general
Value created for the community and the society at large

2.4.2 The Six Capitals


In the process of creating value, organizations depend on six different types of capital: financial,
manufactured, intellectual, human, social and relationship, and natural. These six capitals can be
transformed. For example, an organization may spend money to develop the skills of its employees.
Doing so is a financial expense that reduces financial capital but improves human capital.

1–54 Module 3 A.1. Financial


© Becker Professional Education Corporation. Statements:
All rights reserved.Part 2
PART 1 UNIT
3 1 A.1. Financial Statements: Part 2

The International Integrated Reporting Council (IIRC) framework describes the six capitals as follows:
Financial Capital: The pool of funds available to an organization for use in the production
of goods or the provision of services. These funds change over time and are obtained
through financing, such as debt, equity or grants, and/or are generated through operations
or investments. Additional investments by owners and/or profits generated from operations
and reinvested cause financial capital to increase.
Manufactured Capital: The manufactured physical objects (as distinct from natural
physical objects) that are available to an organization for use in the production of goods or
the provision of services, including property, plant, and equipment. Manufactured capital
includes all infrastructures available such as bridges, roads, water supply, and wastewater
treatment facilities. Manufactured capital includes not only those created or manufactured
by other organizations but also those assets created and manufactured by the organization
itself for sale and/or for self-use.
Intellectual Capital: All intellectual property created by the employees of the organization,
including the knowledge employees use to create knowledge-based intangibles such as
patents, copyrights, software, licenses, and other rights that can be used by the organization
to create value. Intellectual capital also includes organizational capital such as policies,
procedures, systems, and protocols.
Human Capital: Human capabilities, skills, and experience, including employees' motivation
to innovate; their alignment with and support for an organization's governance framework,
risk management approach, and ethical values; the employees' ability to understand,
develop, and implement an organization's strategy; and their ability to lead, manage, and
collaborate for the benefit of improving processes, goods, and services.
Social and Relationship Capital: Includes the organization and its relationships within and
between communities, groups of shareholders, and other networks, and the ability to share
information for the best interest of all stakeholders. Social and relationship capital includes
the norms, values, behaviors, trust, and all the related intangible benefits the organization
has created through its brand and reputation.
Natural Capital: The renewable and nonrenewable environmental resources and processes
that provide goods or services supporting the past, current, or future prosperity of an
organization. Natural capital includes air, water, land, minerals, and forests as well as
biodiversity and ecosystem health.

Pass Key

Remember HMS FIN to recall all six capitals:


——Human
——Manufactured
——Social and relationship

——Financial
——Intellectual
——Natural

All six capitals aid the organization in creating shared value.

© Becker Professional Education Corporation. All rights reserved. Module 3 1–55 A.1
3 A.1. Financial Statements: Part 2 PART 1 UNIT 1

2.4.3 Value Creation Process


When an organization creates value for other stakeholders, value is created for owners as well.
Value creation considers the following:
External Environment: Includes the economic conditions, technological change, societal issues,
and environmental challenges that set the context within which the organization operates.
Mission and Vision: Mission and vision should be set with clear intentions and concise terms.
Governance: Those charged with governance (the board of directors and committees of the
2. FUNDAMENTAL CONCEPTS CONTINUED
board) are responsible for overseeing operations to ensure that the organization is creating
shared value.
Outputs and Outcomes: The organization's operations should make use of the available
capitals and transform them into goods and services. To continuously create value, those
2D The value creation process 2.23 At the core of the organization is its business
charged with governance must ensure the business model, model'swhichadaptability
draws on various to changes
capitals as inputsin
the2.20
society. Outcomes
The value areisthe
creation process internal
depicted in and external and,
consequences (whether
through its business positive
activities, converts themor
Figure 2. It is explained briefly in the following
negative) resulting from business
paragraphs, which also identify how the
activities. to outputs (products, services, by-products and
waste). The organization’s activities and its
Business Activities:
components Include
of Figure theinplanning,
2 (underlined the text) design, and manufacturing
outputs lead to outcomes inof goods
terms and
of effects on the
align with the Content Elements in Chapter 4.
provision of services utilizing the skills, intellectual abilities, and
capitals. The experiences
capacity of the businessof theto
model
2.21 The external adapt to changes (e.g., in the availability, quality
employees of theenvironment, includingInnovation
organization. economic in methods of production and delivery, better
and affordability of inputs) can affect the
conditions, technological change, societal issues
use of available technology, and efficiencies
and environmental challenges, sets the context in production reduce
organization’s longernegative effects
term viability. on the
(See Content
community
within and
which society.
the organization operates. The Element 4C Business model.)
mission and vision encompass the whole 2.24 Business activities include the planning, design
Continuous Monitoring:
organization, identifying itsRequired
purpose and to understand changes in the external environment that
and manufacture of products or the deployment
can create new
intention opportunities
in clear, concise terms. or(Seeadditional
Content risks. Management must reallocate
of specialized skills and knowledge in theresources to
maximize opportunities
Element 4A Organizational and to mitigate
overview and risk. provision of services. Encouraging a culture of
external environment.) innovation is often a key business activity in terms
Information Collection: To facilitate decision making,
2.22 Those charged with governance are responsible
management
of generating must
new products define
and services that
performance information
for creating an appropriateto be collected
oversight structure to and methods of collection.
anticipate customer demand, introducing
support the ability of the organization to create efficiencies and better use of technology,
Outlook: The value creation process is never static; substituting
value. (See Content Element 4B Governance.)
management shouldadverse
inputs to minimize monitor socialthe
or
process to determine if changes are needed. environmental effects, and finding alternative
uses for outputs.
The graphic below, created by the IIRC as part of the IR framework, illustrates the value creation
process. Note the six capitals on the left (inputs) and the six outcomes on the right.
Figure 2: The value creation process:

Source: With permission from the International Integrated Reporting Council © 2020.

www.theiirc.org The International <IR> Framework 13


1–56 Module 3 A.1. Financial
© Becker Professional Education Corporation. Statements:
All rights reserved.Part 2
PART 1 UNIT
3 1 A.1. Financial Statements: Part 2

2.5 Elements of the Integrated Report LOS 1A1k

The integrated report includes the following eight elements:


Organizational overview and external environment
Governance
Business model
Risks and opportunities
Strategy and resource allocation
Performance
Outlook
Basis of preparation and presentation

2.5.1 Organizational Overview and External Environment


This section defines what the organization does and the circumstances under which it operates.
This section includes the following:
The mission and the vision of the organization
Information about the culture, ethics, values, ownership, and operating structure
Information about principal activities and markets, including a description of the competitive
framework and the organization positioning in the market
The competition and the competitive structure of the market and the competitive advantage
of the organization, including discussion of the bargaining power of customers and
suppliers and the position of the organization within the value chain
Quantitative information about the employees' statistics, revenues, and their change over
time and countries in which the organization operates in
The external environment including the legal, commercial, social, environmental, and
political context that affects the organization and its ability to create value
Macroeconomic and microeconomic conditions such as stability and industry trends

2.5.2 Governance
This section describes the organization's governance structure and how that structure supports
value creation over the short-, medium-, and long-term, including the following:
The organization leadership structure, including its skills and diversity and whether the
governance structure is influenced by the regulatory requirements
The processes used to make strategic decisions, including attitudes towards risk and
mechanisms for addressing integrity and ethical issues
Actions taken by those charged with governance to influence and monitor strategic direction
of the organization
The culture, ethics, and values of the organization and how they have influenced the use of
the capitals available
The organization's implementation, if any, of governance practices that meet or exceed legal
requirements
The enablement of innovation, if any, by those charged with governance
Linkage, if any, of remuneration to value creation in the short-, medium- and long-term

© Becker Professional Education Corporation. All rights reserved. Module 3 1–57 A.1
3 A.1. Financial Statements: Part 2 PART 1 UNIT 1

2.5.3 Business Model


This section includes information about the organization's business model and how the organization
transforms inputs into outputs (products and services) into internal outcomes (such as employee
morale, reputation, revenues, and cash flows), and into external outcomes (such as customer
satisfaction, tax payments, environmental effects) through the organization's business model.

2.5.4 Risks and Opportunities


This section addresses specific risks and opportunities affecting the organization's ability
to create value over the short-, medium- and long-term. This section also addresses the
organization's ability to respond to the identified risks and opportunities.

2.5.5 Strategy and Resource Allocation


This section requires the organization to identify strategic direction and plans by following:
its short-, medium-, and long-term objectives;
its resource allocation plans to achieve the objectives;
the measures used to evaluate the achievement of the short-, medium-, and long-term
goals; and
the methods the organization employs to respond to changes in the external environment,
the benefits the organization obtains from the available opportunities, and the policies and
procedures the organization has implemented to respond to risks.

2.5.6 Performance
This section covers the extent to which the organization has achieved strategic objectives and
the effects on the six capitals, including the following:
Qualitative and quantitative information about performance, including a discussion of the
organization's effect on the capitals and the state of key stakeholders' relationships
Linkages between past and current performance and between current performance and the
organization's forecast
Key performance indicators (KPIs)
Effect of regulations on performance

2.5.7 Outlook
This section covers challenges and uncertainties the organization is likely to encounter in
pursuing its strategy, and includes the following:
Highlights about future changes, such as expectations about the external environment in
the short-, medium-, and long-term
The impact these changes will have on the organization and whether the organization is
equipped to respond to these changes
A discussion of the potential effect of the external environment on the achievement of
strategic objectives

2.5.8 Basis of Preparation and Presentation


This section includes a description of how an organization sets materiality levels and what
constitutes material information for the purpose of the integrated report. Also included is a
description of the process used by management to identify relevant matters for inclusion in the
report and to evaluate the importance of the relevant matters in determining their materiality.

1–58 Module 3 A.1. Financial


© Becker Professional Education Corporation. Statements:
All rights reserved.Part 2
PART 1 UNIT
3 1 A.1. Financial Statements: Part 2

The key principles set by the International Integrated Reporting Council (IIRC) for the preparation
of the integrated report are as follows:
Strategic Focus and Future Orientation: A focus on how strategy can help create value in the
short-, medium-, and long-term and an explanation of the use of and effect on the six capitals.
Connectivity of Information: Interdependence of the factors that affect the ability of the
organization to create shared values.
Stakeholder Relationship: The organization's understanding of stakeholders' legitimate
needs and interests and how the organization responds to those needs.
Materiality: Inclusion of information that substantially affects the organization's ability to
create value in the future.
Conciseness: The report should be concise.
Reliability and Completeness: Inclusion of all material matters, whether positive or
negative, without misstatements or errors.
Consistency and Comparability: Information presented in a manner consistent over time
and comparable to other organizations.

2.6 Benefits and Challenges of Adopting the Integrated Report LOS 1A1l

Implementation of IR is a significant undertaking with myriad associated benefits, costs, and


challenges. The Institute of Management Accountants lists many of the benefits, costs, and
challenges of implementing integrated reporting in organizations.

2.6.1 Benefits of Adopting IR


The benefits of adopting IR include the following:
Serving as a form of discipline for a company; ensuring the company concisely reports
material information in a manner that shows how well the company is performing in
nonfinancial dimensions affecting the quality of the company's formulated strategy and the
execution of that strategy.
Enhancing companies' needs to communicate nonfinancial performance in the context of
strategy and value creation.
Requiring managers to think about when and under what conditions trade-offs and
interdependencies between financial and nonfinancial performance arise.
Improving internal measurement and control systems for producing reliable and timely
nonfinancial information.
Mandating companies to increase the quality of information systems as well as the internal
controls and monitoring systems.
Reducing the expectation-reality gap between the company and external parties by
communicating in a holistic and transparent way about the performance, position, philosophy,
vision, and mission of an organization in both financial terms and sustainability terms.
Serving as a platform for improved dialogue, engagement, and relationships with all
stakeholders.
Engendering in customers who care about sustainability more commitment and loyalty to
the company's products and/or services.
Promoting higher employee engagement through internal coordination and collaboration.
Attracting new long-term investors by communicating a company's vision of the future and how
that vision addresses nonfinancial challenges and opportunities enhancing the confidence of
investors in the leadership of the company and the company's ability to build sustainable value.

© Becker Professional Education Corporation. All rights reserved. Module 3 1–59 A.1
3 A.1. Financial Statements: Part 2 PART 1 UNIT 1

2.6.2 Costs and Challenges of Adopting IR


The challenges of adopting IR include the following:
Collecting and analyzing both structured data and unstructured data.
Requiring infrastructure investments in new information systems and data sets.
Establishing new processes and control systems, dedicating resources, and obtaining
assurance from third parties.
Requiring the support of the board of directors and of the CEO for the adoption of
integrated reporting.
Employing needs and investment in skilled experts to interpret and explain this data and to
incorporate that data in financial reporting.
Disclosing more information to competition about the organization.
Lacking guidance and generally accepted standards for measuring and reporting
nonfinancial information.
Understanding the material issues that should be reported.
Assuring that the reports are reliable and comparable. Meeting this challenge is costly and
imposes the need for appropriate auditing standards to deal with these types of reports.
Ensuring data quality.

Question 1 MCQ-12349

Pat James is the CFO for Smallships Inc. James is concerned as year-end approaches that
the company will report negative cash flow from operations for the current year. What
action can James take to avoid or reduce negative cash flow from operations for the
current year?
a. Issue 50,000 new shares of common stock.
b. Reduce the allowance for doubtful accounts on aging receivables.
c. Retire at par 10,000 5-year 4.50 percent callable bonds and issue at par 10,000
new 5-year bonds at 3.50 percent.
d. Sell equity trading securities and use the proceeds to purchase held-to-maturity
debt securities.

1–60 Module 3 A.1. Financial


© Becker Professional Education Corporation. Statements:
All rights reserved.Part 2
PART 1 UNIT
3 1 A.1. Financial Statements: Part 2

Question 2 MCQ-12352

The previous year's income statement for Seessa Enterprises reflected net income of
$220,000, depreciation of $35,000, and a gain on the sale of a fixed asset of $18,000.
Seessa's balance sheet covering the same period reflects the following:
yyAccounts receivable increased from $35,000 to $39,000.
yyAccounts payable decreased from $22,000 to $19,000.
yyInventory decreased from $84,500 to $79,700.
yyInterest payments on debt issued at par totaled $65,000.
yyPrincipal payments totaled $125,000.
yyLong-term liabilities increased from $45,000 to $50,000.
yyThe market value of trading securities decreased from $29,400 to $24,900.
Based on the information provided above, Seessa's cash flow from operations using the
indirect method is closest to:
a. $239,300.
b. $275,300.
c. $304,300.
d. $309,300.

Question 3 MCQ-12354

Which of the following is unlikely to be included in an integrated report?


a. Annual financial statements.
b. Management discussion and analysis.
c. Variance analysis reports prepared for management decision making.
d. An assessment of the external environment and its effect on company strategy.

© Becker Professional Education Corporation. All rights reserved. Module 3 1–61 A.1
3 A.1. Financial Statements: Part 2 PART 1 UNIT 1

Question 4 MCQ-12356

The organization's operations make use of available capitals, transforming them into
products and services. In integrated reporting, management needs to report on outputs
and outcomes. Which of the following statement is correct?
a. Outputs are quantifiable in currency units, but outcomes are never possible
to measure.
b. The term outputs and the term outcomes have the same meaning, but they differ
in the measurement method.
c. Outputs refer to the dollars generated from sales, whereas outcomes refer to the
quantities produced of an item.
d. Outcomes are achievements that occurred because of the activities or services the
organization provides.

Question 5 MCQ-12347

A restaurant changed its strategy and business model and transformed into the business
of processing healthy food for sale in school cafeterias. To measure whether the business
achieved its goals, the manager will collect information for the annual integrated report
distributed to stakeholders. The manager wants the report to highlight the outcomes of the
new model. In preparation for writing the report, the manager creates a list of questions
that need to be answered. Which one of the following questions could be eliminated
because it does not measure outcomes?
a. Did employees learn new processing methods?
b. How many students were served in the various schools?
c. Have students' health been improved?
d. Are children achieving better in school?

1–62 Module 3 A.1. Financial


© Becker Professional Education Corporation. Statements:
All rights reserved.Part 2
4
MODULE
PART 1 UNIT 1

A.2. Asset Valuation:


Receivables
Part 1
Unit 1

This module covers the following content from the IMA Learning Outcome Statements.

CMA LOS Reference: Part 1—Section A.2. Asset Valuation: Receivables

The candidate should be able to:


a. identify issues related to the valuation of accounts receivable, including timing of
recognition and estimation of the allowance for credit losses
b. distinguish between receivables sold (factoring) on a with-recourse basis and those
sold on a without-recourse basis, and determine the effect on the balance sheet

1 Receivables

Accounts receivable are oral promises to pay debts that represent the right to the receipt of cash
in the future by an organization. Generally, receivables are classified as current assets, but can
be non-current depending upon the terms of agreement.
Receivables are classified either as trade receivables (accounts receivable from purchasers of the
company's goods and services) or nontrade receivables (accounts receivable from persons other
than customers, such as advances to employees, tax refunds, etc.).
The net realizable value of accounts receivable is the balance of the accounts receivable account
adjusted for allowances for receivables that may be uncollectible, sales discounts, and sales
returns and allowances.

1.1 Accounts Receivable and Analysis Format LOS 1A2a

The preparation of an account analysis may increase your ability to "squeeze" or otherwise
derive various answers to CMA Exam questions regarding accounts receivable, allowance for
doubtful accounts, and many other accounts. The T-account format can be a beneficial way to
quickly calculate these answers.

Accounts Receivable

Beg. Balance
Credit sales
Subsequent cash collections
Write-offs
Conversion to note receivable
End. Balance

© Becker Professional Education Corporation. All rights reserved. Module 4 1–63 A.2
4 A.2. Asset Valuation: Receivables PART 1 UNIT 1

Illustration 1 Accounts Receivable

Evelyn Enterprises has an accounts receivable balance of $85,000 as of January 1, Year 1.


At December 31, Year 1, the balance in accounts receivable was $50,000. During the
year, Evelyn collected $805,000 from credit sales during the year. Credit sales during
the year were $795,000. Bad debts written off during the year can be calculated using
this information.

Accounts Receivable
Beg. Balance $ 85,000
Credit sales 795,000
$805,000 Subsequent cash collections
Trial balance $ 75,000
Write-offs = 25,000
End. Balance $ 50,000

If the balance before write-offs is $75,000 based on analysis of the T-account and the
ending balance is $50,000, the write-offs for the year must be $25,000.

1.2 Receivable Recognition and Valuation


Accounts receivable are initially valued at the original transaction amount, adjusted for any sales
discounts or sales returns and allowances.

1.2.1 Sales or Cash Discounts


Sales discounts are generally based on a percentage of the sales price. For example, a discount
of 2/10, n/30 offers the purchaser a discount of 2 percent of the sales price if the payment is
made within 10 days. If the discount is not taken, the entire (gross) amount is due in 30 days.
The calculation of cash discounts typically follows one of two forms: the gross method and the
net method, depending on how frequently customers take the offered discount.

Read as: Two-ten, net thirty

2 / 10 , n / 30
If you
Take a 2% Pay the full Within
pay within Otherwise
discount amount 30 days
10 days

1–64 Module 4 A.2. Asset Valuation:


© Becker Professional Education Corporation. Receivables
All rights reserved.
PART 1 UNIT
4 1 A.2. Asset Valuation: Receivables

Gross Method
The gross method records a sale without adjustment for the available discount. If payment
is received within the discount period, a sales discount (contra-revenue) account is debited
to reflect the sales discount with a corresponding credit to accounts receivable to reduce the
value to the amount collected.
Net Method
The net method records sales and accounts receivable net of the available discount, taking
the discount into consideration at the date of sale. An adjustment is not needed if payment
is received within the discount period. If payment is received after the discount period, a
sales discount forfeited (not taken) account, which serves as a revenue account, must be
credited with an offsetting debit for the additional cash received.

1.2.2 Trade Discounts


Trade discounts (quantity discounts) are offered to customers usually as a percentage reduction
in the list price of the item. These discounts are known and do not represent uncertainty in future
amounts to be collected from customers and, therefore, sales transactions are recorded net of any
trade discounts offered. Helpful hint: Apply trade discounts sequentially when determining the
transaction price.

Example 1 Application of Trade Discounts

Facts: Caitlyn & Brown sells coats with a list price of $1,000. They are sold to stores for list
price minus trade discounts of 40 percent and 10 percent.
Required: Calculate the Caitlyn & Brown accounts receivable balance if 100 coats are sold
on credit.
Solution:

List price $100,000


Less: 40% discount (40,000)
List price after 40% discount 60,000
Less: 10% discount (6,000)
Accounts receivable balance $ 54,000

1.3 Estimating Uncollectible Accounts


Accounts receivable should be presented on the balance sheet at net realizable value. The
amount recorded at the initial transaction should be reduced by an estimate of any uncollectible
receivables. Although both the direct write-off method and the allowance method exist, only the
allowance method is allowed for GAAP financial statements.
The allowance method recognizes not every customer will pay its bill and includes an estimate of
the bad debts associated with accounts receivables held by the company. Companies estimate
the necessary balance in the allowance for uncollectible accounts using the current expected
credit losses (CECL) model. Under the CECL model, the balance in the allowance for uncollectible
accounts should be based on current conditions, past experience, and future expectations. The
allowance should consider the possibility of credit losses over the entire life of each receivable.

© Becker Professional Education Corporation. All rights reserved. Module 4 1–65 A.2
4 A.2. Asset Valuation: Receivables PART 1 UNIT 1

A company can apply any method that reasonably captures its expectation of credit losses,
including estimates based on the following:
Individual accounts
A percentage of the entire account receivable balance
Aging of accounts receivable and estimating bad debts based on aging categories
The allowance for uncollectible accounts is a contra-asset account used to reduce the carrying
value of accounts receivables to reflect the amounts the company expects to collect from
customers. Transactions affecting the allowance for uncollectible accounts can be summarized
in the T-account below.

Allowance for Uncollectible Accounts


Beg. Balance
(+) Provision for bad debts
(−) Write-offs
(+) Reversal of write-offs for
subsequent collection
End. Balance

The normal balance for the allowance account is a credit because it is a contra-asset. Increases
to the allowance account include current bad debts estimates and reversals of previously
written-off receivables where subsequent cash collections have occurred. Decreases to the
account occur when specific accounts deemed uncollectible are identified and written off.

1.3.1 Percentage of Accounts Receivable Method


Uncollectible accounts may be estimated as a percentage of accounts receivable at year-end.
The percentage is based on the company's experience and is used to calculate the ending
balance of the allowance for doubtful accounts on the balance sheet. The difference between
the unadjusted balance and the desired ending balance is debited (or credited) to arrive at the
balance in the allowance for uncollectible accounts. The offset (debit or credit, depending on the
calculation) goes to bad debt expense.

Percentage of Accounts Receivables Method:


Example 2
Normal Balance

Facts: DEF Co. uses a percentage for uncollectible accounts based on the year-end balance
in accounts receivable. DEF Co. estimates that the balance in the allowance account must
be 2 percent of year-end accounts receivable of $80,000. The balance in the allowance
account is a $1,000 credit before adjustment.
Required: Prepare the journal entry to record the adjustment to the allowance account
at year-end.

(continued)

1–66 Module 4 A.2. Asset Valuation:


© Becker Professional Education Corporation. Receivables
All rights reserved.
PART 1 UNIT
4 1 A.2. Asset Valuation: Receivables

(continued)

Solution:
The ending balance in the allowance account should be $1,600 ($80,000 × 2%).

Allowance for Uncollectible Accounts


$1,000 Beg. balance

x = Adjustment needed
$1,600 Desired balance

In order to achieve the desired balance in the allowance account of $1,600 an entry in the
amount of $600 is necessary.
Journal entry to record increase in allowance account:

DR Bad debt expense $600


CR Allowance for uncollectible accounts $600

Example 3 Percentage of Accounts Receivables Method: Debit Balance

Facts: DEF Co. uses a percentage for uncollectible accounts based on the year-end balance
in accounts receivable. DEF Co. estimates that the balance in the allowance account must
be 2 percent of year-end accounts receivable of $80,000. The balance in the allowance
account is a $1,000 debit before adjustment.
Required: Prepare the journal entry to record the adjustment to the allowance account
at year-end.
Solution: The ending balance in the allowance account should be $1,600 ($80,000 × 2%).

Allowance for Uncollectible Accounts


$1,000 Beg. balance
x = Adjustment needed
$1,600 Desired balance

In order to achieve the desired balance in the allowance account of 1,600, an entry in the
amount of 2,600 is necessary.
Journal entry to record increase in allowance account:

DR Bad debt expense $2,600


CR Allowance for uncollectible accounts $2,600

© Becker Professional Education Corporation. All rights reserved. Module 4 1–67 A.2
4 A.2. Asset Valuation: Receivables PART 1 UNIT 1

Pass Key

Under both the percentage of accounts receivable and the aging analysis methods, the
desired balance in the allowance account is calculated first. The entry recorded will be a
"plug" entry, working backwards to achieve the desired balance in the account.

1.3.2 Aging of Accounts Receivable


Another method used to estimate uncollectible accounts is aging of accounts receivable.
A schedule is prepared categorizing accounts by the number of days or months outstanding.
Each category's total dollar amount is then multiplied by the percentage estimated to be
uncollectable based on the entity's experience. The sum from each aging category is added
together to find the ending balance in the allowance account.

Example 4 Aging of Accounts Receivables Method

Facts: DEF Co. uses an aging of accounts receivable to estimate uncollectible accounts. The
balance in the allowance account is a $1,000 credit before adjustment. Below is the aging
schedule prepared by DEF Co. at year-end.
Balance in Each Estimated % Estimated
Aging Category Category Uncollectible Uncollectible
Current $10,000 0.01 $ 100
31–60 days 6,667 0.03 200
61–90 days 5,000 0.10 500
Over 90 days 4,000 0.20 800
$25,667 $1,600

Required: Prepare the journal entry to record the adjustment to the allowance account
at year-end.
Solution:

Allowance for Uncollectible Accounts


$1,000
x = Adjustment needed

$1,600 Desired balance

To achieve the desired balance in the allowance account of $1,600, an entry in the amount
of $600 is necessary.
Journal entry to record increase in allowance account:

DR Bad debt expense $600


CR Allowance for uncollectible accounts $600

1–68 Module 4 A.2. Asset Valuation:


© Becker Professional Education Corporation. Receivables
All rights reserved.
PART 1 UNIT
4 1 A.2. Asset Valuation: Receivables

1.4 Bad Debt Expense


The amount charged to earnings for the bad debt expense of the period includes:
1. the provision made each period throughout the year; and
2. an adjustment made at year-end to increase/decrease the balance in the allowance for
uncollectible accounts, if needed.

1.5 Write-off of a Specific Accounts Receivable


Although the entity records uncollectible amounts at the time the estimate is made, the
company does not know which customers will eventually end up not paying their account.
When a receivable is formally determined to be uncollectible, the following entry is made:

DR Allowance for uncollectible accounts $XXX


CR Accounts receivable $XXX

1.6 Subsequent Collection of Accounts Receivable Written Off


If a collection is made on a receivable that was previously written off, the following entries are made:
Step 1: To restore the account previously written off:

DR Accounts receivable $XXX


CR Allowance for uncollectible accounts $XXX

Step 2: To record the cash collection on the account:

DR Cash $XXX
CR Accounts receivable $XXX

Illustration 2 Year-End Adjusting Entry

Scott Enterprises provides customers with 45 days of credit. The company uses the
allowance method for its uncollectible accounts receivable. At year-end, an aging of
accounts receivable is prepared and the allowance for uncollectible accounts is adjusted
based on the determined desired balance in the allowance account.
At the end of Year 1, accounts receivable was $565,000 and the allowance for uncollectible
accounts had a credit balance of $25,000. Activity during Year 2 related to accounts
receivables is shown below.

Beginning A/R balance $ 565,000


Credit sales 2,580,000
Write-offs (62,000)
Collections (2,750,000)
Ending A/R balance $ 333,000

© Becker Professional Education Corporation. All rights reserved. Module 4 1–69 A.2
4 A.2. Asset Valuation: Receivables PART 1 UNIT 1

LOS 1A2b 2 Factoring of Accounts Receivable

Factoring is a process by which a company can convert its receivables into cash by selling them
to a "factor" either with or without recourse. Under factoring arrangements, the customer may
or may not be notified. Factors include finance companies or banks that purchase receivables
from a company for a fee and collect the remittances from customers.
In these types of transactions, receivables are sold according to the terms of the arrangement as
either with or without recourse. When receivables are sold with recourse, a recourse obligation/
liability must be recorded. Recourse in a factoring arrangement represents the right of the factor
to receive payment from the transferor (seller) even if some of the receivables in the sale prove
to be uncollectible. This right creates a recourse obligation/liability for the transferor (seller).
When this requirement is present, the seller must estimate and record the fair value of its
recourse obligation/liability and record the contingency.

2.1 Without Recourse


If a sale is without recourse, the sale is final and the factor assumes the risk of any losses
on collections. If the factor is unable to collect all the accounts receivable, the factor has no
recourse against the seller (i.e., the factor cannot ask for more money from the seller).
Journal entry to factor accounts receivable without recourse:

DR Cash $XXX
DR Due from factor (factor's margin) XXX
DR Loss on sale of receivable XXX
CR Accounts receivable $XXX

The entry to the asset account "Due from factor" reflects the proceeds retained by the factor.
This amount protects the factor against sales returns, sales discounts, allowances, and customer
disputes. If the returns, discounts, and allowances are less than the retained amount, the
balance will be returned to the seller.

Example 5 Factoring Receivables Without Recourse (True Sale)

Facts: Emmons Corp. needs some money for operations. In September Year 1, the company
elected to sell $5,500,000 to a factor without recourse. The factor charges a 5 percent fee and
retains another 10 percent as security to cover returns, allowances, and discounts taken by
the customer. The factor will collect the accounts receivable.
Required: Record the sale of the accounts receivable.
Solution:
DR Cash $4,675,000
DR Due from factor (factor's margin)* 550,000
DR Loss on sale of receivable** 275,000
CR Accounts receivable $5,500,000

*$5,500,000 × 10% = $550,000 (protection of factor against returns, allowances, and


discounts taken by the customer)
**$5,500,000 × 5% = $275,000 (fee charge by factor for purchase of receivables)

1–70 Module 4 A.2. Asset Valuation:


© Becker Professional Education Corporation. Receivables
All rights reserved.
PART 1 UNIT
4 1 A.2. Asset Valuation: Receivables

2.2 With Recourse


If a sale is on a recourse basis, the factor has an option to sell any uncollectible receivables back
to the seller to limit collection losses.
If accounts receivable is transferred to a factor with recourse, two treatments are possible.
The transfer may be considered either a sale or a loan with the receivables serving as collateral.
To be considered a sale, the transfer must meet the following conditions:
yy Transferred assets are beyond the reach of the transferor and its creditors.
yy The transferor surrenders control of the future economic benefits of the receivables to
the factor (the buyer).
yy The transferor cannot be required to repurchase the receivables but may be required to
replace the receivables with other similar receivables.
If any of the above conditions are not met, the transfer is treated as a loan.

Example 6 Factoring Receivables With Recourse

Facts: Emmons Corp. needs some money for operations. In September Year 1, the company
elected to sell $5,500,000 to a factor with recourse. The factor charges a 5 percent fee and
retains another 10 percent as security to cover returns, allowances, and discounts taken by
the customer. The fair value of the estimated recourse liability is $100,000.
Required: Record the sale of the accounts receivable.
Solution:

DR Cash $4,675,000
DR Due from factor (factor's margin)* 550,000
DR Loss on sale of receivable** 375,000
CR Accounts receivable $5,500,000
CR Recourse liability 100,000

*$5,500,000 × 10% = $550,000 (protection of factor against returns, allowances, and


discounts taken by the customer)
**$5,500,000 × 5% = $275,000 (fee charge by factor for purchase of receivables) + $100,000
estimated recourse liability

© Becker Professional Education Corporation. All rights reserved. Module 4 1–71 A.2
4 A.2. Asset Valuation: Receivables PART 1 UNIT 1

Question 1 MCQ-12358

Strawser Financial Services utilizes the allowance method to account for bad debts.
During the current year, Strawser received payment from a customer whose account had
been previously written off. What is the effect on each account indicated based on the
subsequent cash collection of account previously written off?

Allowance for
Accounts Uncollectible
Receivable Accounts
a. Increase Increase
b. Decrease Decrease
c. No effect No effect
d. No effect Increase

Question 2 MCQ-12361

Harris Hat's Enterprises had credit sales of $1,200,000 during the current year. During the
year, the company identified $25,000 in accounts deemed uncollectible. Cash collections
during the year were $1,150,000 and the accounts receivable balance decreased to $35,000
at year-end. What is the accounts receivable balance at the beginning of the year?
a. $10,000 debit
b. $25,000 debit
c. $15,000 debit
d. $35,000 debit

Question 3 MCQ-12364

On June 5 of the current year, Sew-Well Enterprises sold goods to a customer for $9,500
with credit terms 2/10, net 30. Sowell uses the gross method of accounting for sales
discounts. The customer returned $2,000 of goods on June 6. Sowell receives payment
from the customer on June 8. What amount of cash was received by Sowell from
the customer?
a. $9,500
b. $9,310
c. $7,500
d. $7,350

1–72 Module 4 A.2. Asset Valuation:


© Becker Professional Education Corporation. Receivables
All rights reserved.
PART 1 UNIT
4 1 A.2. Asset Valuation: Receivables

Question 4 MCQ-12366

Karrington Co. is trying to determine how to account for the financing of receivables with
recourse. Karrington knows the transaction must be recorded as either a sale or a secured
borrowing. Under U.S. GAAP, which element is required to account for the transaction as
a sale?
a. The transferor has right to pledge or exchange the receivables received.
b. The transferor maintains control over the transferred assets.
c. The transferor and its creditors have access to transferred assets.
d. The transferor of the receivables surrenders control of assets.

© Becker Professional Education Corporation. All rights reserved. Module 4 1–73 A.2
4 A.2. Asset Valuation: Receivables PART 1 UNIT 1

NOTES

1–74 Module 4 A.2. Asset Valuation:


© Becker Professional Education Corporation. Receivables
All rights reserved.
5
MODULE
PART 1 UNIT 1

A.2. Asset Valuation:


Inventory
Part 1
Unit 1

This module covers the following content from the IMA Learning Outcome Statements.

CMA LOS Reference: Part 1—Section A.2. Asset Valuation: Inventory

The candidate should be able to:


c. identify issues in inventory valuation, including which goods to include, what costs to
include, and which cost assumption to use
d. identify and compare cost flow assumptions used in accounting for inventories
e. demonstrate an understanding of the lower of cost or market rule for LIFO and the
retail inventory method and the lower of cost and net realizable value rule for all other
inventory methods
f. calculate the effect on income and on assets of using different inventory methods
g. analyze the effects of inventory errors
h. identify advantages and disadvantages of the different inventory methods
i. recommend the inventory method and cost flow assumption that should be used for a
firm given a set of facts
ff. identify and describe the following differences between U.S. GAAP and IFRS: (iii)
inventories, with respect to costing methods, valuation, and write-downs (e.g., LIFO)

1 Overview of Inventory

Inventory represents assets held for resale or for use by an entity to manufacture finished goods
held for resale. Inventory must be periodically counted, valued, and adjusted through journal
entries for proper financial statement presentation. In general, there are four types of inventory.
Wholesale Inventory or Retail Inventory: This inventory is resold by merchandising
companies in substantially the same form as it was purchased.
Raw Materials Inventory: Raw materials inventory is inventory held for use by
manufacturing companies in the production process. Raw materials inventory includes
both direct and indirect materials that have not yet entered the production process but are
currently available to use in production.
Work-in-Process Inventory (WIP): WIP is inventory that has been placed into production,
but the production process is not yet complete. Work-in-process inventory consists of the
raw materials issued into production (direct materials), direct labor, and manufacturing
overhead applied during the period.
Finished Goods Inventory: Finished goods inventory is production inventory that is complete
and ready for sale. Until finished goods are sold, they remain on the balance sheet as inventory.
Once they are sold, the cost of the goods moves from the balance sheet to the income statement.

© Becker Professional Education Corporation. All rights reserved. Module 5 1–75 A


5 A.2. Asset Valuation: Inventory PART 1 UNIT 1

LOS 1A2c 2 Goods and Materials to Be Included in Inventory

At the end of each accounting period, inventory should include all goods and materials in which
the company has legal title, even if the company does not have physical possession of the goods.

2.1 Goods in Transit


Title passes from the seller to the buyer in the manner and under the conditions explicitly
agreed on by the parties. If no conditions are explicitly agreed on ahead of time, title passes
from the seller to the buyer at the time and place where the seller’s performance regarding
delivery of goods is complete.
FOB means “free on board” and requires the seller to deliver the goods to the location indicated
as FOB at the seller’s expense. The following terminology is most commonly used in passing title
from the seller to the buyer:
FOB Shipping Point: With FOB shipping point, title passes to the buyer when the seller
delivers the goods to a common carrier. Goods shipped in this manner should be included
in the buyer’s inventory upon shipment.
FOB Destination: With FOB destination, title passes to the buyer when the buyer receives
the goods from the common carrier.

2.2 Shipment of Nonconforming Goods


If the seller ships the wrong goods, the title reverts to the seller upon rejection by the buyer.
Thus, the goods should not be included in the buyer’s inventory, even if the buyer possesses the
goods prior to their return to the seller.

Example 1 Inventory in Transit

Facts: Harris Co. has several inventory shipments in process at year-end.


1. $55,000 of goods in transit from Emmons Corp., a supplier. The goods are shipped FOB
destination.
2. $38,000 of goods in transit from Jones Co., a supplier. The goods are shipped FOB
shipping point.
3. $15,800 of goods in transit to Dawson Inc., a customer. The goods are shipped FOB
shipping point.
4. $25,680 of goods in transit to Mongeau Co., a customer. The goods are shipped FOB
destination.
Required: Determine which inventory purchases should be included by Harris Co. in the
determination of ending inventory.

(continued)

1–76 Module 5 A.2. Asset


© Becker Professional Education Corporation. AllValuation: Inventory
rights reserved.
PART 1 UNIT
5 1 A.2. Asset Valuation: Inventory

(continued)

Solution:

Include in Exclude From


Inventory Inventory
1. $55,000 
2. $38,000 
3. $15,800 
4. $25,680 
1. Harris does not include the inventory purchased FOB destination until Harris receives
the goods and does not reject them as being the wrong goods.
2. The goods purchased from Jones should be included in inventory because title passed
at when the goods were picked up by the shipper.
3. Harris does not include the goods sold to Dawson in inventory because title
transferred to Dawson when the carrier received the goods.
4. Harris should include the goods sold to Mongeau in inventory while the goods are in
transit because title does not pass until the goods are delivered to the customer.

2.3 Goods Held on Consignment


Companies may arrange to physically transfer inventory items to another company in order for
that other company to sell the inventory on behalf of the consignor (owner). The consignor of
the goods possesses legal title to the goods until sold by the consignee (holder of the goods).
If sold by the consignee, the selling price less any consignee commission is remitted to the
consignor. Because the inventory is merely being held—but not owned—by the consignee, legal
title of the inventory remains with the consignor, and the inventory should be maintained on the
consignor company's accounting records, not on the consignee's accounting records.

Pass Key

Consignor (owner): Retains title of inventory despite the lack of physical possession.
Consignee (agent/holder): Has physical possession but lacks ownership or title of the
inventory and acts as an intermediary in the selling process.

© Becker Professional Education Corporation. All rights reserved. Module 5 1–77 A


5 A.2. Asset Valuation: Inventory PART 1 UNIT 1

Example 2 Inclusion in Inventory

Facts: Harris Corp. has an ending inventory balance of $500,000 and had the following
consignment arrangements at year-end:
1. Goods with a cost of $78,500 out on consignment to Dawson Corp. The inventory is
currently excluded from Harris' year-end inventory balance.
2. Goods with a cost of $123,000 held on consignment for Emmons Corp. This amount is
currently included in Harris' year-end inventory balance.
Required: Determine the correct year-end inventory balance.
Solution:

Inventory balance before adjustment $500,000


Inventory out on consignment with Dawson 78,500
Inventory held on consignment for Emmons (123,000)
Adjusted inventory balance $455,500

The goods held on consignment by Harris and owned by Emmons Corp. should be
excluded from Harris' inventory records. Although Harris has physical possession of the
goods, the legal title to the goods remains with Emmons Corp. until the goods are sold to a
third-party buyer (and then title transfers to that third-party buyer).
The goods out on consignment from Harris to Dawson Corp. should be included in Harris'
year-end inventory balance. Legal title to the goods remains with Harris even though the
goods are out on consignment to Dawson. No transfer of title will take place until the goods
are sold to a third-party buyer (and then title transfers to that third-party buyer).

3 Inventory Systems

There are two types of inventory systems used to track and count inventory: the periodic
inventory system and the perpetual inventory system. Both systems are allowed in financial
reporting; however, because of technological advances, perpetual inventory is now the most
common system for many companies.

3.1 Periodic Inventory System


With a periodic inventory system, the quantity of inventory is determined only by physical count,
usually at least annually. Therefore, units of inventory and the associated costs are counted
and valued at the end of the accounting period. The actual cost of goods sold for the period
is determined after each physical inventory by "squeezing" the difference between beginning
inventory plus purchases less ending inventory, based on the physical count.

1–78 Module 5 A.2. Asset


© Becker Professional Education Corporation. AllValuation: Inventory
rights reserved.
PART 1 UNIT
5 1 A.2. Asset Valuation: Inventory

The periodic method does not keep a running total of the inventory balances. Ending inventory
is physically counted and priced. Cost of goods sold is calculated as shown below:

Beginning inventory $ 70,000

+ Purchases 300,000

Cost of goods available for sale 370,000

− Ending inventory (physical count) (270,000)


Cost of goods sold $ 100,000

3.2 Perpetual Inventory System


With a perpetual inventory system, the inventory record for each item of inventory is updated
for each purchase and each sale as they occur. The actual cost of goods sold is determined and
recorded with each sale. Therefore, the perpetual inventory system keeps a running total of
inventory balances.
Ending inventory under the perpetual inventory system is still physically counted and costed
and then compared to the perpetual inventory balance. If the amount of ending inventory
determined from the physical count is less than the ending inventory amount shown in the
ledger, the difference is adjusted to an inventory shrinkage/spoilage account.

Comparison of Periodic and Perpetual Inventory


Example 3
Methods (Sales)

Facts: ABC Company sold 20,000 units of inventory for $7 per unit. The inventory had
originally cost $5 per unit.
Required: Prepare the journal entries to record the sale using the periodic and perpetual
methods.
Solution:
Journal entry to record sale under periodic method (cost of goods sold will be recorded after the
periodic inventory count):

DR Cash $140,000
CR Sales $140,000

Journal entry to record sale under perpetual method:

DR Cash $140,000
CR Sales $140,000
DR Cost of goods sold 100,000
CR Inventory 100,000

© Becker Professional Education Corporation. All rights reserved. Module 5 1–79 A


5 A.2. Asset Valuation: Inventory PART 1 UNIT 1

Comparison of Periodic and Perpetual Inventory


Example 4
Methods (Purchases)

Facts: ABC Company purchased 50,000 units of merchandise for $6 a unit to be held
as inventory.
Required: Prepare the journal entries to record the purchase of inventory under the
periodic and the perpetual methods.
Solution: The periodic method debits purchases; the perpetual method debits inventory.
Journal entry to record purchase under periodic method:

DR Purchases $300,000
CR Cash $300,000

Journal entry to record purchase under perpetual method:

DR Inventory $300,000
CR Cash $300,000

LOS 1A2d 4 Primary Inventory Cost Flow Assumptions


LOS 1A2f
Inventory valuation is dependent on the cost flow assumption underlying the computation.
Under U.S. GAAP, the cost flow assumption used by a company is not required to have a rational
relationship with the physical inventory flows; however, the primary objective is the selection of
the method that will most clearly reflect periodic income.
When similar goods are purchased at different times, it may not be possible to identify and
match the specific costs of the item sold. Frequently, the identity of goods and their specific
related costs are lost between the time of acquisition and the time of sale. This has resulted in
the development and general acceptance of several assumptions with respect to the flow of
cost factors (FIFO, LIFO, and average cost) to provide practical bases for the measurement of
periodic income.

4.1 Specific Identification Method


Under the specific identification method, the cost of each item in inventory is uniquely identified
to that item. The cost follows the physical flow of the item in and out of inventory to cost of
goods sold. Specific identification is usually used for physically large or high-value items and
allows for greater opportunity for manipulation of income. Specific identification allows for
greater opportunity for manipulation of income because the company often can use lower cost
inventory items that are identical to higher-priced items in order to match against current period
sales, thereby increasing profits.

4.2 First-In, First Out (FIFO)


Under FIFO, the first costs inventoried are the first costs transferred to cost of goods sold.
Ending inventory includes the most recently incurred costs; thus, the ending balance
approximates replacement cost. Ending inventory and cost of goods sold are the same whether
a periodic or perpetual inventory system is used.

1–80 Module 5 A.2. Asset


© Becker Professional Education Corporation. AllValuation: Inventory
rights reserved.
PART 1 UNIT
5 1 A.2. Asset Valuation: Inventory

Pass Key

In periods of rising prices, the FIFO method results in the highest ending inventory, the
lowest costs of goods sold, and the highest net income (i.e., current costs are not matched
with current revenues).

Example 5 FIFO Method

Facts: During its first year of operations, Helix Corporation has purchased all of its
inventory in three batches. Batch 1 was for 4,000 units at $4.25 per unit. Batch 2 was
for 2,000 units at $4.50 per unit. Batch 3 was for 3,000 units at $4.75 per unit. In total,
4,000 units were sold, 3,000 units after the first purchase and 1,000 units after the
second purchase.
Required: Determine the amounts of ending inventory and cost of goods sold using the
FIFO method and the periodic and perpetual systems.
Solution:
FIFO: Periodic Inventory System
Units Ending Goods Available
Bought Cost/Unit Inventory for Sale
4,000 $4.25 $ 17,000
2,000 4.50 $ 9,000 9,000
3,000 4.75 14,250 14,250
$23,250 40,250
Ending inventory 23,250
Cost of goods sold $17,000

FIFO: Perpetual Inventory System

Units Units Change in Inventory


Bought Sold Cost/Unit Inventory Balance COGS
4,000 $4.25 $17,000
3,000 4.25 (12,750) 4,250 $12,750
2,000 4.50 9,000 13,250
1,000 4.25 (4,250) 9,000 4,250
3,000 4.75 14,250 23,250
Ending inventory $23,250
Cost of goods sold $17,000

Note that the ending inventory under both methods is $23,250 and the amount of cost of
goods sold under both methods is $17,000.

© Becker Professional Education Corporation. All rights reserved. Module 5 1–81 A


5 A.2. Asset Valuation: Inventory PART 1 UNIT 1

4.3 Weighted Average Method


Under the weighted average method, at the end of the period, the average cost of each item
in inventory would be the weighted average of the costs of all items in inventory. The weighted
average is determined by dividing the total costs of inventory available by the total number of
units of inventory available, remembering that beginning inventory is included in both totals.
This method is particularly suitable for homogeneous products and a periodic inventory system.

Cost of goods available for sale


during the period
= Weighted average cost per unit
Number of units available during
the period

Example 6 Weighted Average Method

Facts: Assume the same information for Helix Corporation as in the previous example for FIFO.
Requirement: Determine the amounts of ending inventory and cost of goods sold under
the weighted average method.
Solution:

Unit Cost Units Purchased Total


$4.25 4,000 $ 17,000
4.50 2,000 9,000
4.75 3,000 14,250
Total 9,000 $40,250

Weighted average cost per unit = $4.4722 ($40,250/9,000)


Cost of goods sold = $17,889 (4,000 units × $4.4722)
Ending inventory = $22,361 (5,000 units × $4.4722)

4.4 Moving Average Method


The moving average method computes the weighted average cost after each purchase by dividing
the total cost of inventory available after each purchase (inventory plus current purchase) by the total
units available after each purchase. The moving average is more current than the weighted average.
A perpetual inventory system is necessary to use the moving average method.

Example 7 Moving Average Method

Facts: Assume the same information for Helix Corporation as in the previous example for FIFO.
Required: Determine the amounts of ending inventory and cost of goods sold under the
moving average method.

(continued)

1–82 Module 5 A.2. Asset


© Becker Professional Education Corporation. AllValuation: Inventory
rights reserved.
PART 1 UNIT
5 1 A.2. Asset Valuation: Inventory

(continued)

Solution:
Purchases/(Sales) Inventory Balances (Rounded)
Quantity Cost Total Quantity Average Cost Total
4,000 $4.25 $17,000 4,000 $4.25 $17,000
(3,000) 4.25 (12,750) 1,000 4.25 4,250
2,000 4.50 9,000 3,000 4.4167* 13,250
(1,000) 4.4167 (4,417) 2,000 4.4167 8,833
3,000 4.75 14,250 5,000 4.6166** 23,083

*Weighted average cost per unit = ($4,250 + $9,000) / 3,000 = $4.4167


**Weighted average cost per unit = ($8,833 + $14,250) / 5,000 = $4.6166
Cost of goods sold is $17,167 ($12,750 + $4,417)
Ending inventory is $23,083

4.5 Last In, First Out (LIFO) Method (Not Permitted Under IFRS)
Under LIFO, the last costs inventoried are the first costs transferred to cost of goods sold.
Ending inventory, therefore, includes the oldest costs. The ending balance of inventory will
typically not approximate replacement cost.
LIFO does not generally relate to actual flow of goods in a company because most
companies sell or use their oldest goods first to prevent holding old or obsolete items.
If LIFO is used for tax purposes, it must also be used in the GAAP financial statements.

4.5.1 LIFO Financial Statement Effects


The use of the LIFO method generally better matches expense against revenues because it
matches current costs with current revenues; thus, LIFO eliminates holding gains and reduces
net income during times of inflation.
If sales exceed production (or purchases) for a given period, LIFO will result in a distortion of net
income because old inventory costs (called "LIFO layers") will be matched with current revenue.
LIFO is susceptible to income manipulation by intentionally reducing purchases in order to
use old layers at lower costs.

Pass Key

In periods of rising prices, the LIFO method generally results in the lowest ending
inventory, the highest costs of goods sold, and the lowest net income.
Remember: LIFO = Lowest

© Becker Professional Education Corporation. All rights reserved. Module 5 1–83 A


5 A.2. Asset Valuation: Inventory PART 1 UNIT 1

4.5.2 LIFO Layers


The last in, first out method of determining inventory requires that records be maintained as to
the base year inventory amount and additional layers that may be added yearly.
After an original LIFO amount is created (base year), it may decrease, or additional layers may
be created in each year according to the amount of ending inventory. An additional LIFO layer
is created in any year in which the ending inventory is greater than the beginning inventory.
An additional LIFO layer is priced at the earliest costs of the year in which it was created,
because the LIFO method matches the most current costs incurred with current revenues,
leaving the first cost incurred to be included in any inventory increase.

Illustration 1 LIFO Layer Container

Purchases Cost of goods sold


at varying costs Last in, first out LIFO

Layer 3 at $1.30

Layer 2 at $1.20
Layer 1 at $1.00
Ending inventory

4.5.3 LIFO Liquidation


A LIFO liquidation occurs in any period in which sales are greater than the number of units
produced or purchased for sale during the period. When inventory quantities decline during
the period under LIFO, older, less-costly inventory items are included in the calculation of cost
of goods sold and are offset against current period selling prices. This results in higher profit
margins and can be used by companies as a form of income manipulation.

Example 8 LIFO Liquidation

Facts: Dawson Enterprises uses the LIFO inventory method. The company began the year
with 15,000 units of inventory at a cost of $10 each. During the year an additional 25,000
units were purchased for $18 each. The company sold 32,000 units during the year.
Required: Calculate the cost of goods sold for the current year and the effect of the use of
LIFO on income during the period.
Solution:
Cost of Goods Sold:

25,000 units × $18 cost per unit = $450,000


7,000 units × $10 cost per unit = 70,000
32,000 units COGS $520,000

(continued)

1–84 Module 5 A.2. Asset


© Becker Professional Education Corporation. AllValuation: Inventory
rights reserved.
PART 1 UNIT
5 1 A.2. Asset Valuation: Inventory

(continued)

Impact on Net Income:


When inventory quantities decline during the year, liquidation of older layers of inventory
from past purchases will produce increased income because current selling prices are
matched against older, lower costs.
In this example, the effect of the liquidation of 7,000 units from beginning inventory
increased income (ignoring any tax effects) by $56,000:

7,000 units × ($18 current inventory cost − $10 prior year inventory cost) = $56,000

Pass Key

LIFO layers are added in periods where Production/purchases > Sales


LIFO layers are depleted ("LIFO liquidation") in periods where Sales > Production/purchases

Example 9 LIFO Method

Facts: Assume the same facts for Helix Corporation as in previous examples.
Required: Determine the amounts of ending inventory and cost of goods sold using the LIFO method
and periodic and the perpetual systems.
Solution:
LIFO: Periodic Inventory System
Units Bought Cost/Unit Ending Inventory Goods Available for Sale
4,000 $4.25 $17,000 $ 17,000
2,000 4.50 4,500 9,000
3,000 4.75 14,250
40,250
Ending inventory $21,500 (21,500)
Cost of goods sold $18,750

LIFO: Perpetual Inventory System


Units Bought Units Sold Cost/Unit Inventory Balance COGS
4,000 $4.25 $17,000
3,000 4.25 (12,750) $12,750
2,000 4.50 9,000
1,000 4.50 (4,500) 4,500
3,000 4.75 14,250
$23,000 $17,250
Under the periodic inventory system, ending inventory is $21,500 and cost of goods sold is $18,750.
Under the perpetual inventory system, ending inventory is $23,000 and cost of goods sold is $17,250.

© Becker Professional Education Corporation. All rights reserved. Module 5 1–85 A


5 A.2. Asset Valuation: Inventory PART 1 UNIT 1

LOS 1A2h 5 Comparison of Inventory Cost Flow Assumptions


LOS 1A2i
In a period of rising prices, FIFO results in the highest ending inventory and the lowest cost of
goods sold, LIFO results in the lowest ending inventory and the highest cost of goods sold, and
the average method balances fall between the LIFO and FIFO balances.

Illustration 2 Comparison of Inventory Methods

The following is a comparison LIFO, FIFO, and the average methods for Helix Corp.:
Periodic Inventory System Ending Inventory Cost of Goods Sold
FIFO $23,250 $17,000
Weighted Average 22,361 17,889
LIFO 21,500 18,750
Perpetual Inventory System Ending Inventory Cost of Goods Sold
FIFO $23,250 $17,000
Moving Average 23,083 17,167
LIFO 23,000 17,250

The relative advantages and disadvantages of the different inventory cost flow assumptions
should be considered when determining which inventory cost flow assumption to use.

Methods Advantages Disadvantages


Specific identification — Results in a true matching of — Company can manipulate net
actual cost and revenue income by selection of item sold
Average cost — Simple to apply, objective
— Difficult to manipulate net income
First-in, first-out — This method approximates the — Current costs not matched to
physical flow of goods current revenue
— Cannot manipulate income
— Inventory value approximates
current cost
Last-in, first-out — Matches current costs to — Financial net income is lower
current revenue
— LIFO liquidations can distort
— Results in lower taxes with net income
rising prices (lower taxes mean
— Inventory is understated
better cash flow)
because old costs remain in
— Low inventory value means ending inventory
a write-down is less likely to
be needed

1–86 Module 5 A.2. Asset


© Becker Professional Education Corporation. AllValuation: Inventory
rights reserved.
PART 1 UNIT
5 1 A.2. Asset Valuation: Inventory

6 Valuation of Inventory LOS 1A2e

U.S. GAAP requires that inventory be stated at its cost. Where evidence indicates that cost will be
recovered with an approximately normal profit on a sale in the ordinary course of business, no
loss should be recognized even though replacement or reproduction costs are lower.

6.1 Cost
Inventories are generally accounted for at cost, which is defined as the price paid or
consideration given to acquire an asset. Methods used to determine the cost of inventory
include first-in, first-out (FIFO); last-in, first-out (LIFO); average cost; and the retail inventory
method. IFRS does not permit the use of LIFO.

6.2 Departures From the Cost Basis


6.2.1 Lower of Cost or Market, and Lower of Cost and Net Realizable Value
In the ordinary course of business, when the utility of goods is no longer as great as their cost,
a departure from the cost basis principle of measuring inventory is required. This is usually
accomplished by stating such goods at market value or net realizable value, as appropriate. The
purpose of reducing inventory to an amount below cost is to show the probable loss sustained
(conservatism) in the period in which the loss occurred (matching principle).

6.2.2 Recognize Loss in Current Period


Under U.S. GAAP, the write-down of inventory is usually reflected in cost of goods sold, unless the
amount is material, in which case the loss should be identified separately in the income statement.
IFRSs do not specify where an inventory write-down should be reported on the income statement.

6.2.3 Reversal of Inventory Write-downs


Under U.S. GAAP, reversals of inventory write-downs are prohibited. IFRSs allow the reversal of
inventory write-downs for subsequent recoveries of inventory value. The reversal is limited to
the amount of the original write-down and is recorded as a reduction of total inventory costs on
the income statement (COGS) in the period of reversal.

6.2.4 Exceptions
The lower of cost or market and lower of cost and net realizable value rules will not apply when:
the subsequent sales price of an end product is not affected by its market value; or
the company has a firm sales price contract.

© Becker Professional Education Corporation. All rights reserved. Module 5 1–87 A


5 A.2. Asset Valuation: Inventory PART 1 UNIT 1

6.3 Lower of Cost and Net Realizable Value (IFRS and U.S. GAAP)
Under U.S. GAAP, the lower of cost and net realizable value method is used for all inventory
that is not costed using LIFO or the retail inventory method. This method is required to value
all inventory under IFRS. The lower of cost and net realizable value principle may be applied
to a single item, a category, or total inventory, provided that the method most clearly reflects
periodic income.

6.3.1 Net Realizable Value


Net realizable value is an item’s net selling price less the costs to complete and dispose of
the inventory. Net realizable value is the same as the “market ceiling” in the lower of cost or
market method.

6.4 Lower of Cost or Market (U.S. GAAP Only)


Under U.S. GAAP, the lower of cost or market method is used when inventory is costed using LIFO
(not allowed under IFRS) or the retail inventory method. The lower of cost or market principle may
be applied to a single item, a category, or total inventory, provided that the method most clearly
reflects periodic income.

6.4.1 Market Value


Under U.S. GAAP, the term market in the phrase lower of cost or market generally means
current replacement cost (whether by purchase or reproduction), provided the current
replacement cost does not exceed net realizable value (the market ceiling) or fall below net
realizable value reduced by normal profit margin (the market floor).

6.4.2 Definitions
Market Value
Under U.S. GAAP, market value is the median (middle value) of an inventory item’s
replacement cost, its market ceiling, and its market floor.
Replacement Cost
Replacement cost is the cost to purchase the item of inventory as of the valuation date.
Market Ceiling
Market ceiling is an item’s net selling price less the costs to complete and dispose (called the
net realizable value).
Market Floor
Market floor is the market ceiling less a normal profit margin.

1–88 Module 5 A.2. Asset


© Becker Professional Education Corporation. AllValuation: Inventory
rights reserved.
PART 1 UNIT
5 1 A.2. Asset Valuation: Inventory

Example 10 Lower of Cost or Market (U.S. GAAP Only)

Facts: The following information pertains to a company's inventory at the end of the
current year. The company uses LIFO and values its ending inventory using the lower of
cost or market method.
Replacement Selling Costs of Normal
Item Cost Cost Price Completion Profit
1 $20.50 $19.00 $25.00 $1.00 $6.00
2 26.00 20.00 30.00 2.00 7.00
3 10.00 12.00 15.00 1.00 3.00
4 40.00 55.00 60.00 6.00 4.00
Required: Calculate the lower of cost or market for the above four items.
Solution:
Item 1: Determine the maximum ("ceiling") and minimum ("floor") limits for the
replacement cost.
Ceiling = $24.00 ($25 − $1)
Floor = $18.00 ($25 − $1) − $6
Since replacement cost ($19) falls between the maximum and minimum, market price is
$19.00. Market ($19.00) is lower than cost ($20.50); therefore inventory would be valued at
market ($19.00).
Item 2: Determine the maximum and minimum limits for the replacement cost.
Ceiling = $28.00 ($30 − $2)
Floor = $21.00 ($30 − $2) − $7
Since replacement cost ($20) is less than the minimum, market value is the minimum, or
$21.00. Market ($21.00) is lower than cost ($26.00), therefore inventory would be valued at
market ($21.00).
Item 3: Determine the maximum and minimum limits for the replacement cost.
Ceiling = $14.00 ($15 − $1)
Floor = $11.00 ($15 − $1) − $3
Replacement cost ($12) falls within these limits. Since cost ($10.00) is less than replacement
cost ($12.00), the cost of $10.00 is used.
Item 4: Determine the maximum and minimum limits for the replacement cost.
Ceiling = $54.00 ($60 − $6)
Floor = $50.00 ($60 − $6) − $4
Since the replacement cost ($55) exceeds the maximum limit, the maximum ($54.00) is
compared with cost ($40.00). Inventory is valued at cost ($40.00).
When market is lower than cost, the maximum prevents a loss in future periods by valuing
the inventory at its estimated selling price less costs of completion and disposal. The
minimum prevents any future periods from realizing any more than a normal profit.
Journal entry to record the write-down to a separate account:

DR Inventory loss due to decline in market value $XXX


CR Inventory $XXX

© Becker Professional Education Corporation. All rights reserved. Module 5 1–89 A


5 A.2. Asset Valuation: Inventory PART 1 UNIT 1

Example 11 Lower of Cost and Net Realizable Value

Facts: The following information pertains to a company's year-end inventory. The company
uses FIFO and values its ending inventory using the lower of cost and net realizable
value method.
Selling Costs of
Item Cost Price Completion
1 $28.50 $30.00 $3.00
2 21.00 26.00 4.00

Required: Calculate the lower of cost and market for the above two items.
Solution: Determine the lower of cost or net realizable value for the above two items.
Item 1: Determine the net realizable value (NRV):
NRV = $27.00 ($30 − $3)
Net realizable value ($27.00) is lower than cost ($28.50); therefore, inventory would be
valued at net realizable value ($27.00).
Item 2: Determine the net realizable value:
NRV = $22.00 ($26 − $4)
Net realizable value ($22.00) is greater than cost ($21.00); therefore, inventory would be
valued at cost ($21.00).

LOS 1A2g 7 Inventory Errors

Errors in inventory result in the overstatement or understatement of ending inventory balances.


These errors result from mistakes in the physical count of inventory items, inclusion or exclusion
of costs as part of inventory, or due to mistakes in inventory pricing. Both costs of goods sold
reported on the income statement and ending inventory balances reported on the balance sheet
will reflect these over/understatements.
The effect of errors on the financial statements can be determined by considering the
inventory formula:

Beginning inventory
+ Purchases
= Cost of goods available for sale
– Ending inventory
= Cost of goods sold

1–90 Module 5 A.2. Asset


© Becker Professional Education Corporation. AllValuation: Inventory
rights reserved.
PART 1 UNIT
5 1 A.2. Asset Valuation: Inventory

If an error exists in beginning inventory, then cost of goods available for sale and cost of goods
sold will all have the same error. Errors in ending inventory balances will result in the opposite
effect on cost of goods sold.
Cost of goods sold errors will affect the calculation of net income and flow through to retained
earnings. Errors in beginning and ending inventory distort the inventory balance reported on the
balance sheet as a current asset.

Illustration 3 Effects of Inventory Errors

The following table shows the effect on cost of goods sold, net income, and retained
earnings for four independent inventory errors. Note that the effects of taxes on net
income and retained earnings have been ignored.
——Scenario A: Beginning inventory is understated by $20,000
——Scenario B: Ending inventory is understated by $15,000
——Scenario C: Beginning inventory is overstated by $25,000
——Scenario D: Ending inventory is overstated by $30,000

Assume Each Scenario Is Independent


Scenario A Scenario B Scenario C Scenario D
Beginning inventory U – 20,000 O – 25,000
+ Purchases
= Cost of goods available for sale U – 20,000 O – 25,000
– Ending inventory U – 15,000 O – 30,000
= Cost of goods sold U – 20,000 O – 15,000 O – 25,000 U – 30,000

Revenues
– Cost of goods sold U – 20,000 O – 15,000 O – 25,000 U – 30,000
= Gross profit O – 20,000 U – 15,000 U – 25,000 O – 30,000
– Other expense
= Net income (ignoring income taxes) O – 20,000 U – 15,000 U – 25,000 O – 30,000

Beginning retained earnings


+ Net income O – 20,000 U – 15,000 U – 25,000 O – 30,000
– Dividends
Ending retained earnings O – 20,000 U – 15,000 U – 25,000 O – 30,000

Balance sheet—current assets


Inventory U – 20,000 U – 15,000 O – 25,000 O – 30,000

© Becker Professional Education Corporation. All rights reserved. Module 5 1–91 A


5 A.2. Asset Valuation: Inventory PART 1 UNIT 1

Example 12 Effects of Inventory Errors Over a Two-Year Period

Facts: During Year 3, BGSE Inc. discovered that its ending inventories reported in its
financial statements were misstated by the following material amounts:
Year 1 Understated $120,000
Year 2 Overstated $150,000

BGSE Inc. uses a periodic inventory system and the FIFO cost method.
Required: Determine the effect of these errors on retained earnings at January 1, Year 3
before any adjustments and without considering the effect of income taxes.
Solution:
Effect of Year Effect of Year
Effect of Year 1 Error 1 Error on 2 Error on Net Effect at
on Year 1 Year 2 Year 2 End of Year 2
Beg. inventory OK Beg. inventory U – $120,000
Ending inventory U – $120,000 Ending inventory N/A O - $150,000 O – $150,000
Cost of goods sold O – $120,000 Cost of goods sold U – $120,000 U - $150,000 U – $270,000
Net income U – $120,000 Net income O – $120,000 O - $150,000 O – $270,000
Retained earnings U – $120,000 Retained earnings O – $120,000 O - $150,000 O – $150,000

Note that the $120,000 understatement of retained earnings in Year 1 is offset by an


$120,000 overstatement in Year 2. At the end of Year 2, the only misstatement remaining in
retained earnings is due to the Year 2 error.

LOS 1A2ff 8 Inventory Differences Between U.S. GAAP and IFRS

The following table summarizes four differences in the treatment of inventory for financial
reporting purposes between U.S. GAAP and IFRS:

Subject U.S. GAAP Requirements IFRS Requirements


Costing Permitted. Not permitted.
method—LIFO
Consistent use of No requirement to use the same costing The same costing method
costing method method to all inventories. must be used to all inventories
similar in nature or use.
Valuation NRV is used for all costing methods except Inventory is measured at the
for LIFO and retail methods (lower of cost lower of cost or net realizable
or market is used). Market value is the value (NRV). Note that NRV
replacement cost given that it does not is the selling price minus all
exceed the NRV (ceiling) and does not go costs needed to complete and
below NRV—profit margin (floor). sell the items.
Write-downs Write-downs in inventory cost cannot Write-downs in inventory cost
be reversed. are reversed if the reasons
for the decline in value are no
longer there.

1–92 Module 5 A.2. Asset


© Becker Professional Education Corporation. AllValuation: Inventory
rights reserved.
PART 1 UNIT
5 1 A.2. Asset Valuation: Inventory

Question 1 MCQ-12381

The inventory method that will produce the same ending inventory and cost of goods sold
regardless of use of perpetual inventory system or the periodic inventory system would be:
a. Periodic average cost
b. LIFO
c. FIFO
d. Perpetual weighted average

Question 2 MCQ-12382

During a period of decreasing prices, cost of goods sold will be:


a. Higher under LIFO than FIFO
b. Higher under average cost than FIFO
c. Lower under FIFO than LIFO
d. Lower under LIFO than average cost

Question 3 MCQ-12383

Dawson Corp. uses the FIFO perpetual inventory system. During the first six months of
operations, Dawson made the following purchases and sales of inventory:

Purchases
January 1 40 units @ $110 each
February 15 60 units @ $115 each
June 4 75 units @ $125 each
Sales
February 1 25 units
March 15 40 units

What is cost of goods sold for Dawson Corp.?


a. $7,350
b. $8,125
c. $7,275
d. $7,679

© Becker Professional Education Corporation. All rights reserved. Module 5 1–93 A


5 A.2. Asset Valuation: Inventory PART 1 UNIT 1

Question 4 MCQ-12384

Dawson Corp. uses the LIFO perpetual inventory system. During the first six months of
operations, Dawson made the following purchases and sales of inventory:

Purchases
January 1 40 units @ $110 each
February 15 60 units @ $115 each
June 4 75 units @ $125 each
Sales
February 1 25 units
March 15 40 units

What is the cost of goods sold for Dawson Corp.?


a. $7,350
b. $8,125
c. $7,275
d. $7,679

Question 5 MCQ-12386

BGSE Inc. operates a large shoe-making factory. During Year 1, the correct ending finished
goods inventory balance should be $57,000 but is incorrectly reported as $63,000. As a
result, Year 1 ending inventory is overstated by $6,000. In Year 2, ending inventory was
appropriately recorded. Which of the following is true related to Year 2?
Year 2 Beginning Inventory Year 2 Cost of Goods Sold Year 2 Gross Profit
a. Overstated by $6,000 Overstated by $6,000 Understated by $6,000
b. Overstated by $6,000 Understated by $6,000 Overstated by $6,000
c. Understated by $6,000 Understated by $6,000 No effect
d. Understated by $6,000 Overstated by $6,000 Understated by $6,000

1–94 Module 5 A.2. Asset


© Becker Professional Education Corporation. AllValuation: Inventory
rights reserved.
6
MODULE
PART 1 UNIT 1

A.2. Investments and


Long-Term Assets
Part 1
Unit 1

This module covers the following content from the IMA Learning Outcome Statements.

CMA LOS Reference: Part 1—Section A.2. Investments and Long-Term Assets

The candidate should be able to:


j. demonstrate an understanding of the following debt security types: trading,
available-for-sale, and held-to-maturity
k. demonstrate an understanding of the valuation of debt and equity securities
l. determine the effect on the financial statements of using different depreciation methods
m. recommend a depreciation method for a given set of data
n. demonstrate an understanding of the accounting for impairment of long-term assets
and intangible assets, including goodwill
ff. identify and describe the following differences between U.S. GAAP and IFRS: (ii) intangible
assets, with respect to development costs and revaluation; (v) long-lived assets, with
respect to revaluation, depreciation, and capitalization of borrowing costs; and (vi)
impairment of assets, with respect to determination, calculation, and reversal of loss

1 Investments in Debt Securities LOS 1A2j

1.1 Debt Securities


A debt security is any security representing a creditor relationship with an entity.
Debt securities include:
— Corporate bonds
— Redeemable preferred stock
— Government securities
— Convertible debt
— Commercial paper
Debt securities do not include:
— Option, futures, or forward contracts
— Lease contracts
— Accounts and notes receivable

1.2 Classification
Debt securities should be classified into one of three categories, based on the intent of the company.

© Becker Professional Education Corporation. All rights reserved. Module 6 1–95 A.2. Inves
6 A.2. Investments and Long-Term Assets PART 1 UNIT 1

1.2.1 Trading Securities


Trading securities are debt securities that are bought and held principally for the purpose of
selling them in the near term. Trading securities generally reflect active and frequent buying and
selling with the objective of generating profits on short-term differences in price. Debt securities
classified as trading securities are generally reported as current assets, although they can be
reported as non-current, if appropriate.

1.2.2 Available-for-Sale Debt Securities


Available-for-sale debt securities are those not meeting the definitions of the other two
classifications (trading or held-to-maturity). Debt securities classified as available-for-sale
securities are reported as either current assets or non-current assets, depending on the intent of
the corporation.

1.2.3 Held-to-Maturity Debt Securities


Investments in debt securities are classified as held-to-maturity only if the corporation has the
positive intent and ability to hold these securities to maturity. If the intent is to hold the security
for an indefinite period of time, but not necessarily to maturity, then the security is classified
as available-for-sale. If a security can be paid or otherwise settled in a manner that the holder
may not recover substantially all of its investment, the held-to-maturity classification may not
be used. Securities classified as held-to-maturity are reported as current or non-current assets,
based on their time to maturity.

LOS 1A2k 1.3 Valuation


1.3.1 Debt Securities Reported at Fair Value
Debt securities classified as trading and available-for-sale must be reported at fair value.
Fair value is the market price of the security or what a willing buyer and seller would pay and
accept to exchange the security. Changes in the fair value of trading and available-for-sale debt
securities result in unrealized holdings gains or losses. The reporting of these gains or losses in
the financial statements depends on the classification of the securities. Although two general
ledger accounts are normally maintained (i.e., one for the original cost of the security and the
other for the valuation account), the presentation on the balance sheet is one net amount.
Unrealized Gains and Losses (Trading Securities): Unrealized holding gains and losses
on debt securities classified as trading securities are included in earnings. Therefore, the
unrealized gain or loss on trading securities is recognized in net income.
Journal entry to record loss in net income:

DR Unrealized loss on trading securities $XXX


CR Valuation account (fair value adjustment) $XXX

Unrealized Gains and Losses (Available-for-Sale Debt Securities): Unrealized holding


gains and losses on available-for-sale securities are recognized in other comprehensive
income.
Journal entry to record unrealized loss in other comprehensive income:

DR Unrealized loss on available-for-sale securities $XXX


CR Valuation account (fair value adjustment) $XXX

Realized Gains and Losses: Realized gains or losses are recognized when a debt security
is sold and when an available-for-sale debt security is deemed to be impaired. All realized
gains or losses are recognized in net income.

1–96 Module 6 A.2. Investments


© Becker Professional Education Corporation. and Long-Term
All rights reserved.Assets
PART 1 UNIT
6 1 A.2. Investments and Long-Term Assets

1.3.2 Financial Assets Reported at Amortized Cost


Held-to-maturity debt securities are reported at amortized cost. Amortized cost accounting is
described in detail in the bonds section of F5. Unrealized gains and losses on held-to-maturity
securities are not recognized in the financial statements, as held-to-maturity securities are not
marked-to-market at period end.

Classification Balance Sheet Reported Unrealized Gain/Loss Cash Flow

Current or Operating or
Trading Fair value Net income
non‑current investing*

Current or Other comprehensive


Available-for-sale Fair value Investing
non‑current income (PUFIER)

Current or
Held-to-maturity Amortized cost None Investing
non‑current

* Under U.S. GAAP, trading debt security transactions are classified in operating cash flows or
investing cash flows based on the nature and purpose for which the securities were acquired.
If trading debt securities are classified as non-current on the balance sheet, then trading
debt security transactions will be reported as investing cash flows. If trading debt securities
are classified as current on the balance sheet, then trading debt security transactions will be
reported as operating cash flows.

1.4 Reclassification
Transfers between categories should occur only when justified. Transfers from the held-to-
maturity category should be rare and should only be made when there is a change in the entity's
intent to hold a specific security to maturity that does not call into question the entity's intent to
hold other debt securities to maturity. Transfers to and from the trading category should also
be rare.
Any transfer of a particular security from one group (trading, available-for-sale, or held‑to‑maturity)
to another group (trading, available-for-sale, or held-to-maturity) is accounted for at fair value. Any
unrealized holding gain or loss on that security is accounted for as follows:
From Trading Category: The unrealized holding gain or loss at the date of transfer is
already recognized in earnings and shall not be reversed.
To Trading Category: The unrealized holding gain or loss at the date of transfer shall be
recognized in earnings immediately.
Held-to-Maturity Transferred to Available-for-Sale: The unrealized holding gain or loss
at the date of transfer shall be reported in other comprehensive income. Remember that
this debt security was valued at amortized cost as a held-to-maturity security and is being
transferred to a category valued at fair value.
Available-for-Sale Transferred to Held-to-Maturity: The unrealized holding gain or
loss at the date of transfer is already reported in other comprehensive income. The
unrealized holding gain or loss shall be amortized over the remaining life of the security
as an adjustment of yield in a manner consistent with the amortization of any premium
or discount.

© Becker Professional Education Corporation. All rights reserved. Module 6 1–97 A.2. Inves
6 A.2. Investments and Long-Term Assets PART 1 UNIT 1

Summary of Transfers Between Categories

From To Transfer Acct. For Unrealized Holding Gain/Loss

It has already been recognized in income


Trading Any other FV
so no adjustment is necessary

Any other Trading FV Recognized in current earnings

Held-to-maturity Available-for-sale FV Record in other comprehensive income

Amortize gain or loss from other


Available-for-sale Held-to-maturity FV comprehensive income with any bond
premium/discount amortization

1.5 Income From Investments in Debt Securities


Interest income from an investment in debt securities classified as trading or available-for-sale is
recorded on the income statement.
Journal entry to record interest income:

DR Cash $XXX
CR Interest income $XXX

1.6 Impairment of Debt Securities


Under the current expected credit losses (CECL) model, available-for-sale debt securities and
held-to-maturity debt securities should be reported at the net amount expected to be collected
using an allowance for expected credit losses. Expected credit losses are determined based on
current conditions, past experience, and future expectations. A credit loss is recognized as a
current period expense on the income statement and as an offsetting allowance on the balance
sheet. Increases and decreases in expected credit losses are reflected on the income statement
in the period incurred when the estimate of expected credit losses changes.

1.6.1 Impairment of Held-to-Maturity Securities


If it is determined that all amounts due (principal and interest) will not be collected on a debt
investment reported at amortized cost, the investment should be reported at the present value
of the principal and interest that is expected to be collected. The credit loss is the difference
between the present value and the amortized cost.

1–98 Module 6 A.2. Investments


© Becker Professional Education Corporation. and Long-Term
All rights reserved.Assets
PART 1 UNIT
6 1 A.2. Investments and Long-Term Assets

Example 1 Held-to-Maturity Security Impairment

Facts: On January 2, Year 3, TGPO Co. purchased a $500,000, four-year bond at par
with annual interest at 4.25 percent paid on December 31 each year. TGPO classified
the investment as held-to-maturity. At the end of Year 3, TGPO received the full interest
payment of $21,250, but determined that it would only collect $11,500 each year in interest
for the remaining three years (along with the face value of $500,000 at maturity).
Present value of $1 at 4.25 percent for three periods = 0.88262
Present value of an ordinary annuity of 1 at 4.25 percent for three periods = 2.76198
Required: Prepare the entry that TGPO will record at the end of Year 3 to recognize the
impairment.
Solution:
The first step is to calculate the present value as of December 31, Year 3.
Present value:
Interest payments: $11,500 × 2.76198 = $31,763
Principal payment: $500,000 × 0.88262 = $441,310
Total present value = $473,073
The credit loss is calculated as:
Present value − Amortized cost = $473,073 − $500,000 = ($26,927)
The journal entry will be as follows:

DR Credit loss $26,927


CR Allowance for credit losses $26,927

1.6.2 Impairment of Available-for-Sale Debt Securities


Impairment on available-for-sale securities is accounted for differently from impairment on
held-to-maturity securities, because the investor has the option to sell an available-for-sale
security if the loss on the sale will be less than the expected credit loss. As a result, the credit
loss reported in net income on an available-for-sale security is limited to the amount by which
fair value is below amortized cost. Any additional loss is reported as an unrealized loss in other
comprehensive income.

© Becker Professional Education Corporation. All rights reserved. Module 6 1–99 A.2. Inves
6 A.2. Investments and Long-Term Assets PART 1 UNIT 1

Example 2 Available-for-Sale Security Impairment

Facts: The same facts as in Example 1, except the investment is an available-for-sale


debt security.
Required: Determine the expected credit loss and/or unrealized holding gain/loss to be
recognized on December 31, Year 3, for each of the following fair-value scenarios, and
prepare the journal entry for each scenario.
——Scenario 1: $510,000 fair value
——Scenario 2: $480,000 fair value
——Scenario 3: $450,000 fair value
Solution:
Scenario 1 Scenario 2 Scenario 3
Amortized cost, 1/2/Year 3 $500,000 $500,000 $500,000
Fair value, 12/31/Year 3 $510,000 $480,000 $450,000
Expected credit loss* $26,927 $26,927 $26,927
Expected credit loss (net income) 0 $20,000 $26,927
Unrealized gain (OCI) $10,000 0 0
Unrealized loss (OCI) 0 0 $23,073
* The expected credit loss is the difference between present value and amortized cost,
calculated as shown in Example 1 for the held-to-maturity debt security.

For Scenario 1, the journal entry would be:

DR Valuation account (fair value adjustment) $10,000


CR Unrealized gain on available-for-sale security $10,000

For Scenario 2, the journal entry would be:

DR Credit loss $20,000


CR Allowance for credit losses $20,000

For Scenario 3, the journal entry would be:

DR Credit loss $26,927


DR Unrealized loss on available-for-sale security 23,073
CR Allowance for credit losses $26,927
CR Valuation account (fair value adjustment) 23,073

1–100 Module 6 A.2. Investments


© Becker Professional Education Corporation. and Long-Term
All rights reserved.Assets
PART 1 UNIT
6 1 A.2. Investments and Long-Term Assets

1.7 Sale of Debt Securities


A sale of a debt security from any category results in a realized gain or loss and is recognized in
net income for the period. The valuation account, if used, also would have to be removed on the
sale of a security.
Trading Securities: The realized gain or loss reported when a trading debt security is sold
is the difference between the adjusted cost (original cost plus or minus unrealized gains and
losses previously recognized in net income) and the selling price.
Trading securities:

DR Cash $XXX
CR Trading security $XXX
CR Realized gain on trading security XXX

Available-for-Sale Securities: The realized gain or loss reported when an available-for-sale


debt security is sold is the difference between the selling price and the original cost of the
security. Any unrealized gains or losses in accumulated other comprehensive income must
be reversed at the time the security is sold.
Available-for-sale securities:

DR Cash $XXX
DR Unrealized gain on available-for-sale security XXX
CR Available-for-sale security $XXX
CR Realized gain on available-for-sale security XXX

2 Investments in Equity Securities

2.1 Equity Securities


An equity security is a security that represents an ownership interest in an enterprise or the right
to acquire or dispose of an ownership interest in an enterprise at fixed or determinable prices.
Equity securities include:
yy ownership shares (common, preferred, and other forms of capital stock);
yy rights to acquire ownership shares (stock warrants, rights, and call options); and
yy rights to dispose of ownership shares (put options).
Equity securities do not include:
yy preferred stock redeemable at the option of the investor or stock that must be
redeemed by the issuer;
yy treasury stock (the company's own stock repurchased and held); and
yy convertible bonds.

© Becker Professional Education Corporation. All rights reserved. Module 6 1–101 A.2. Inves
6 A.2. Investments and Long-Term Assets PART 1 UNIT 1

2.2 Classification
2.2.1 Fair Value Through Net Income (FVTNI)
Equity securities are generally carried at fair value through net income (FVTNI). This requirement
does not apply to investments accounted for under the equity method, consolidated investees,
or when the practicability exception is applied.

2.2.2 Practicability Exception


The practicability exception allows an entity to measure an equity investment at cost less
impairment, plus/minus observable price changes (in orderly transactions) of identical or
similar investments from the same issuer. This exception is applicable for equity investments
that do not have a readily determinable fair value. Reporting entities that are broker-dealers in
securities, investment companies, or postretirement benefit plans cannot use this exception.

LOS 1A2k 2.3 Valuation


Equity securities are generally reported at fair value through net income (FVTNI). Unrealized
holding gains and losses on equity securities are included in earnings as they occur.
Journal entry to record loss in net income:

DR Unrealized loss on equity security $XXX


CR Valuation account (fair value adjustment) $XXX

2.4 Income From Investments in Equity Securities


Dividend income from an equity security investment is recognized on the income statement as
nonoperating income, unless the dividend is a liquidating dividend.
Normal (Nonliquidating) Dividend
Journal entry to record normal dividend income:

DR Cash $XXX
CR Dividend income $XXX

Liquidating Dividend: A liquidating dividend is a distribution that exceeds the investor's


share of the investee's retained earnings. A liquidating dividend is a return of capital that
decreases the investor's basis in the investment.
Journal entry to record liquidating dividend:

DR Cash $XXX
CR Investment in investee $XXX

1–102 Module 6 A.2. Investments


© Becker Professional Education Corporation. and Long-Term
All rights reserved.Assets
PART 1 UNIT
6 1 A.2. Investments and Long-Term Assets

Example 3 Liquidating Dividend

Facts: ABC Corporation owns a 10 percent interest in XYZ Corporation. During the
current year, XYZ Corp. paid a dividend of $10,000,000. XYZ had retained earnings of
$8,000,000 when the dividend was declared. ABC will receive a dividend of $1,000,000
($10,000,000 × 10%) from XYZ and will record dividend income of $800,000 for its share
of XYZ's retained earnings ($8,000,000 × 10%). The $200,000 difference reduces ABC's
investment in XYZ.
Required: Prepare the journal entry that ABC will record for this liquidating dividend.
Solution:
Journal entry to record $1,000,000 ($10,000,000 × 10%) dividend received from XYZ Corporation:

DR Cash $1,000,000
CR Dividend income ($8,000,000 × 10%) $800,000
CR Investment in XYZ Corporation 200,000

2.5 Impairment
Equity investments that do not have readily determinable fair values are measured at cost minus
impairment (the practicability exception). An entity should consider the following qualitative
indicators in order to determine whether an equity investment with no readily determinable fair
value is impaired:
Heightened concerns regarding the ability of an investee to continue as a going concern due
to factors such as noncompliance with capital or debt requirements, deficiencies in working
capital, or negative operating cash flows.
Significant and adverse changes in the industry, geographic area, technology, or regulatory
or economic environment of the investee.
A significant decline in earnings, business prospects, asset quality, or credit rating of the
investee.
Offers to buy from the investee (and willingness to sell on the part of the investee) the same
or a similar investment for less than the investor's carrying value.
When a qualitative assessment indicates that impairment exists, the cost basis of the security is
written down to fair value and the amount of the write-down is accounted for as a realized loss
and included in earnings.

© Becker Professional Education Corporation. All rights reserved. Module 6 1–103 A.2. Inves
6 A.2. Investments and Long-Term Assets PART 1 UNIT 1

2.6 Sale of Security


The sale of an equity security does not give rise to a gain or loss if all changes in the equity's fair
value have been reported in earnings as unrealized gains or losses as they occurred.
Journal entry for sale with no gain or loss:

DR Cash $XXX
CR Equity security* $XXX

If an entity has not recorded an equity security's change in fair value up to the point of sale, a
gain or loss is recorded at the time of the sale equal to the difference between adjusted cost
(original cost plus or minus unrealized gains and losses previously recognized in earnings) and
the selling price.
The journal entry for sale of equity security with a gain:

DR Cash $XXX
CR Equity security* $XXX
CR Gain on equity security XXX
* Note that any valuation account would also have to be removed when the security is sold.

Example 4 Investments in Debt and Equity Securities

Facts: The following information pertains to Fox Inc.'s portfolio of marketable investments
for the year ended December 31, Year 2:

Fair value at Year 2 activity Fair value at


Cost 12/31/Year 1 Purchases Sales 12/31/Year 2
Held-to-maturity debt securities
Security ABC $100,000 $ 95,000

Available-for-sale debt securities


Security GHI $190,000 $165,000 $175,000
Security JKL 170,000 175,000 160,000

Equity Securities
Security DEF $150,000 $160,000 155,000

Security ABC was purchased at par. There are no expected credit losses on Fox's portfolio of
debt investments.
Required:
1. Calculate the carrying amount of each security on the balance sheet at
December 31, Year 2.
2. Calculate any realized gain or loss recognized in Year 2 net income.
3. Calculate any unrealized gain or loss recognized in Year 2 net income.
4. Calculate any unrealized gain or loss to be recognized in Year 2 other
comprehensive income.

(continued)

1–104 Module 6 A.2. Investments


© Becker Professional Education Corporation. and Long-Term
All rights reserved.Assets
PART 1 UNIT
6 1 A.2. Investments and Long-Term Assets

(continued)

Solution:
1. Carrying amount of each security at December 31, Year 2
Security ABC $100,000
At year-end, the held-to-maturity debt investment is reported at amortized cost,
because there is no expected credit loss. The amortized cost of security ABC is the
purchase price of $100,000.
Security DEF $155,000
The year-end carrying amount of the equity investment is the fair value at year-end.
Fair value of security DEF is $155,000.
Security GHI was sold
Security JKL $160,000
The year-end carrying amount of available-for-sale debt investments is the fair value
at year‑end because there are no expected credit losses. Fair value of security JKL
is $160,000.
2. Realized gain or loss in net income
Security GHI ($15,000)
The $175,000 sales proceeds less the $190,000 cost yields a realized loss of $15,000.
The sale of security GHI will be recorded with the following journal entry:

DR Cash $175,000
DR Realized loss 15,000
CR Security GHI $165,000
CR Unrealized loss (OCI) 25,000

3. Unrealized gain or loss in net income


Security DEF ($5,000)
The unrealized loss on the equity investment is reported in net income. The $160,000
carrying value of security DEF must be reduced to the $155,000 fair value and an
unrealized loss of $5,000 is recognized in net income.
4. Unrealized gain or loss (current year change): Other comprehensive income
Security GHI & JKL (net) $10,000

12/31/Year 1 12/31/Year 2
Accumulated OCI Year 2 OCI Accumulated OCI
Gain (Loss) Gain (Loss) Gain (Loss)
Security GHI ($25,000)1 $25,000 −0−
Security JKL 5,000 2
(15,000) (10,000)
($20,000) $10,000 ($10,000)
1
Security GHI − Loss in 12/31/Y1 AOCI = $165,000 fair value − $190,000 cost = ($25,000)
2
Security JKL − Gain in 12/31/Y1 AOCI = $175,000 fair value − $170,000 cost = $5,000

© Becker Professional Education Corporation. All rights reserved. Module 6 1–105 A.2. Inves
6 A.2. Investments and Long-Term Assets PART 1 UNIT 1

3 Property, Plant, and Equipment

Property, plant, and equipment (PP&E), or fixed assets, are assets that are acquired for use
in operations and not as inventory for resale.
They possess physical substance, are long-term in nature, and are subject to depreciation.
The following fixed assets must be shown separately on the balance sheet (or footnotes) at
original cost (historical cost):
Land (Property)
Buildings (Plant)
Equipment: May show machinery, tools, furniture, and fixtures separately, if these
categories are significant.
Accumulated Depreciation Account (Contra-Asset): May be combined for two or more
asset categories.

LOS 1A2ff 3.1 Valuation of Fixed Assets Under U.S. GAAP


Historical cost is the basis for valuation of purchased fixed assets. Historical cost is measured
by the cash or cash equivalent price of obtaining the asset and bringing it to the location and
condition necessary for its intended use.

3.2 Valuation of Fixed Assets Under IFRS


Under IFRS, fixed assets are initially recognized at the cost to acquire the asset. Subsequent to
acquisition, fixed assets can be valued using the cost model or the revaluation model.

3.2.1 Cost Model


Under the cost model, fixed assets are reported at historical cost adjusted for accumulated
depreciation and impairment.

Cost model Historical Accumulated


= – – Impairment
carrying value cost depreciation

3.2.2 Revaluation Model


Under the revaluation model, a class of fixed assets is revalued to fair value and then reported
at fair value less subsequent accumulated depreciation and impairment. Revaluations must be
made frequently enough to ensure that carrying amount does not differ materially from fair
value at the end of the reporting period. When fair value differs materially from carrying value, a
further revaluation is required.

Revaluation model Fair value at Subsequent accumulated Subsequent


= – –
carrying value revaluation date depreciation impairment

1–106 Module 6 A.2. Investments


© Becker Professional Education Corporation. and Long-Term
All rights reserved.Assets
PART 1 UNIT
6 1 A.2. Investments and Long-Term Assets

Revaluation must be applied to all items in a class of fixed assets, not to individual fixed assets.
Land and buildings, machinery, furniture and fixtures, and office equipment are examples of
fixed asset classes. When fixed assets are reported at fair value, the historical cost equivalent
(cost − accumulated depreciation − impairment) must be disclosed.
Revaluation Losses
When fixed assets are revalued, revaluation losses (fair value < carrying value before
revaluation) are reported on the income statement, unless the revaluation loss reverses
a previously recognized revaluation gain. A revaluation loss that reverses a previously
recognized revaluation gain is recognized in other comprehensive income and reduces the
revaluation surplus in accumulated other comprehensive income.
Revaluation Gains
Revaluation gains (fair value > carrying value before revaluation) are reported in other
comprehensive income and accumulated in equity as revaluation surplus, unless the revaluation
gain reverses a previously recognized revaluation loss. Revaluation gains are reported on the
income statement to the extent that they reverse a previously recognized revaluation loss.
Impairment
If revalued fixed assets subsequently become impaired, the impairment is recorded by first
reducing any revaluation surplus to zero with further impairment losses reported on the
income statement.

Example 5 Revaluation Model (IFRS)

Facts: On December 31, Year 1, an entity chose to revalue all of its fixed assets under IFRS.
On that date, the fixed assets had the following carrying values and fair values:

Carrying Value Fair Value


Land $10,500,000 $11,100,000
Buildings 6,400,000 6,000,000
Equipment 3,300,000 3,600,000
Required: Calculate the revaluation gain and loss to be reported on the December 31,
Year 1, financial statements.

(continued)

© Becker Professional Education Corporation. All rights reserved. Module 6 1–107 A.2. Inves
6 A.2. Investments and Long-Term Assets PART 1 UNIT 1

(continued)

Solution:
Revaluation Loss: The entity will report a loss on the revaluation of the buildings because
fair value is less than carrying value:
Loss on revaluation of buildings = $6,000,000 − $6,400,000
= ($400,000)
The loss, which is essentially an impairment loss, will be reported on the income statement.
Revaluation Gain: The entity will report a gain on the revaluation of the land and
equipment because the fair values of these assets exceed their respective carrying values:
Gain on revaluation of land = $11,100,000 − $10,500,000
= $600,000
Gain on revaluation of equipment = $3,600,000 − $3,300,000
= $300,000
The total revaluation gain of $900,000 would be reported as revaluation surplus in other
comprehensive income.

4 Depreciation

The basic principle of matching revenue and expenses is applied to long-lived assets that are
not held for sale in the ordinary course of business. The systematic and rational allocation
used to achieve "matching" is usually accomplished by depreciation, amortization, or depletion,
according to the type of long-lived asset involved.

4.1 Types of Depreciation


Physical Depreciation: This type of depreciation is related to an asset's deterioration and
wear over a period of time.
Functional Depreciation: Functional depreciation arises from obsolescence or inadequacy
of the asset to perform efficiently. Obsolescence may result from diminished demand
for the product that the depreciable asset produces or from the availability of a new
depreciable asset that can perform the same function for substantially less cost.

4.2 Definitions
Salvage Value: Salvage or residual value is an estimate of the amount that will be realized
at the end of the useful life of a depreciable asset. Frequently, depreciable assets have little
or no salvage value at the end of their estimated useful life and, if immaterial, the amount(s)
may be ignored in calculating depreciation.
Estimated Useful Life: Estimated useful life is the period of time over which an asset's cost
will be depreciated. It may be revised at any time, but any revision must be accounted for
prospectively, in current and future periods only (change in estimate).

1–108 Module 6 A.2. Investments


© Becker Professional Education Corporation. and Long-Term
All rights reserved.Assets
PART 1 UNIT
6 1 A.2. Investments and Long-Term Assets

Pass Key

Sometimes an asset will be placed in service during the year. Therefore, it requires
computing depreciation for a part of the year rather than the full year. Candidates must
always check the date the asset was placed in service.

4.3 Composite Depreciation and Component Depreciation


Under IFRS, the depreciation method used should reflect the expected pattern of fixed asset
consumption. This is not required under U.S. GAAP.

4.3.1 Component Depreciation


Component depreciation is the separate depreciation of each part of an item of property, plant,
and equipment that is significant to the total cost of the fixed asset. Component depreciation is
permitted but rarely used under U.S. GAAP.
IFRS requires component depreciation. Separate significant components of a fixed asset with
different lives should be recorded and depreciated separately. The carrying amount of parts or
components that are replaced should be derecognized.

Example 6 Component Depreciation

Facts: On January 1, Year 1, an entity that uses IFRS acquired a machine with a cost of
$250,000 and an estimated life of 20 years. The cost of the machine included the cost of
a cylinder that must be replaced every five years for $20,000 and an inspection cost of
$5,000. The machine must be reinspected every 10 years at an additional cost of $5,000
per inspection.
Required: Compute Year 1 depreciation using component depreciation.
Solution: Under the component approach, the machine, the cylinder, and the inspection
cost are recognized and depreciated separately:
Cost Useful Life Depreciation
Machine $225,000 20 $11,250
Cylinder 20,000 5 4,000
Inspection cost 5,000 10 500
Total $250,000 $15,750

4.3.2 Composite or Group Depreciation


Composite depreciation is the process of averaging the economic lives of a number of property
units and depreciating the entire class of assets over a single life (e.g., all at five years), thus
simplifying record keeping of assets and depreciation calculations.

© Becker Professional Education Corporation. All rights reserved. Module 6 1–109 A.2. Inves
6 A.2. Investments and Long-Term Assets PART 1 UNIT 1

Asset Retirement
When a group or composite asset is sold or retired, the accumulated depreciation is
treated differently from the accumulated depreciation of a single asset. If the average
service life of the group of assets has not been reached when an asset is retired, the gain
or loss that results is absorbed in the accumulated depreciation account. The accumulated
depreciation account is debited (credited) for the difference between the original cost and
the cash received.
Depreciation Methods
Composite and component depreciation can be done using any acceptable depreciation
method, including the straight-line, sum-of-the-years'-digits, and declining balance methods.

Example 7 Composite (Group) Depreciation

Facts: A schedule of machinery owned by Lester Manufacturing Company is


presented below:

Estimated Estimated
Total Cost Salvage Value Life in Years
Machine A $550,000 $50,000 20
Machine B 200,000 20,000 15
Machine C 40,000 – 5

Lester computes depreciation on the straight-line method.


Required: Depreciate the machinery using composite depreciation.
Solution: Based on the information presented, the composite life of these assets (in years)
should be 16 years, computed as follows:

Estimated Estimated
Total Salvage Depreciable Life in Annual
Machine Cost Value Cost Years Depreciation
A $550,000 $50,000 $500,000 20 $25,000
B 200,000 20,000 180,000 15 12,000
C 40,000 – 40,000 5 8,000
Totals $790,000 – $70,000 = $720,000 $45,000

Average composite life = $720,000 divided by $45,000 = 16 years

1–110 Module 6 A.2. Investments


© Becker Professional Education Corporation. and Long-Term
All rights reserved.Assets
PART 1 UNIT
6 1 A.2. Investments and Long-Term Assets

Illustration 1 Disposal of Group or Composite Asset

Assume that the Lester Company sells Machine A, that originally cost $550,000, in 10 years
for $260,000. Because the loss on disposal is not recognized, accumulated depreciation
must be reduced or debited.
The journal entry is as follows:

DR Cash $260,000
DR Accumulated depreciation 290,000
CR Asset A $550,000

4.4 Basic Depreciation Methods


The goal of a depreciation method should be to provide for a reasonable, consistent matching
of revenue and expense by systematically allocating the cost of the depreciable asset over its
estimated useful life.
The actual accumulation of depreciation in the books is accomplished by using a contra-account,
such as accumulated depreciation or allowance for depreciation.

4.4.1 Straight-Line Depreciation


Straight-line depreciation is determined by the formula:

Cost − Salvage value


= Depreciation
Estimated useful life

Estimated useful life is usually stated in periods of time, such as years or months.

Illustration 2 Straight-Line Depreciation

Assume that an asset cost $11,000, has a salvage value of $1,000, and has an estimated
useful life of five years.

$11,000 − $1,000
= $2,000 depreciation per year
5 years

If the asset was acquired within the year instead of at the beginning of the year, partial
depreciation expense is taken in the first year.

© Becker Professional Education Corporation. All rights reserved. Module 6 1–111 A.2. Inves
6 A.2. Investments and Long-Term Assets PART 1 UNIT 1

4.4.2 Sum-of-the-Years'-Digits Depreciation


The sum-of-the-years'-digits method is one of the accelerated methods of depreciation that
provides higher depreciation expense in the early years and lower charges in the later years.
To find the sum-of-the-years'-digits, each year is progressively numbered and then added. For
example, the sum-of-the-years'-digits for a five-year life would be:
1 + 2 + 3 + 4 + 5 = 15
For four years: 1 + 2 + 3 + 4 = 10
For three years: 1 + 2 + 3 = 6
The sum-of-the-years'-digits becomes the denominator. The numerator is the remaining life of
the asset at the beginning of the current year. For example, the first year's depreciation for a
five-year life would be 5/15 of the depreciable base of the asset.

Remaining life of asset


Depreciation expense = (Cost − Salvage value) ×
Sum-of-the-years' digits

When dealing with an asset with a long life, use the general formula for finding the
sum-of-the-years'-digits:

N × (N + 1)
S =
2
Where:
N = Estimated useful life

Example 8 Sum-of-the-Years'-Digits Method

Facts: Assume that an asset cost $11,000, has a salvage value of $1,000, and has an
estimated useful life of four years.
Required: Calculate the amount of depreciation expense for each of the four years of the
asset's useful life.

(continued)

1–112 Module 6 A.2. Investments


© Becker Professional Education Corporation. and Long-Term
All rights reserved.Assets
PART 1 UNIT
6 1 A.2. Investments and Long-Term Assets

(continued)

Solution: The first step is to determine the depreciable base:


Cost of asset $11,000
Less: salvage value (1,000)
Depreciable base $10,000

The sum-of-the-years'-digits for four years is:

1 + 2 + 3 + 4 = 10

The first year's depreciation is 4/10, the second year's is 3/10, the third year's is 2/10, and
the fourth year's is 1/10, as follows:
1st Year: 4/10 × $10,000 = $4,000
2nd Year: 3/10 × $10,000 = 3,000
3rd Year: 2/10 × $10,000 = 2,000
4th Year: 1/10 × $10,000 = 1,000
Total depreciation = $10,000

4.4.3 Units-of-Production (Productive Output) Depreciation


The units-of-production method relates depreciation to the estimated production capability of
an asset and is expressed in a rate per unit or hour.
The formula is:

Cost − Salvage value


= Rate per unit or hour
Estimated units or hours

Rate per unit Number of units produced Depreciation


× =
(or hour) (or hours worked) expense

4.4.4 Declining Balance Depreciation


The most common of these accelerated methods is the double-declining-balance method,
although other alternative (less than double) methods are acceptable.
Under double-declining balance, each year's depreciation rate is double the straight-line rate. In
the final year, the asset is depreciated to its salvage value, if any.
Double-declining-balance depreciation is calculated using the following formula, where n is the
number of years of the asset's useful life:

1
Depreciation expense = 2 × × (Cost − Accumulated depreciation)
n

© Becker Professional Education Corporation. All rights reserved. Module 6 1–113 A.2. Inves
6 A.2. Investments and Long-Term Assets PART 1 UNIT 1

No allowance is made for salvage value because the method always leaves a remaining balance,
which is treated as salvage value. However, the asset should not be depreciated below the
estimated salvage value.

Pass Key

The only methods that ignore salvage value in the annual calculation of depreciation are the
declining balance methods. Salvage value is only used as the limitation on total depreciation.

Example 9 Double-Declining-Balance Method

Facts: An asset costing $10,000 with a salvage value of $2,000 has an estimated useful life
of 10 years.
Required: Using the double-declining-balance (200%) method, calculate the depreciation
expense for each year of the useful life of the asset.
Solution: First, the regular straight-line method percentage is determined, which in this
case is 10 percent (10-year life). The amount is multiplied by two resulting in 20 percent and
applied each year to the remaining book value, as follows:

Amount of
Double Net Book Value Depreciation
Year Percentage Remaining Expense
1 20 $10,000 $2,000
2 20 8,000 1,600
3 20 6,400 1,280
4 20 5,120 1,024
5 20 4,096 819
6 20 3,277 655
7 20 2,622 524
8 20 2,098 98
Salvage value 2,000 0

Note: Had the preceding illustration been 1½ times declining balance (150 percent), the
rate would have been 15 percent of the remaining book value (10% × 1.5).
If the asset had been placed in service halfway through the year, the first year's
depreciation would have been $1,000 (one-half of $2,000), and the second year's
depreciation would have been 20 percent of $9,000 (remaining value after the first year),
or $1,800.
In Year 8, only $98 depreciation expense is taken because, regardless of the method, book
value cannot drop below salvage value. In addition, no depreciation expense is recorded in
Years 9 and 10.

1–114 Module 6 A.2. Investments


© Becker Professional Education Corporation. and Long-Term
All rights reserved.Assets
PART 1 UNIT
6 1 A.2. Investments and Long-Term Assets

4.4.5 Partial-Year Depreciation


When an asset is placed in service during the year, the depreciation expense is taken only for the
portion of the year that the asset is used. For example, if an asset (of a company on a calendar
year basis) is placed in service on July 1, only six months' depreciation is taken.

4.5 Effects of Depreciation on Net Income LOS 1A2l

The method of deprecation elected impacts the reported income on the income statement.
Straight-line depreciation results in a uniform amount of depreciation expense each year of
the asset's useful life. Straight-line is the easiest to calculate and has a consistent effect on
income from period to period.
Double-declining-balance depreciation results in higher depreciation expense in the early
years of the life of an asset. Net income will be lower in the earlier years due to the higher
depreciation expense and higher in the later years as depreciation expense decreases.
Sum-of-the-years' digits depreciation results in higher depreciation earlier in the life of an
asset. Depreciation expense will decrease each year. This method is rarely used in practice,
but it has the same effect as other accelerated methods like declining balance.
Units-of-production depreciation measures depreciation based on the usage of an asset during
the period. The effect on net income is variable, as more depreciation expense is taken when
the asset is used more heavily. Greater usage implies greater production, which hopefully aligns
with an increase in sales, matching the higher expense against the periods of higher revenues.

4.6 Determining the Appropriate Method of Depreciation LOS 1A2m

Factors to consider when determining the most appropriate method of deprecation include
evaluating the following:
How will the asset be used by the business?
Will the benefit be received evenly over the useful life or will the majority of the benefit be
received earlier on?
Is there a measurable output produced by the company?

Illustration 3 Depreciation Method Selection

Dawson Diggers Inc. purchased several new assets during the current fiscal year.
Management has been discussing depreciation alternatives for assets purchased and
determined the following:
— Assets Purchased:
— Building: The building has an estimated useful life of 25 years and will provide a
uniform benefit over its useful life. There is no measurable output for the building.
— Equipment: The equipment has an estimated useful life of 5 years. The equipment
will be used in the construction process and is estimated to have 100,000 hours of run
time at the time of acquisition. Usage of the equipment will vary each year based on
the number of jobs contracted with customers.
The company determines the straight-line method of depreciation is appropriate for the
building because benefit is received uniformly over the life of the asset. The equipment will
be depreciated using units of production based on management's assessment of the life of
the machine in terms of hours utilized. Each year, the amount of depreciation determined
will be based on hours used for the equipment.

© Becker Professional Education Corporation. All rights reserved. Module 6 1–115 A.2. Inves
6 A.2. Investments and Long-Term Assets PART 1 UNIT 1

Pass Key

Note: Management may elect to utilize any method of depreciation available under U.S.
GAAP to allocate cost over the period of benefit for fixed assets. Management is free to
apply one method of depreciation to all asset classes or utilize different depreciation
methods for different classes of assets.

5 Intangible Assets

Intangible assets are long-lived legal rights and competitive advantages developed or acquired
by a business enterprise. They are typically acquired to be used in operations of a business and
provide benefits over several accounting periods.
Intangible assets differ considerably in their characteristics, useful lives, and relationship to
operations of an enterprise, and are classified accordingly.

5.1 Classification of Intangible Assets


Patents, copyrights, franchises, trademarks, and goodwill are common intangible assets.
Intangible assets may be either specifically identifiable (e.g., patents, copyrights,
franchise, etc.) or not specifically identifiable (e.g., goodwill).

5.1.1 Manner of Acquisition


Purchased Intangible Assets
Intangible assets acquired from other enterprises or individuals should be recorded as an
asset at cost. Legal and registration fees incurred to obtain an intangible asset should also
be capitalized.
Internally Developed Intangible Assets
yy Under U.S. GAAP, the cost of intangible assets not acquired from others (i.e., developed
internally) should be expensed when incurred because U.S. GAAP prohibits the
capitalization of research and development costs.
yy Examples (must be expensed):
—Trademarks
— (except for the capitalizable costs identified below)
—Goodwill
— from advertising
—The
— cost of developing, maintaining, or restoring goodwill
yy The exception is that certain costs associated with internally developed intangibles that
are specifically identifiable can be capitalized, such as:
—Legal
— fees and other costs related to a successful defense of the asset
—Registration
— or consulting fees
—Design
— costs (e.g., of a trademark)
—Other
— direct costs to secure the asset

1–116 Module 6 A.2. Investments


© Becker Professional Education Corporation. and Long-Term
All rights reserved.Assets
PART 1 UNIT
6 1 A.2. Investments and Long-Term Assets

5.1.2 Expected Period of Benefit


Classification of the intangible asset depends on whether the economic life can be determined
or is indeterminable.

5.1.3 Separability LOS 1A2ff


The classification of the intangible asset depends on whether the asset can be separated from
the entity (e.g., a patent) or is substantially inseparable from it (e.g., a trade name or goodwill).
Under IFRS, research costs related to an internally developed intangible asset must be expensed,
but an intangible asset arising from development is recognized if the entity can demonstrate all
of the following:
— Technological feasibility has been established.
— The entity intends to complete the intangible asset.
— The entity has the ability to use or sell the intangible asset.
— The intangible asset will generate future economic benefits.
— Adequate resources are available to complete the development and sell or use the asset.
The entity can reliably measure the expenditure attributable to the development of the
intangible asset.

5.2 Capitalization of Costs


A company should record the cost of intangible assets acquired from other enterprises or
individuals in an "arm's-length" transaction as assets.
Cost is measured by:
— the amount of cash disbursed or the fair value of other assets distributed;
— the present value of amounts to be paid for liabilities incurred; and
— the fair value of consideration received for stock issued.
Cost may be determined either by the fair value of the consideration given or by the fair
value of the property acquired, whichever is more clearly evident.
The cost of unidentifiable intangible assets is measured as the difference between the
cost of the group of assets or enterprise acquired and the sum of the costs assigned to
identifiable assets acquired, less liabilities assumed.
The cost of identifiable assets should not include goodwill.

5.3 Amortization
The value of intangible assets eventually disappears; therefore, the cost of each type of
intangible asset (except for goodwill and assets with indefinite lives) should be amortized by
systematic charges to income over the period estimated to be benefited.

Pass Key

A patent is amortized over the shorter of its estimated life or remaining legal life.

© Becker Professional Education Corporation. All rights reserved. Module 6 1–117 A.2. Inves
6 A.2. Investments and Long-Term Assets PART 1 UNIT 1

5.3.1 Method
The straight-line method of amortization should be applied, unless a company demonstrates
that another systematic method is more appropriate. The method and estimated useful lives
of intangible assets should be adequately disclosed in the notes to the financial statements.
Expenses that increase the useful life of the intangible asset require an adjustment to the
calculation of the annual amortization.

5.3.2 Change in Useful Life


If the life of an existing intangible asset is reduced or extended, the remaining net book value is
amortized over the new remaining life.

5.4 Sale of Intangible Assets


If an intangible asset is sold, simply compare its carrying value at the date of sale with the selling
price to determine the gain or loss.

5.5 Valuation
5.5.1 U.S. GAAP
Under U.S. GAAP, finite life intangible assets are reported at cost less amortization and
impairment. Indefinite life intangible assets are reported at cost less impairment.

LOS 1A2ff 5.5.2 IFRS


Under IFRS, intangible assets can be reported under either the cost model or the revaluation model.
Cost Model: Under the cost model, intangible assets are reported at cost adjusted for
amortization (finite life intangible assets only) and impairment.
Revaluation Model: Under the revaluation model, intangible assets are initially recognized
at cost and then revalued to fair value at a subsequent revaluation date. Revalued
intangible assets are reported at fair value on the revaluation date adjusted for subsequent
amortization (finite life intangible assets only) and subsequent impairment.

Revaluation model Fair value on Subsequent Subsequent


= – –
carrying value revaluation date amortization impairment

Revaluations must be performed regularly so that at the end of each reporting period
the carrying value of the intangible asset does not differ materially from fair value. If an
intangible asset is accounted for using the revaluation model, all other assets in its class
must also be revalued unless there is no active market for the intangible assets.
— Revaluation Losses: Revaluation losses (fair value on revaluation date < carrying value
before revaluation) are reported on the income statement, unless the revaluation loss
reverses a previously recognized revaluation gain. A revaluation loss that reverses a
previously recognized revaluation gain is recognized in other comprehensive income
and reduces the revaluation surplus in accumulated other comprehensive income.

1–118 Module 6 A.2. Investments


© Becker Professional Education Corporation. and Long-Term
All rights reserved.Assets
PART 1 UNIT
6 1 A.2. Investments and Long-Term Assets

— Revaluation Gains: Revaluation gains (fair value on revaluation date > carrying value
before revaluation) are reported in other comprehensive income and accumulated
in equity as revaluation surplus, unless the revaluation gain reverses a previously
recognized revaluation loss. Revaluation gains are reported on the income statement to
the extent that they reverse a previously recognized revaluation loss.
— Impairment: If revalued intangible assets subsequently become impaired, the
impairment is recorded by first reducing any revaluation surplus in equity to zero with
further impairment losses reported on the income statement.

Example 10 IFRS Intangible Asset Revaluation

Facts: On December 31, Year 2, an entity that had adopted the IFRS revaluation model in
Year 1 adjusted its patents to fair value. On that date, the patents had the following carrying
value and fair value:
Carrying Value Fair Value
Patents $8,200,000 $9,100,000
The entity had recorded a revaluation loss of $500,000 in Year 1.
Required: Compute the revaluation gains to be reported in Year 2 net income and other
comprehensive income.
Solution: Total revaluation gain = $9,100,000 − $8,200,000 = $900,000
Of this gain, $500,000 will be reported on the income statement as a reversal of the $500,000
revaluation loss reported in Year 1. The remaining $400,000 ($900,000 − $500,000) gain will
be reported in other comprehensive income as revaluation surplus.

6 Impairment of Assets LOS 1A2n

6.1 Long-Term Assets and Intangible Assets


PP&E and intangible assets are tested for impairment when events or circumstances indicate the
book value may not be recoverable. Intangible assets include finite life intangible assets, such as
patents, copyrights, and trademarks, and indefinite life intangible assets such as goodwill.

6.2 Property, Plant, and Equipment


The carrying amounts of fixed assets held for use and to be disposed of need to be reviewed
at least annually or whenever events or changes in circumstances indicate that the carrying
amount may not be recoverable.

6.2.1 Test for Recoverability


When a fixed asset is tested for impairment, the future cash flows expected to result from the
use of the asset and its eventual disposition need to be estimated. If the sum of undiscounted
expected (future) cash flows is less than the carrying amount, an impairment loss needs to
be recognized.

© Becker Professional Education Corporation. All rights reserved. Module 6 1–119 A.2. Inves
6 A.2. Investments and Long-Term Assets PART 1 UNIT 1

6.2.2 Calculation of the Impairment Loss


The impairment loss is calculated as the amount by which the carrying amount exceeds the fair
value of the asset.

Undiscounted future net cash flows*


< Net carrying value >

Positive Negative

No impairment loss Impairment

Assets held Assets held


for use for disposal

FV or PV future net cash flows FV or PV future net cash flows


< Net carrying value > < Net carrying value >

Impairment loss Impairment loss


+ Cost of disposal
1. Write asset down Total impairment loss
2. Amortize new cost
3. Restoration not permitted 1. Write asset down
2. No amortization taken
3. Restoration permitted

6.2.3 Reporting the Impairment Loss: General


The impairment loss is reported as a component of income from continuing operations before
income taxes or in a statement of activities (related to not-for-profit entities). The impairment
loss is recognized by reducing the carrying value of the asset to its lower fair value. Restoration
of previously recognized impairment losses is prohibited under U.S. GAAP unless the asset is
held for disposal.
A fixed asset impairment loss under IFRS is calculated using a one-step model in which the
carrying value of the fixed asset is compared with the fixed asset's recoverable amount. IFRSs
define the recoverable amount as the greater of the asset's fair value less costs to sell and the
asset's value in use. Value in use is the present value of the future cash flows expected from the
fixed asset. IFRSs allow the reversal of impairment losses.

1–120 Module 6 A.2. Investments


© Becker Professional Education Corporation. and Long-Term
All rights reserved.Assets
PART 1 UNIT
6 1 A.2. Investments and Long-Term Assets

6.3 Impairment of Intangible Assets Other Than Goodwill


Intangible assets with indefinite useful life (including goodwill) are tested for impairment at least
annually, and intangible assets with finite useful life are tested whenever events or changes in
circumstances indicate that the carrying amount may not be recoverable.
Under U.S. GAAP, the impairment test applied to an intangible asset other than goodwill is
determined by the asset's life. An intangible asset has a finite life when it is possible to estimate
the useful life of the asset. If it is not possible to determine the useful life of an intangible asset,
then the asset has an indefinite (not infinite) life. If an intangible asset has a finite life, it is
amortized over that life. If it has an indefinite life, it is not amortized.

6.3.1 Intangible Assets With Finite Lives (Two-Step Impairment Test)


An intangible asset with a finite life is tested for impairment using a two-step impairment test.
Step 1: The carrying amount of the asset is compared with the sum of the undiscounted
cash flows expected to result from the use of the asset and its eventual disposition.
Step 2: If the carrying amount exceeds the total undiscounted future cash flows, then the
asset is impaired and an impairment loss equal to the difference between the carrying
amount of the asset and its fair value is recorded.

Pass Key

It is important to note the following when testing property, plant, and equipment, or an
intangible asset with a finite life, for impairment:
——Determining the Impairment: Use undiscounted future net cash flows.
——Amount of Impairment: Use fair value (FV).

6.3.2 Intangible Assets With Indefinite Lives (One‑Step Impairment Test)


When testing an intangible asset with an indefinite life (including goodwill) for impairment, it
is generally not possible to estimate total future cash flows expected to result from the use of
the assets and its disposition. As a result, an intangible asset with an indefinite life is tested for
impairment by comparing the fair value of the intangible asset to its carrying amount. If the
asset's fair value is less than its carrying amount, an impairment loss is recognized in an amount
equal to the difference.
This quantitative impairment test is not necessary if, after assessing relevant qualitative factors,
an entity determines that it is not more likely than not that the fair value of the indefinite-life
intangible asset is less than its carrying amount.

6.3.3 Reporting an Impairment Loss


An impairment loss is reported as a component of income from continuing operations before
income taxes, unless the impairment loss is related to discontinued operations. The carrying
amount of the asset is reduced by the amount of the impairment loss. Restoration of previously
recognized impairment losses is prohibited, unless the asset is held for disposal.

© Becker Professional Education Corporation. All rights reserved. Module 6 1–121 A.2. Inves
6 A.2. Investments and Long-Term Assets PART 1 UNIT 1

Pass Key

U.S. GAAP
Finite Life Indefinite Life
Characteristics Useful life is limited Life extends beyond the foreseeable
future or cannot be determined
Amortization Over useful economic life None
Impairment test Two-step test: One-step test:
— Undiscounted net cash flows — Fair value
— Fair value

6.3.4 Calculation of the Impairment Loss


The impairment loss is calculated as the amount by which the carrying amount exceeds the fair
value of the asset.
Impairment Test

Qualitative test for impairment


(indefinite life intangibles only)

Undiscounted future net cash flows*


< Net carrying value >

Positive Negative

No impairment loss Impairment

Assets held Assets held


for use for disposal

Fair value Fair value


< Net carrying value > < Net carrying value >

Impairment loss Impairment loss


+ Cost of disposal
1. Write asset down Total impairment loss
2. Amortize new cost
3. Restoration not permitted 1. Write asset down
2. No amortization taken
3. Restoration permitted

* When testing indefinite-life intangible assets for impairment, fair value must be used instead of
undiscounted future net cash flows:
Fair value − Net carrying value = Positive (no impairment) or negative (impairment)

1–122 Module 6 A.2. Investments


© Becker Professional Education Corporation. and Long-Term
All rights reserved.Assets
PART 1 UNIT
6 1 A.2. Investments and Long-Term Assets

Illustration 4 No Impairment Loss

Facts:
— Asset's net carrying value is $900,000.
— Net future cash flows are projected as $1,000,000.
$1,000,000
(900,000)
$ 100,000

No impairment loss

Illustration 5 Impairment Loss

Facts:
Asset's net carrying value is $1,200,000.
Net future cash flows are projected as $1,000,000.
Assumption 1: Asset held for use, and
— FV/PV net cash flows are $700,000.
Assumption 2: Asset is held for disposal, and
— FV/PV net cash flows are $700,000.
— Cost of disposal will be $100,000.
$1,000,000
(1,200,000)
$ (200,000)

Impairment
Assets held Assets held
for use for disposal

$ 700,000 $ 700,000
(1,200,000) (1,200,000)
$ 500,000 $ 500,000
+ 100,000
1. Write asset down $ 600,000
2. Amortize new cost
3. Restoration not permitted 1. Write asset down
2. No amortization taken
3. Restoration permitted

© Becker Professional Education Corporation. All rights reserved. Module 6 1–123 A.2. Inves
6 A.2. Investments and Long-Term Assets PART 1 UNIT 1

6.4 Goodwill Impairment


Under U.S. GAAP, goodwill impairment is calculated at the reporting unit level. Impairment
exists when the carrying amount of the reporting unit including goodwill exceeds its fair value
including goodwill.

6.4.1 Definition of Reporting Unit


A reporting unit is an operating segment, or one level below an operating segment. The goodwill
of one reporting unit may be impaired, while the goodwill for other reporting units may or may
not be impaired.

6.4.2 Qualitative Evaluation of Goodwill Impairment


Under U.S. GAAP, the goodwill impairment test has been simplified by allowing companies
to test qualitative factors to determine whether it is necessary to perform the quantitative
goodwill impairment test. Examples of qualitative factors include:
— Macroeconomic conditions
— Overall financial performance
— Entity-specific events such as bankruptcy, litigation, or changes in management,
strategy, or customers
— Industry and market conditions
— Sustained decrease in share-price
— Cost factors that could have a negative effect on earnings and cash flows
The quantitative impairment test is not necessary if, after assessing the relevant qualitative
factors, an entity determines that it is not more likely than not that the fair value of the
reporting unit is less than its carrying amount.
If the qualitative assessment indicates that there is a greater than 50 percent chance that
the fair value of the reporting unit is less than its carrying amount, then the entity must
perform the quantitative impairment test.

LOS 1A2ff 6.4.3 Quantitative Evaluation of Goodwill Impairment


The evaluation of goodwill impairment involves comparing the carrying value of the reporting
unit including goodwill to the fair value of the reporting unit including goodwill. If the fair value
exceeds the carrying value, there is no impairment. If the fair value is less than the carrying
value, there will be an impairment charge equal to the difference between the fair value and
carrying value. The impairment charge cannot exceed the value of the goodwill that is allocated
to that reporting unit.

1–124 Module 6 A.2. Investments


© Becker Professional Education Corporation. and Long-Term
All rights reserved.Assets
PART 1 UNIT
6 1 A.2. Investments and Long-Term Assets

Example 11 Goodwill Impairment (U.S. GAAP)

Facts: Omega Inc. has two reporting units, Alpha and Beta, which have book values
including goodwill of $500,000 and $675,000, respectively. Alpha reports goodwill of
$50,000 and Beta reports goodwill of $75,000. As part of the company's annual review for
goodwill impairment, Omega determined that the fair values including goodwill of Alpha
and Beta were $480,000 and $700,000, respectively, at December 31, Year 1.
Required: Determine whether the reporting units' goodwill is potentially impaired.
Solution:
——Alpha: Reporting Unit FV − Reporting Unit BV = $480,000 − 500,000 = ($20,000)
——Beta: Reporting Unit FV − Reporting Unit BV = $700,000 − 675,000 = $25,000
Because Alpha's fair value is less than its book value, there is goodwill impairment. Beta's
goodwill is not impaired.
The impairment charge for Alpha will be equal to the difference between the book value of
$500,000 and the fair value of $480,000. Because this difference of $20,000 is less than Alpha's
reported goodwill of $50,000, the full $20,000 will be recognized as an impairment loss. If
goodwill had been reported as $15,000 instead of $50,000, the impairment charge would have
been capped at $15,000.
Journal entry to record goodwill impairment at December 31, Year 1:

DR Loss due to impairment $20,000


CR Goodwill $20,000

Under IFRS, goodwill impairment testing is done at the cash-generating unit (CGU) level. A cash-
generating unit is defined as the smallest identifiable group of assets that generates cash inflows
that are largely independent of the cash inflows from other assets or groups of assets. The
goodwill impairment test is a one-step test in which the carrying value of the CGU is compared
with the CGU's recoverable amount, which is the greater of the CGU's fair value less costs to sell
and its value in use. Value in use is the present value of the future cash flows expected from
the CGU. An impairment loss is recognized to the extent that the carrying value exceeds the
recoverable amount. The impairment loss is first allocated to goodwill and then allocated on a
pro rata basis to the other assets of the CGU.

© Becker Professional Education Corporation. All rights reserved. Module 6 1–125 A.2. Inves
6 A.2. Investments and Long-Term Assets PART 1 UNIT 1

LOS 1A2ff 7 Asset Differences Between U.S. GAAP and IFRS

The following tables summarize differences in the treatment of assets for financial reporting
purposes between U.S. GAAP and IFRS.

7.1 IFRS Differences: Intangible Assets


Subject U.S. GAAP Requirements IFRS Requirements
Development All development costs are expensed Internal development expenditures
costs immediately. are capitalized when technical and
economic feasibility of a project can
There is one exception that is
be demonstrated. This requirement
related to the development cost of
applies to all intangible assets
computer software developed for
internally developed.
external use, which has separate
standards requiring capitalization of
development cost from the date the
project is proved to be feasible up to
the point when the first master copy
of the computer software is ready.
Revaluation Not permitted. Revaluation of intangibles (other than
goodwill) is permitted. This revaluation
is permitted if a class of intangibles is
revalued and only if there is an active
market for those intangibles.

7.2 IFRS Differences: Long-Lived Assets and Revaluation


Subject U.S. GAAP Requirements IFRS Requirements
Revaluation Not permitted. Permitted for a class of long-lived
assets.
Component Permitted, not required. Required.
depreciation
Revision of Done if circumstances arise that Done at least every year at the time of
depreciation cause management to believe the preparation of the annual reports.
methods, that the current estimates and/or
residual values, methods are no longer appropriate.
or estimated life
Capitalization of 1. Only interest paid during 1. Interest and all other ancillary
borrowing cost construction can be capitalized. costs and changes in foreign
exchange rates that are regarded
2. Do not offset interest income.
as adjustments to interest are
3. Average interest on average eligible to be capitalized.
accumulated expenditures is
2. Interest expense is offset
capitalized.
against interest income.
3. Actual borrowing cost is
capitalized.

(continued)

1–126 Module 6 A.2. Investments


© Becker Professional Education Corporation. and Long-Term
All rights reserved.Assets
PART 1 UNIT
6 1 A.2. Investments and Long-Term Assets

(continued)

Impairment Two steps: One step approach: Compare the


carrying value of the asset with the
1. compare the carrying value of
greater of:
the asset to the estimated future
undiscounted cash flows (FCF), 1. the value in use; or
if the FCF exceeds the carrying
2. the fair value minus costs to sell.
value, then no impairment is
recorded; otherwise
2. record the impairment loss
that is equal to the difference
between the fair value and the
carrying value.
Reversal of Recovery of previously recorded Since the revaluation system is
impairment loss impairment loss is permitted up allowed, revaluation losses can be
to the previously recorded losses. reversed. Revaluation gains (called
However, this recovery is permitted revaluation surplus) are reported
only if the asset is held for disposal. as part of comprehensive income
Recovery is not permitted for assets unless they reverse a previously
held for use (for example, inventory). recorded impairment loss, in which
case those revaluation gains are
reported as part of income.

Question 1 MCQ-12392

Harris Co. purchased several new investments during its first year of business. Information
related to each investment is presented below.

Fair Value Fair Value


Available-for-sale securities Cost Dec. 31, Year 1 Dec. 31, Year 2
XYZ bonds $217,000 $215,000 $225,000
BGSE bonds $ 65,000 $ 70,000 $ 75,000
Trading securities
ESGB bonds $112,000 $105,000 $107,000
Dawson investments $ 97,000 $ 93,000 $118,000

Amortized Cost Amortized Cost Fair Value Fair Value


Held-to-maturity securities Dec. 31, Year 1 Dec. 31, Year 2 Dec. 31, Year 1 Dec. 31, Year 2
BK bonds $425,000 $415,000 $432,000 $427,000
GHHKG bonds $318,000 $314,000 $305,000 $303,000

What amount will be reported as total investments in bonds on the balance sheet at
Dec. 31, Year 2?
a. $1,237,000
b. $1,255,000
c. $1,220,000
d. $1,254,000

© Becker Professional Education Corporation. All rights reserved. Module 6 1–127 A.2. Inves
6 A.2. Investments and Long-Term Assets PART 1 UNIT 1

Question 2 MCQ-12394

The following securities were purchased by Dawson Enterprises during Year 1 of


operations. The investments were appropriately classified as trading securities. The Year 1
income statement reflected a net unrealized holding gain of $18,500. The following data
related to the investments for Year 2 are indicated below.

Fair Value Cost


GWD investments $75,000 $67,000
HCD investments $79,000 $55,000
What is the amount of the unrealized holding gain/(loss) reported on the income statement
for Year 2?
a. $32,000
b. $13,500
c. $18,500
d. No unrealized gain or loss reported on the income statement.

Question 3 MCQ-12396

Karrington Corp. purchased a delivery van that cost $40,000 with an expected service life of 8
years and an estimated residual value of $4,000 on January 1, Year 1. What would be the book
value of the van at the end of the second year of the asset's life using straight-line depreciation?
a. $40,000
b. $31,000
c. $35,500
d. $31,500

Question 4 MCQ-12397

Impairments of tangible/intangible assets with a finite life occur when events indicate a
change in the total anticipated benefits associated with ownership of assets. Which of the
following events listed below indicates the book value of an asset may not be recoverable?
a. A significant change in the market value of the asset.
b. Significant adverse changes in the legal environment.
c. A change in the estimated utilization of the asset prior to the end of the asset's
useful life.
d. All of the above are events indicating the book value of the asset may not be
recoverable.

1–128 Module 6 A.2. Investments


© Becker Professional Education Corporation. and Long-Term
All rights reserved.Assets
7
MODULE
PART 1 UNIT 1

A.2. Liabilities and Equity


Part 1
Unit 1

This module covers the following content from the IMA Learning Outcome Statements.

CMA LOS Reference: Part 1—Section A.2. Liabilities and Equity

The candidate should be able to:


o. identify the classification issues of short-term debt expected to be refinanced
p. compare the effect on financial statements when using either the assurance warranty
approach or the service warranty approach for accounting for warranties
v. identify transactions that affect paid-in capital and those that affect retained earnings
w. determine the effect on shareholders' equity of large and small stock dividends, and
stock splits

1 Liabilities LOS 1A2o

Liabilities are probable future sacrifices of economic benefits arising from present obligations of
an entity to transfer assets or provide services to other entities in the future as a result of past
transactions or events.
Liabilities must be identified as current or non-current for financial reporting purposes. Current
liabilities are obligations whose liquidation is reasonably expected to require the use of current
assets, the creation of other current liabilities, or the provision of services within the next year or
operating cycle, whichever is longer.
Regular business operations can result in current liabilities as can bank borrowings to meet the
cash needs of the entity. Current liabilities are valued on the balance sheet at their settlement
value. Current liabilities are an important indication of financial strength and solvency. The
ability to pay current debts as they mature is analyzed by interested parties both within and
outside the company.

1.1 Trade Accounts Payable


Trade accounts payable are amounts owed for goods, raw materials, and supplies that are
not evidenced by a promissory note. Purchases of goods and services on credit are usually
determinable as to amounts due and the due date. Cash discounts associated with accounts
payable can be anticipated and journalized. The purchase may be recorded gross or net.
Gross Method
The gross method records the purchase without regard to the discount. If invoices are paid
within the discount period, a purchase discount is credited.

© Becker Professional Education Corporation. All rights reserved. Module 7 1–129


7 A.2. Liabilities and Equity PART 1 UNIT 1

Net Method
Under the net method, purchases and accounts payable are recorded net of the discount.
If payment is made within the discount period, no adjustment is necessary. If payment is
made after the discount period, a purchase discount lost account is debited.

1.2 Current Portions of Long-Term Debt


Debt instruments may be set up such that periodic principal payments are made during the life
of the borrowing. In this case the principal due within the next year (or operating cycle) will be
classified as a current liability.

1.2.1 Current Obligations Expected to Be Refinanced


Under U.S. GAAP, a short-term obligation may be excluded from current liabilities and included
in non-current debt if the company intends to refinance it on a long-term basis and the intent is
supported by the ability to do so as evidenced either by:
the actual refinancing prior to the issuance of the financial statements; or
the existence of a noncancelable financing agreement from a lender having the financial
resources to accomplish the refinancing.
The amount excluded from current liabilities and a full description of the financing agreement
shall be fully disclosed in the financial statements or notes.
The following journal entry would be used to record the reclassification:

DR Short-term liability $XXX


CR Long-term liability $XXX

Example 1 Classification of Debt

Facts: Timber Corp. has $5,000,000 of long-term debt that will mature on April 1, Year 2.
On December 1, Year 1, based on its expected available cash, Timber decides to refinance
$4,000,000 of the debt. The bank issues Timber a six-year, 12 percent note on December
15, Year 1. The entire proceeds of this loan will be used on April 1, Year 2, to pay the
long‑term debt due on that date. Timber prepares financial statements on December 31.
Required: Describe the classification of the $5,000,000 long-term debt, maturing April 1,
Year 2, on Timber Corp.'s balance sheet at December 31, Year 1.
Solution: Only $1,000,000 of the long-term debt is classified as a current liability; the
remaining $4,000,000 is reclassified as long-term because by the end of Year 1, Timber had
initiated, and in this case completed, actual refinancing using long-term debt.

1–130 Module 7 A.2.AllLiabilities


© Becker Professional Education Corporation. and Equity
rights reserved.
PART 1 UNIT
7 1 A.2. Liabilities and Equity

2 Warranties LOS 1A2n


1A2p

Warranties are a seller's promise to "correct" any product defects or to compensate the buyer
for any issues. Sellers offering warranties create a liability account at the time of sale if the
cost of the warranty can be reasonably estimated. The accounting for warranties depends on
whether the warranty represents an assurance warranty or a service warranty.

2.1 Assurance Warranty Approach


An assurance warranty provides the customer a guarantee that the product/service will
work properly for the period covered. An assurance warranty does not represent a separate
performance obligation; therefore, no sales revenue is recorded related to assurance
warranties. If the product/service does not work properly within this time frame, the seller will
correct the situation by repairing the product, substituting a new product/service, or reimbursing
the customer. The liability and the related expense are recognized in the year of sale to match
the cost with the corresponding revenue from selling the product. The accrual is recorded even
if a portion, or all, of the warranty expenditure will be incurred in later years. The classification
of the liability as current or long-term depends on the timing of the expected costs related to
satisfying the warranty. As repair work is completed, the liability is reduced.

Example 2 Warranties

Facts: ABC Corp. has a three-year warranty against defects in the machinery it sells.
When a warranty claim is made, ABC Corp. satisfies the claim by replacing the machinery.
Warranty costs are estimated at 2 percent of sales in the year of sale, and 4 and 6 percent
in the succeeding years. ABC sales and actual warranty expenses for Year 1–Year 3 were
as follows:

Actual
Sales Warranty Costs
Year 1 $ 250,000 $10,000
Year 2 500,000 20,000
Year 3 750,000 30,000
$1,500,000 $60,000

Required: Prepare the journal entries to account for the warranty in Years 1-3 and
determine the balance in the warranty liability account at the end of Year 3.

(continued)

© Becker Professional Education Corporation. All rights reserved. Module 7 1–131


7 A.2. Liabilities and Equity PART 1 UNIT 1

(continued)

Solution: ABC's total liability should be accrued in the year of sale even though it will not be
incurred in that year.
The following journal entries will be recorded in Years 1–3.
Year 1:
DR Warranty expense ($250,000 × 12%) $30,000
CR Warranty liability $30,000
DR Warranty liability (actual costs) 10,000
CR Inventory 10,000

Year 2:
DR Warranty expense ($500,000 × 12%) $60,000
CR Warranty liability $60,000
DR Warranty liability 20,000
CR Inventory 20,000

Year 3:
DR Warranty expense ($750,000 × 12%) $90,000
CR Warranty liability $90,000
DR Warranty liability 30,000
CR Inventory 30,000

The balance in the account at the end of Year 3 is total liability less actual expenditures and
is calculated as follows:
Total liability = Sales × Total estimated expense
= $1,500,000 × 12% [2% + 4% + 6%]
= $180,000
Balance, liability account, 12/31/Year 3 = Total liability − Actual expenditures
= $180,000 − $60,000
= $120,000

2.2 Service Warranty Approach


Service warranties, sometimes referred to as extended warranties, represent a separate
performance obligation, and therefore result in the recognition of revenue. Service warranties
require the recognition of a liability for the warranty obligation and determination of when to
recognize revenue from the sale of the warranty.
Extended warranties are often offered by sellers of products such as electronic devices, automobiles,
and household appliances. Because the warranty is viewed as a separate performance obligation,
the revenue from sales of a service warranty is recorded as unearned revenue, a liability. Unearned
revenue is recognized as earned revenue as time passes, usually on a straight-line basis. The
unearned revenue is classified as current or long-term based on the time period covered. Costs
incurred related to the warranty are expensed in the period in which they occur.

1–132 Module 7 A.2.AllLiabilities


© Becker Professional Education Corporation. and Equity
rights reserved.
PART 1 UNIT
7 1 A.2. Liabilities and Equity

Example 3 Service Warranty

Facts: Evelyn Electronics offers an extended three-year warranty on all blu-ray players the
company sells. At the beginning of Year 1, the company sold, for cash, a total of $6,000 in
extended warranties. The company spent $250 related to these warranties in Year 1.
Required: Prepare the service warranty journal entries for Year 1.
Solution: Each year $2,000 ($6,000 ÷ 3 years) of unearned revenue is converted to earned
revenue; the cost of the repair work each will be recorded as an expense for that year.
January 1, Year 1: Journal entry to record extended warranty sales at the beginning of the year
that create a performance obligation over the next three years:

DR Cash $6,000
CR Unearned revenue—extended warranty $6,000

December 31, Year 1: Journal entry to recognize warranty revenue earned during the first year of
coverage, $6,000 ÷ 3 years = $2,000:

DR Unearned revenue—extended warranty $2,000


CR Revenue—extended warranty $2,000

During Year 1: Journal entry to recognize warranty costs to satisfy customer claims in the period
incurred and match against current period warranty revenues:

DR Warranty expense $250


CR Cash $250

3 Stockholders' Equity

Stockholders' equity (also called shareholders' equity) is the owners' claim to the net assets
(i.e., assets minus liabilities) of a corporation. Stockholders' equity is generally presented on the
statement of financial position (balance sheet) as the last major section, following liabilities.

3.1 Capital Stock (Legal Capital)


Legal capital is the amount of capital that must be retained by the corporation for the protection
of creditors. Legal capital is equal to the par value of the stock issued by the corporation. The par
value or stated value of stock is stated in the corporate charter at the time of the stock's original
issuance. The par value has very little relation to the stock's market price at either the time of
original issuance or at any time thereafter. No-par common stock may be issued as either true
no-par stock or no-par stock with a stated value.
A corporation's charter contains the amounts of each class of stock that the corporation may
legally issue. This amount is referred to as "authorized" capital stock. When a company issues
either all or a portion of the authorized capital stock, this stock is referred to as "issued."
Because a corporation may reacquire shares of its own stock in the form of treasury stock, the
amount of stock issued and in the hands of shareholders is referred to as "outstanding."

© Becker Professional Education Corporation. All rights reserved. Module 7 1–133


7 A.2. Liabilities and Equity PART 1 UNIT 1

Stock is divided into two classes, common and preferred. Corporations may raise funds by
selling common stock, or preferred stock, or both. This source of capital, representing amounts
received from stockholders, is referred to as contributed capital.

3.2 Common Stock


Common stock is the basic ownership interest in a corporation. Common shareholders bear the
ultimate risk of loss and receive the ultimate benefits of success, but they are not guaranteed
dividends or assets upon dissolution. Common shareholders generally control management.
They have the right to vote, the right to share in earnings of the corporation, and the right
to share in assets upon liquidation after the claims of creditors and the claims of preferred
shareholders are satisfied.
If granted in the articles of incorporation, common stockholders may also have preemptive
rights to a proportionate share of any additional stock issued by the corporation. This right gives
common stockholders the first right to purchase additional shares in order to maintain current
levels of ownership when new shares are issued in the market.

3.3 Preferred Stock


Preferred stock is an equity security with preferences and features not associated with common
stock. Preferred stock may include a preference relating to dividends, which may be cumulative
or noncumulative and participating or nonparticipating. Preferred stock may also include a
preference relating to liquidation. Usually, preferred stock does not have voting rights.

LOS 1A2v 3.4 Additional Paid-in Capital


Additional paid-in capital is generally contributed capital in excess of par value or stated value.
It can also arise from many different types of transactions. Additional paid-in capital may be
aggregated and shown as one amount on the balance sheet.
Examples of transactions involving additional paid-in capital include the following:
Issuance of capital stock in amounts in excess of par or stated value
Sale of treasury stock at a gain
Liquidating dividends
Conversion of bonds
Small stock dividends

3.5 Issuance of Stock for Cash


When common stock and/or preferred stock is originally issued or sold to investors, the par
value of the stock is recorded in the appropriate stock account. Any amount the corporation
receives in excess of the par value or stated value of the stock is accounted for as additional
paid-in capital. This account is used to record "gains" on the sale of the company's own stock.

1–134 Module 7 A.2.AllLiabilities


© Becker Professional Education Corporation. and Equity
rights reserved.
PART 1 UNIT
7 1 A.2. Liabilities and Equity

Example 4 Basic Stock Transactions

Facts: Bear Corp. was organized at the beginning of Year 1. The company's common
stock has a par value of $5 and the company's preferred stock has a par value of $20. The
company had the following transactions during its first month of operations:
——January 1: Sold 10,000 shares of common stock and 5,000 shares of preferred stock for
$150,000 in total. Both stocks were sold at par value.
——January 10: Sold 3,000 shares of common stock for $40,000.
——January 15: Sold 1,000 shares of preferred stock for $25,000.
Required: Calculate the effect of these transactions on the balance sheet accounts on
January 1, 10, and 15 of Year 1.
Solution:
January 1:
——Cash increases by $150,000.
——Common stock increases by $50,000: 10,000 shares × $5 par value per share.
——Preferred stock increases by $100,000: 5,000 shares × $20 par value per share.
——No additional paid-in capital is recorded because the stocks were sold at par value.
The January 1 journal entry:

DR Cash $150,000
CR Common stock, $5 par $50,000
CR Preferred stock, $20 par 100,000

January 10:
——Cash increases by $40,000.
——Common stock (CS) increases by $15,000: 3,000 shares × $5 par value per share
——Additional paid-in capital—common stock increases by $25,000: $40,000 cash
received – $15,000 total par value.
The January 10 journal entry:

DR Cash $40,000
CR Common stock, $5 par $15,000
CR Additional paid-in capital—CS 25,000

(continued)

© Becker Professional Education Corporation. All rights reserved. Module 7 1–135


7 A.2. Liabilities and Equity PART 1 UNIT 1

(continued)

January 15:
——Cash increases by $25,000.
——Preferred stock (PS) increases by $20,000: 1,000 shares × $20 par value per share.
——Additional paid-in capital—preferred stock increases by $5,000:
$25,000 cash received – $20,000 total par value.
The January 15 journal entry:

DR Cash $25,000
CR Preferred stock, $20 par $20,000
CR Additional paid-in capital—PS 5,000

Pass Key

The sale of stock affects the balance sheet only. Owner transactions never affect income.

3.6 Issuance of Stock for Assets


Sometimes stock, instead of cash, is used to acquire assets such as land or machinery. In
such situations, the asset is recorded and the stock issued, and the related additional paid-
in capital, if any, is recorded in a combined amount equal to the fair value of the asset the
corporation received.

Example 5 Sale of Stock for Other Than Cash

Facts: Assume that on February 1, Year 1, Parker Corp. acquired land with a fair value of
$150,000 by issuing 10,000 shares of $5 par value common stock.
Required: What effect will this transaction have on the financial statements?
Solution: This transaction increases land by $150,000, increases common stock by $50,000
(10,000 shares × $5 par value per share), and increases additional paid-in capital—common
stock by $100,000 ($150,000 fair value of land – $50,000 total par value).
The February 1 journal entry:

DR Land $150,000
CR Common stock, $5 par $50,000
CR Additional paid-in capital—CS 100,000

1–136 Module 7 A.2.AllLiabilities


© Becker Professional Education Corporation. and Equity
rights reserved.
PART 1 UNIT
7 1 A.2. Liabilities and Equity

3.7 Treasury Stock


Treasury stock is a corporation's own stock that has been issued to shareholders and
subsequently reacquired (but not retired). Treasury stockholders are not entitled to any of
the rights of ownership given to common shareholders, such as the right to vote or to receive
dividends. In addition, a portion of retained earnings equal to the cost of treasury stock may
be restricted and may not be used as a basis for the declaration or payment of dividends
(depending on applicable state law). The cost method is the most common method used to
account for treasury stock transactions.

3.7.1 Cost Method


Under the cost method the treasury shares are recorded and carried at their reacquisition cost.
A gain or loss will be determined only when the treasury stock is resold. The account "additional
paid-in capital from treasury stock" is increased for gains and decreased for losses (if there is
an existing balance in the account) when treasury stock is reissued at prices that differ from
the reacquisition cost. Losses may decrease retained earnings if the additional paid-in capital
from treasury stock account does not have a balance large enough to absorb the loss. Retained
earnings will never be increased through treasury stock transactions, and net income will never
be increased or decreased through treasury stock transactions.

Illustration 1 Cost Method

Original Issue:
10,000 shares $10 par value common stock sold for $15 per share.

DR Cash $150,000
CR Common stock (10,000 × $10 par) $100,000
CR Additional paid-in capital—C/S 50,000

Buy Back Above Issue Price:


200 shares were repurchased for $20 per share.

DR Treasury stock (200 × $20) $4,000


CR Cash $4,000

Reissue Above Cost:


100 shares repurchased for $20 were resold for $22.

DR Cash (100 × $22) $2,200


CR Treasury stock (100 × $20) $2,000
CR Additional paid-in capital—T/S 200

(continued)

© Becker Professional Education Corporation. All rights reserved. Module 7 1–137


7 A.2. Liabilities and Equity PART 1 UNIT 1

(continued)

The following journal entry was made after the preceding entry:
Reissue Below Cost:
100 shares repurchased for $20 were resold for $13.

DR Cash (100 × $13) $1,300


DR Additional paid-in capital—T/S 200
DR Retained earnings 500
CR Treasury stock (100 × $20) $2,000

3.8 Retained Earnings


Retained earnings (or deficit) is accumulated earnings (or losses) during the life of the corporation
that have not been paid out as dividends The amount of accumulated retained earnings is reduced
by distributions to stockholders and transfers to additional paid-in capital for stock dividends.
Retained earnings does not include treasury stock or accumulated other comprehensive income
If the retained earnings account has a negative balance, it is called a deficit.

Net income/loss

− Dividends (cash, property, and stock) declared

± Prior period adjustments

± Accounting changes reported retrospectively

Retained earnings

3.9 Dividends
A dividend is a pro rata distribution by a corporation based on the shares of a particular class of
stock and usually represents a distribution of earnings. Cash dividends are the most common
type of dividend distribution, although there are many other types. Preferred stock usually
pays a fixed dividend, expressed either in dollars or as a percentage based on the par value of
preferred stock outstanding. Common stock receives the amount of the total dividend declared
after reduction for the preferred stock dividend.

3.9.1 Important Dividend Dates


Date of Declaration: the date the board of directors formally approves a dividend. On the
declaration date a dividends payable liability is created and retained earnings are reduced.
Date of Record: the date the board of directors specifies that those who own the stock on
this date will receive the dividend.
Date of Payment: the date on which the corporation pays (paper check or electronic check)
the dividend.

1–138 Module 7 A.2.AllLiabilities


© Becker Professional Education Corporation. and Equity
rights reserved.
PART 1 UNIT
7 1 A.2. Liabilities and Equity

3.9.2 Cash Dividends


Cash dividends paid by a company result in cash to shareholders. Dividends may be declared
on preferred stock or common stock but are not declared on treasury stock. Dividends declared
reduce retained earnings.

Example 6 Dividends on Preferred and Common Stock

Facts: Eagle Corp. has the following stock information available during Year 1:
——10% preferred stock, $25 par value, 10,000 shares outstanding
——Common stock, $5 par value, 20,000 shares outstanding
On October 1, Year 1, Eagle's board of directors declared a cash dividend of $40,000.
The dividend was paid on October 31, Year 1. No stock transactions occurred during Year 2.
On November 1, Year 2, Eagle's board of directors declared a $15,000 cash dividend.
The dividend was paid on November 30, Year 2.
Required: Calculate the dividends each class of shareholders will receive on each
payment date.
Solution: Preferred stockholders are paid first. The amount they will receive is $25,000:
10% stated rate × $25 par value per share × 10,000 shares outstanding.
Year 1: Preferred stockholders receive $25,000 and common stockholders receive $15,000:
$40,000 dividend declared – $25,000 dividend paid to preferred stockholders.
Declaration Date: October 1, Year 1, journal entry:

DR Retained earnings $40,000


CR Dividend payable $40,000

Payment Date: October 31, Year 1:

DR Dividend payable $40,000


CR Cash $40,000

Year 2: Preferred stockholders receive $15,000. Common stockholders do not receive any
dividends because preferred stockholders are entitled to all dividends up to $25,000 each
time a dividend is declared by the board of directors.
Declaration Date: November 1, Year 2, journal entry:

DR Retained earnings $15,000


CR Dividend payable $15,000

Payment Date: November 30, Year 2:

DR Dividends payable $15,000


CR Cash $15,000

© Becker Professional Education Corporation. All rights reserved. Module 7 1–139


7 A.2. Liabilities and Equity PART 1 UNIT 1

3.9.3 Property Dividends


Property dividends distribute noncash assets (e.g., inventory, investment securities, etc.) to
shareholders. On the date of declaration, the property to be distributed is restated to fair value,
and any gain or loss is recognized in income. The dividend liability and the related decrease to
retained earnings are recorded at the fair value of the assets to be transferred.

LOS 1A2w 3.9.4 Treatment of a Small Stock Dividend (< 20–25%)


Stock dividends distribute additional shares of a company's own stock to its shareholders.
When less than 20 to 25 percent of the shares previously outstanding are declared as a stock
dividend, the dividend is treated as a small stock dividend because the issuance is not expected
to affect the market price of the stock. The fair market value of the stock dividend at the date of
declaration is transferred from retained earnings to capital stock and additional paid-in capital.
There is no effect on total shareholders' equity, as paid-in capital is substituted for retained
earnings (i.e., retained earnings are "capitalized" and made part of paid-in capital).

Example 7 Small Stock Dividend

Facts: Capital Corporation has 100,000 shares of $10 par value common stock outstanding.
The company declares a stock dividend of 5,000 shares when the fair market value is $15
(on the date of declaration). 5,000 shares/100,000 shares = 5%, which is considered a small
stock dividend.
Required: Prepare the journal entry to record the dividend.
Solution:
Journal entry:

DR Retained earnings (5,000 × $15 FV) $75,000


CR Common stock (5,000 × $10 par value) $50,000
CR Paid-in capital (difference = $75,000 − 50,000) 25,000

3.9.5 Treatment of a Large Stock Dividend (> 20–25 Percent)


When more than 20 to 25 percent of the previously issued shares outstanding are distributed,
the dividend is treated as a large stock dividend, as it may be expected to reduce the market
price of the stock (similar to a stock split). The par (or stated) value of the stock dividend is
normally transferred from retained earnings to capital stock to meet legal requirements. The
amount transferred is the number of shares issued multiplied by the par (or stated) value of
the stock. However, if state law does not require capitalization of retained earnings for stock
dividends (which is rare because it requires amendment to the articles of incorporation), record
the stock dividend distribution (like a stock split) by changing the number of shares outstanding
and the par (or stated) value per share.

1–140 Module 7 A.2.AllLiabilities


© Becker Professional Education Corporation. and Equity
rights reserved.
PART 1 UNIT
7 1 A.2. Liabilities and Equity

Stock Dividend Greater Than 20–25 Percent


Example 8
of Previous Outstanding Shares

Facts: LMT Corp. declares a 40 percent stock dividend on its 1,000,000 shares of
outstanding $10 par common stock (5,000,000 authorized). On the date of declaration, LMT
stock is selling for $20 per share.

Total stock dividend (0.40 × 1,000,000) 400,000 shares


Value of 400,000 shares @ $10 per share (par) $4,000,000

Required: Prepare the journal entries to record the declaration and distribution of the
stock dividend.
Solution:
Journal entry to record the declaration of the stock dividend at par:

DR Retained earnings $4,000,000


CR Common stock distributable $4,000,000

Journal entry to record the distribution of the stock dividend at par:

DR Common stock distributable $4,000,000


CR Capital stock, $10 par common $4,000,000

3.10 Stock Splits


Stock splits occur when a corporation issues additional shares of its own stock (without charge)
to current shareholders and reduces the par (or stated) value per share proportionately.
There is no change in the total book value of the shares outstanding. Thus, the memo entry to
acknowledge a stock split is merely a formality.
A stock split usually does not affect retained earnings or total shareholders' equity, as is
exhibited below:

Before the Split


Common stock (10,000 shares outstanding @ $10 par) $100,000
After the Split (× 2) (÷ 2)
Common stock (20,000 shares outstanding @ $5 par) $100,000

© Becker Professional Education Corporation. All rights reserved. Module 7 1–141


7 A.2. Liabilities and Equity PART 1 UNIT 1

3.10.1 Reverse Stock Splits


A reverse stock split would involve reducing the number of shares outstanding and increasing
the par (or stated) value proportionately. One way to reduce the amount of outstanding shares
is to recall outstanding stock certificates and issue new certificates.

3.10.2 Stock Splits on Treasury Stock


Stock splits are usually not applied to treasury stock because such stock is not considered
outstanding. However, an exception is made when:
the company is maintaining a ratio of treasury shares to shares outstanding in order to
meet stock option or other contractual commitments; or
state law requires that treasury stock be protected from dilution.

Example 9 Effect of Stock Splits on Outstanding Shares and Equity

Facts: Bard Corp. has 25,000 shares of common stock outstanding with a $9 par value.
Additional paid-in capital—common stock is $125,000, and retained earnings are $195,000.
Required: For each of the following independent situations determine the number of
shares issued and the effect each transaction has on equity.
——Situation 1: Declares and distributes a 60 percent stock dividend when the stock is
selling for $30 per share.
——Situation 2: Declares a 3-for-1 stock split when the market price of the stock is $60
per share.
——Situation 3: Declares a 5 percent stock dividend when the market price of the stock is
$15 per share.
Solution:
——Situation 1 (60 percent stock dividend):
The stock dividend equals 15,000 shares: 25,000 shares × 60% dividend. The dividend is
categorized as a large stock dividend, and the par value of the stock is used to reclassify
the amount from retained earnings into the capital stock account. The stock's selling
price at the time of the stock dividend is irrelevant.
Retained earnings decrease and common stock increases by $135,000:
15,000 shares × $9 par value per share.
Journal entry:

DR Retained earnings $135,000


CR Common stock $135,000

(continued)

1–142 Module 7 A.2.AllLiabilities


© Becker Professional Education Corporation. and Equity
rights reserved.
PART 1 UNIT
7 1 A.2. Liabilities and Equity

(continued)

——Situation 2 (3-for-1 stock split):


The stock split leads to an additional 50,000 shares for a new total of 75,000 shares
outstanding: 25,000 shares previously outstanding × 3 multiplier. There is no effect on
the accounts. The number of shares triples, and the new par value is $3: $9 previous
par value per share ÷ 3. The stock's selling price at the time of the stock split is irrelevant.
No journal entry is needed as this scenario results in a memorandum entry to update
shares outstanding and par value.
——Situation 3 (5% stock dividend):
The stock dividend equals 1,250 shares: 25,000 shares outstanding immediately before
the stock dividend is declared × 5% multiplier. This dividend is a small stock dividend. The
stock's selling price at the date of the declaration of the small stock dividend is relevant.
Retained earnings decrease by $18,750: 1,250 shares × $15 market value per share on
the date of the declaration of the stock dividend. Common stock increases by $11,250:
1,250 shares × $9 par value per share. Additional paid-in capital—common stock
increases by $7,500: $18,750 total value of the small stock dividend – $11,250 increase in
common stock account.
Journal entry:

DR Retained earnings $18,750


CR Common stock $11,250
CR Additional paid-in capital 7,500

Question 1 MCQ-12403

Emmons Industries manufactures and sells a pipe fitting used by oil and gas companies.
Emmons provides a three-year warranty against manufacturer defects. Industry experience
with similar products indicates warranty costs are approximately 2 percent of sales. Sales of
the pipe fitting in Year 1 were $2 million and actual warranty expenditures were $15,000 for
the first year. What amount will Emmons report as a warranty liability at the end of the year?
a. $40,000
b. $25,000
c. $15,000
d. $0

© Becker Professional Education Corporation. All rights reserved. Module 7 1–143


7 A.2. Liabilities and Equity PART 1 UNIT 1

Question 2 MCQ-12406

On September 26, Year 2, Midge Enterprises repurchased for $32 per share 100,000
shares of its 1,000,000 shares of common stock outstanding that have a $5 par value.
On November 15 of Year 2, 75,000 of the shares were reissued for $35 per share. On
January 15 of Year 3, 10,000 shares were reissued for $30 per share. If the cost method of
accounting for treasury stock is used, what is the balance in the paid-in capital treasury
stock account in Year 3?
a. $245,000
b. $225,000
c. $205,000
d. $20,000

Question 3 MCQ-12408

England Enterprises' shareholders' equity on December 31, Year 1, includes the following:

Common stock, $3 par, authorized 10 million shares; $22,500,000


issued and outstanding, 7.5 million shares
Add'l paid-in capital: CS 90,000,000
Retained earnings 78,750,000
On June 30 of Year 2, the board of directors declared a 5 percent stock dividend on
common shares to be distributed on July 15. The market price of the stock on June 30 was
$25, and on July 15 the market value was $30. Determine the value of the stock dividend.
a. $1,125,000
b. $9,375,000
c. $12,500,000
d. $13,125,000

1–144 Module 7 A.2.AllLiabilities


© Becker Professional Education Corporation. and Equity
rights reserved.
A.2. Special Valuation

8
MODULE
PART 1 UNIT 1

Considerations:
Taxes and Leases
Part 1
Unit 1

This module covers the following content from the IMA Learning Outcome Statements.

CMA LOS Reference: Part 1—Section A.2. Special Valuation Considerations:


Taxes and Leases

The candidate should be able to:


q. demonstrate an understanding of interperiod tax allocation/deferred income taxes
r. distinguish between deferred tax liabilities and deferred tax assets
s. differentiate between temporary differences and permanent differences and identify
examples of each
t. distinguish between operating and finance leases
u. recognize the correct financial statement presentation of operating and finance leases
ff. identify and describe the following differences between U.S. GAAP and IFRS: (iv) leases,
with respect to lessee operating and finance leases

1 Accounting for Income Taxes

Accounting for income taxes involves both intraperiod and interperiod tax allocation. Intraperiod
allocation matches a portion of the provision for income tax to the applicable components of net
income and retained earnings. Interperiod allocation is accounting for the temporary differences
between financial accounting frameworks such as GAAP or IFRS, and governing tax policies.

1.1 Comprehensive Interperiod Tax Allocation LOS 1A2q

Income Tax Return vs. Financial Statements

IRS FASB
TAX Differences GAAP
CODE F/S

© Becker Professional Education Corporation. All rights reserved. Module 8 A.2.


1–145Special Valuation Con
8 A.2. Special Valuation Considerations: Taxes and Leases PART 1 UNIT 1

1.1.1 Objective
The objective of interperiod tax allocation is to recognize through the matching principle the
amount of current and future tax related to events that have been recognized in financial
accounting income.
Current Year Taxes: Payable (liability) or refundable (asset)
Or:
Future Year Taxes: Deferred tax asset or deferred tax liability

1.1.2 Comprehensive Allocation


The asset and liability method (sometimes referred to as the balance sheet approach) is
required by GAAP for comprehensive allocation. Under comprehensive allocation, interperiod
tax allocation is applied to all temporary differences. The asset and liability method requires
that either income taxes payable or a deferred tax liability (asset) be recorded for all tax
consequences of the current period.

1.1.3 Accounting for Interperiod Tax Allocation


Total income tax expense (GAAP income tax expense) or benefit for the year is the sum of:
— current income tax expense/benefit, and
— deferred income tax expense/benefit.
Current income tax expense/benefit is equal to the income taxes payable or refundable
for the current year, as determined on the corporate tax return (Form 1120) for the
current year.
Deferred income tax expense/benefit is equal to the change in deferred tax liability or asset
account on the balance sheet from the beginning of the current year to the end of the
current year (called the "balance sheet approach").
Thus, total income tax expense/benefit can be depicted as follows:

Change in the deferred


Current income tax payable income tax asset or liability Total income tax
or refundable as determined ± =
from the beginning to the expense or benefit
on the corporate tax return end of the reporting period

Temporary Difference Financial


Tax Return
Statement

× Current tax rate × Future (enacted) tax rate


+ Deferred liability
Current liability – Deferred asset = Total tax expense

1–146 Module 8 A.2.


© Becker Special Valuation
Professional Education Considerations: Taxes
Corporation. All rights and Leases
reserved.
PART 1 UNIT
8 1 A.2. Special Valuation Considerations: Taxes and Leases

Pass Key

Total tax expense for financial statements is the combination of current tax plus or minus
deferred taxes.

1.2 Permanent Differences LOS 1A2s

A permanent difference is a transaction that affects only income per books or taxable income,
but not both. Income tax expense for a period is calculated only on taxable items. For example,
tax-exempt interest (municipal and state bonds) is included in financial income, but is excluded
in computing income tax expense.
In effect, permanent differences create a discrepancy between taxable income and financial
accounting income that will never reverse.

1.2.1 No Deferred Taxes


Because they do not reverse themselves, no interperiod tax allocation is necessary for
permanent differences. The income tax provision for financial accounting purposes is computed
on the basis of pretax book income adjusted for all permanent differences.

1.2.2 Examples
Permanent differences are either (a) nontaxable, (b) nondeductible, or (c) special tax allowances.
Examples are:
Tax-exempt interest (municipal, state)
Life insurance proceeds on officer's key man policy
Life insurance premiums when corporation is beneficiary
Certain penalties, fines, bribes, kickbacks, etc.
Nondeductible portion of meal and entertainment expense
Dividends-received deduction for corporations
Excess percentage depletion over cost depletion

© Becker Professional Education Corporation. All rights reserved. Module 8 A.2.


1–147Special Valuation Con
8 A.2. Special Valuation Considerations: Taxes and Leases PART 1 UNIT 1

Example 1 Permanent Differences

Facts: ABC Company reported $200,000 of pretax financial income. Included in this income
was $10,000 of life insurance premium expense for policies on which the corporation is the
beneficiary and interest income on municipal bonds of $50,000.
Required: Calculate and record the tax expense for ABC Company, assuming a 21 percent
tax rate.
Solution:

Tax Return Differences Income Statement


Income $160,000 Income $160,000
Municipal interest -0- Permanent Municipal interest 50,000
Life ins. premium -0- Permanent Life ins. premium (10,000)
Taxable income $160,000 Pretax financial income $200,000

× 21% × 21%
$ 33,600 + -0- = $ 33,600

Note that there are no deferred taxes resulting from temporary differences, and that the
income tax expense and the income tax liability are the same.
Journal entry to record income tax expense and income tax liability:

DR Income tax expense $33,600


CR Income tax payable $33,600

1.3 Temporary Differences


Temporary differences are the differences between the tax basis of an asset or liability
and its reported amount in the financial statements that will result in taxable or deductible
amounts in future years when the reported amount of the asset or liability is recovered or
settled, respectively.

1.3.1 Transactions That Cause Temporary Differences


There are four basic causes of temporary differences, which reverse in future periods.
1. Revenues or gains that are included in taxable income, after they have been included in
financial accounting income, which results in a deferred tax liability.
2. Revenues or gains that are included in taxable income, before they are included in financial
accounting income, which results in a deferred tax asset.
3. Expenses or losses deducted from taxable income, after they have been deducted from
financial accounting income, which results in a deferred tax asset.
4. Expenses or losses deducted for taxable income, before they are deducted from financial
accounting purposes, which results in a deferred tax liability.

1–148 Module 8 A.2.


© Becker Special Valuation
Professional Education Considerations: Taxes
Corporation. All rights and Leases
reserved.
PART 1 UNIT
8 1 A.2. Special Valuation Considerations: Taxes and Leases

1 Financial statement 2 Tax return


income first income first
Tax return Financial statement
income later income later

Tax income later → Deferred tax liability Tax income first → Deferred tax asset

1. Installment sales 1. Unearned rent


2. Contractors accounting (% vs. completed) 2. Unearned interest
3. Equity method (undistributed dividends) 3. Unearned royalties

3 Financial statement 4 Tax return


expense first expense first
Tax return Financial statement
expense later expense later

Tax deduct later → Deferred tax asset Tax deduct first → Deferred tax liability

1. Bad debt expense (allowance vs. direct w/o) 1. Depreciation expense


2. Est. liability/warranty expense 2. Amortization of franchise
3. Start-up expenses 3. Prepaid expenses (cash basis for tax)

5. Additional causes of temporary differences are:


— Differences between the financial reporting and tax basis of assets and liabilities arising
in a business combination accounted for as an acquisition.
— Differences in the tax basis of assets due to indexing, whenever the local currency is the
functional currency.

1.4 Deferred Tax Liabilities LOS 1A2r

Deferred tax liabilities are anticipated future tax liabilities derived from situations in which
future taxable income will be greater than future financial accounting income due to temporary
differences. All deferred tax liabilities are recognized on the balance sheet.

© Becker Professional Education Corporation. All rights reserved. Module 8 A.2.


1–149Special Valuation Con
8 A.2. Special Valuation Considerations: Taxes and Leases PART 1 UNIT 1

Example 2 Deferred Tax Liability

Facts: Stone Co. began operations in Year 1 and reported $225,000 in financial income
for the year. Stone Co.'s Year 1 tax depreciation exceeded its book depreciation by
$25,000. Stone's tax rate for Year 1 and years thereafter was 21 percent. In Year 2, book
depreciation exceeded tax depreciation by $25,000. This is a reversal of the temporary
difference between GAAP and tax accounting and results in the reversal of the deferred tax
liability in Year 2.
Required: Prepare the tax journal entries for Year 1 and Year 2.
Solution:

Tax Return Temporary Difference Financial Statement


Taxable income $200,000 $25,000 Pretax financial income $225,000

× 21% × 21%
$ 42,000 + $ 5,250 = $ 47,250

The excess depreciation on the tax return results in a future liability, a financial accounting
expense in future years that will not be deductible in future years because it was deducted
in Year 1. The deferred tax liability reflects the fact that less depreciation will be deducted
on the tax return in future years, compared with the financial statements. This yields a
future taxable income which will be greater than the future financial accounting income.
Journal entry to record the taxes in Year 1:

DR Income tax expense—current $42,000


DR Income tax expense—deferred 5,250
CR Deferred tax liability $ 5,250
CR Income tax payable 42,000

Journal entry to record the Year 2 reversal of the deferred tax liability:

DR Deferred tax liability $5,250


CR Income tax benefit—deferred $5,250

1.5 Deferred Tax Assets


Deferred tax assets arise when the amount of taxes paid in the current period exceeds the
amount of income tax expense in the current period. They are anticipated future benefits
derived from situations in which future taxable income will be less than future financial
accounting income due to temporary differences.

1–150 Module 8 A.2.


© Becker Special Valuation
Professional Education Considerations: Taxes
Corporation. All rights and Leases
reserved.
PART 1 UNIT
8 1 A.2. Special Valuation Considerations: Taxes and Leases

1.5.1 Valuation Allowance (Contra-Account)


If it is more likely than not (a likelihood of more than 50 percent) that part or all of the deferred
tax asset will not be realized, a valuation allowance is recognized. The net deferred tax asset
should equal that portion of the deferred tax asset which, based on available evidence, is more
likely than not to be realized.
Valuation allowances are not permitted under IFRS. Instead, a deferred tax asset is recognized
when it is probable (more likely than not) that sufficient taxable profit will be available against
which the temporary difference can be utilized.

Example 3 Deferred Tax Asset

Facts: Black Co., organized on January 2, Year 1, had pretax accounting income of $500,000
and taxable income of $800,000 for the year ended December 31, Year 1. The enacted tax
rate for all years is 21 percent. The only temporary difference is accrued product warranty
costs, which are expenses to be paid as follows:
Year 2, $100,000; Year 3, $100,000; Year 4, $100,000
Required: Prepare the tax journal entries for Year 1 and Year 2.
Solution:

Tax Return Temporary Difference Financial Statement


Taxable income $800,000 $300,000 Pretax financial income $500,000

× 21% × 21%
$168,000 − $ 63,000 = $105,000

Journal entry to record the Year 1 taxes:

DR Deferred tax asset $ 63,000


DR Income tax expense—current 168,000
CR Income tax payable $168,000
CR Income tax benefit—deferred 63,000

When the company pays the warranty costs of $100,000 in Year 2, the company will take a
$21,000 ($100,000 × 21%) tax deduction related to the warranty costs and will reverse out
the related deferred tax asset.

Journal entry to record reversal of a portion of the deferred tax asset for warranty costs paid and
deducted in Year 2:

DR Income tax expense—deferred $21,000


CR Deferred tax asset $21,000

© Becker Professional Education Corporation. All rights reserved. Module 8 A.2.


1–151Special Valuation Con
8 A.2. Special Valuation Considerations: Taxes and Leases PART 1 UNIT 1

Example 4 Valuation Allowance

Facts: Black expects to have taxable income of $100,000 in Year 2, but no taxable income
after Year 2.
Required: Prepare the journal entry to record the deferred tax asset and valuation
allowance in Year 1.
Solution: The deferred tax asset would be limited to the amount to be realized in Year 2
($21,000 = $100,000 × 21%). A deferred tax asset of $63,000 would be recognized, but a
valuation account of $42,000 would result in a net deferred tax asset of $21,000.
Journal entry:

DR Deferred tax asset $ 63,000


DR Income tax expense—current 168,000
CR Deferred tax asset valuation allowance $ 42,000
CR Income tax benefit—deferred 21,000
CR Income tax payable 168,000

Example 5 Permanent and Temporary Differences

Facts: Foxy Inc.'s financial statement and taxable income for Year 1 follows (income before
the effect of tax-related differences was $140,000):
Financial statement pretax income $115,000
Differences: municipal interest income (12,000)
Penalty expense 7,000
Tax depreciation $40,000
Book depreciation (30,000)
Excess tax depreciation (10,000)
Income tax return $100,000
The enacted tax rate is 21 percent for this year and future years.

Required: Prepare the tax journal entry for Year 1.

(continued)

1–152 Module 8 A.2.


© Becker Special Valuation
Professional Education Considerations: Taxes
Corporation. All rights and Leases
reserved.
PART 1 UNIT
8 1 A.2. Special Valuation Considerations: Taxes and Leases

(continued)

Solution:

Tax Return Temporary Differences Income Statement


Income $140,000 Income $140,000
Municipal interest -0- Permanent Municipal interest 12,000
Penalty -0- Permanent Penalty (7,000)
$140,000 $145,000
Depreciation (40,000) $10,000 Depreciation (30,000)
Taxable income $100,000 Pretax financial income $115,000

× 21% × 21%
$ 21,000 + $ 2,100 = $ 23,100

Journal entry:

DR Income tax expense—current $ 21,000


DR Income tax expense—deferred 2,100
CR Income taxes currently payable $21,000
CR Deferred tax liability 2,100

Pass Key

— Deferred tax liability (DTL) → Future tax accounting income > Future financial
accounting income
— Deferred tax asset (DTA) → Future tax accounting income < Future financial
accounting income

2 Accounting for Leases

Leases are used by public and private entities as a means of gaining access to assets and
reducing their exposure to the full risks of asset ownership. A lease is defined as a contractual
agreement between a lessor who conveys the right to use real or personal property (an asset)
and a lessee who agrees to pay consideration for this right over a specific period of time. In order
for a contract to be a lease or contain a lease, both of the criteria below must be met.
The contract must depend on an identifiable asset in which the lessor does not have a
substantive substitution right.
The contract must convey the right to control the use of the asset over the lease term to
the lessee. The lessee will have the right to obtain substantially all of the economic benefits
from using the asset and have the right to direct its use.

© Becker Professional Education Corporation. All rights reserved. Module 8 A.2.


1–153Special Valuation Con
8 A.2. Special Valuation Considerations: Taxes and Leases PART 1 UNIT 1

Illustration 1 Definition of a Lease

Bentley Corp. has a written agreement in place to allow Riggs Inc. to use scientific
equipment with a book value of $75,000 for the next five years. Bentley has the right to
replace the equipment with a comparable piece of equipment during the term, but Riggs
is able to use the asset as it wishes for the next five years while keeping any cash inflows
associated with outputs from the equipment.
This is an example of a lease, as there is a contract in place that defines the asset itself,
recognizes Bentley's right to substitute the asset, and provides Riggs with the economic
benefits of and direction for the use of the asset.

LOS 1A2t 2.1 Lease Classification as Operating or Finance


Leases transfer substantially all of the benefits and risks inherent in ownership of property to
the lessee.
This is an accounting transaction, which is in substance an installment purchase in the form
of a leasing arrangement.
The lessee accounts for a lease as either an operating or a finance lease, reflective of the
acquisition of both an asset and a related liability.
The lessor accounts for the lease as either an operating lease, sales-type lease, or direct
financing lease.
Based on the lease criteria, a lease will be considered either an operating or a finance lease.
Within the finance lease category, two types, sales-type and direct financing, are applicable to
lessors.
Lessee Lessor

Operating Lease Finance Lease Operating Lease Finance Lease

Direct
Sales-Type Lease
Financing Lease

Pass Key

IFRS uses a single model in which lessees only record finance leases; operating leases are
not applicable for lessees. Lessors can recognize operating and finance leases.

1–154 Module 8 A.2.


© Becker Special Valuation
Professional Education Considerations: Taxes
Corporation. All rights and Leases
reserved.
PART 1 UNIT
8 1 A.2. Special Valuation Considerations: Taxes and Leases

2.1.1 Criteria
At the onset of a lease, both parties (lessee and lessor) must determine whether the lease will
be classified as an operating lease or a finance lease. The assessment, based on a defined set
of criteria, shown below, will focus on whether the lessee will in effect assume control of the
underlying asset.
The criteria below are applicable to lessors and lessees. If any one of the five criteria is met, the
lease will be classified as a sales-type lease by the lessor and a finance lease by the lessee.
Ownership of the underlying asset transfers from the lessor to the lessee by the end of the
lease term.
The lessee has the written option to purchase the underlying asset; the option is one that
the lessee is "reasonably certain" to exercise.
The net present value of all lease payments and any guaranteed residual value is equal to
or substantially exceeds the underlying asset's fair value.
The term of the lease represents the major part of the economic life remaining for the
underlying asset.
The asset is specialized such that it will not have an expected, alternative use to the lessor
when the lease term ends.
If none of the above criteria are met, or if the lease is considered short term (less than 12
months), it should be treated as an operating lease by the lessee. For the lessor, if none of the
criteria above are met, the classification will depend on whether both of the following criteria
are met.
Present value of the sum of the lease payments, lessee guaranteed residual value not
included in the lease payments, and any third-party guaranteed residual value is equal to or
substantially exceeds the underlying asset's fair value.
Collection of the lease payments and any amounts necessary to satisfy residual value
guarantees is probable.
When both of the criteria above are met, the lessor will classify the lease as a direct financing
lease. If only one or neither are met, the lessor will classify the lease as operating.

Pass Key

Criteria for finance lease—remember OWNES


——Ownership
——Written bargain option
——Net present value
——Economic life
——Specialized
Criteria for direct financing lease—must meet both PC
——Present value equal or greater to asset fair value
——Collection of lease payments is probable

© Becker Professional Education Corporation. All rights reserved. Module 8 A.2.


1–155Special Valuation Con
8 A.2. Special Valuation Considerations: Taxes and Leases PART 1 UNIT 1

2.1.2 Lessee Decision Tree


In applying the OWNES criteria to determine whether a lease should be treated as a finance or
an operating lease, the following tree can be used:

Ownership transfers to the lessee at the


end of the lease term

No

Written purchase option which lessee is


reasonably certain to exercise

No

Net present value equal to or


substantially exceeds fair value of asset

"Yes" to any of the OWNES No


criteria = Finance Lease
Economic life (major part) of the
underlying asset within lease term

No

Specialized asset such that it will not


have an expected, alternative use to lessor

No

"No" to all of the OWNES


criteria = Operating Lease

Pass Key

Assuming that a lease is greater than 12 months in duration, the determination of how a
lessor and lessee account for a lease is based on the OWNES PC criteria.
Lessor
— Sales-Type Lease: At least one of the OWNES criteria is met.
— Direct Financing Lease: None of the OWNES criteria are met, but both of the PC criteria
are met.
— Operating Lease: None of the OWNES criteria are met, and either one or no PC criteria
are met.
Lessee
— Finance Lease: At least one of the OWNES criteria is met.
— Operating Lease: None of the OWNES criteria are met.

1–156 Module 8 A.2.


© Becker Special Valuation
Professional Education Considerations: Taxes
Corporation. All rights and Leases
reserved.
PART 1 UNIT
8 1 A.2. Special Valuation Considerations: Taxes and Leases

2.2 Lessee Accounting for Operating Leases


If the lease is an operating lease, the balance sheet will reflect a right-of-use (ROU) asset and
lease liability and both will be amortized over the life of the lease using the effective interest
method. The ROU asset and lease liability amounts are calculated using the present value of the
lease payments, using the appropriate discount rate. On the income statement, lease expense
will be recognized each year over the lease term using the straight-line method for expense
measurement. Instead of reporting interest expense on the income statement, the lessee will
report the interest as part of lease expense.

Pass Key

When calculating the present value of the lease payments for the purpose of calculating
the ROU asset and lease liability, keep in mind the following:

——Beginning of period = PV of an annuity due


Periodic payment
——End of period = PV of an annuity (in arrears/ordinary)

Purchase option
Or: ——PV of $1
Guaranteed residual

Note: Although leases generally require payment at the beginning of the period (first
payment at lease inception), some exam questions may state that the lease payments are
made at the end of each period. Read each question carefully to determine whether you are
dealing with an annuity due (payment at the beginning of each period) or an ordinary annuity
(payment at the end of each period) and be sure to use the correct present value factors.

Initial entry:

DR ROU asset $XXX


CR Lease liability $XXX

Subsequent entries:

DR Lease expense $XXX


CR Cash/lease liability $XXX

DR Lease liability $XXX


CR Accumulated amortization—ROU asset $XXX

© Becker Professional Education Corporation. All rights reserved. Module 8 A.2.


1–157Special Valuation Con
8 A.2. Special Valuation Considerations: Taxes and Leases PART 1 UNIT 1

Example 6 Reporting an Operating Lease: Lessee's Books

Facts: On January 1, Year 2, a lessee enters into a three-year asset operating (capital) lease
with annual payments of $18,000 per year. The first payment will be made December 31
and the interest rate implicit in the lease is 5.75 percent. (The present value of an ordinary
annuity for three years at 5.75% = 2.685424.)
Required: Prepare the journal entries for the lessee at the commencement date, the end
of Year 2, the end of Year 3, and the end of Year 4.
Solution:
January 1, Year 2, journal entry:
The present value of $18,000 for three years (first payment made at the end of Year 1) at a
rate of 5.75 percent per year is equal to $48,338.

DR ROU asset $48,338


CR Lease liability $48,338

Lease Lease Interest Reduction in Carrying Value


Date Liability Expense Expense ROU Asset of ROU Asset
$48,338 $48,338
12/31/Year 2 $33,117 $18,000 $2,779 $15,221 $33,117
12/31/Year 3 $17,021 $18,000 $1,904 $16,096 $17,021
12/31/Year 4 – $18,000 $ 979 $17,021 –

December 31, Year 2, journal entry:


The lease payment of $18,000 comprises interest expense and the amortization of the ROU
asset, as calculated in the table above.

DR Lease expense $18,000


DR Lease liability 15,221
CR Cash $18,000
CR Accumulated amortization—ROU asset 15,221

(continued)

1–158 Module 8 A.2.


© Becker Special Valuation
Professional Education Considerations: Taxes
Corporation. All rights and Leases
reserved.
PART 1 UNIT
8 1 A.2. Special Valuation Considerations: Taxes and Leases

(continued)

December 31, Year 3, journal entry:

DR Lease expense $18,000


DR Lease liability 16,096
CR Cash $18,000
CR Accumulated amortization—ROU asset 16,096

December 31, Year 4, journal entry:

DR Lease expense $18,000


DR Lease liability 17,021
CR Cash $18,000
CR Accumulated amortization—ROU asset 17,021

Once the final entry has been recorded, the ROU asset is fully amortized.

2.2.1 Finance Leases


If the lease is a finance lease, the lessee will recognize both an ROU asset and a corresponding
liability on its balance sheet. The liability will equal the present value of lease payments owed.
The ROU asset will include initial direct costs (such as commissions paid, legal and consulting
fees, etc.) that were incurred as a result of the lease execution, as well as any lease payments
made by the lessee to the lessor at or before lease commencement. Any incentives received by
the lessee from the lessor will reduce the value of the asset.
Initial entry:

DR ROU asset $XXX


CR Lease liability $XXX

Subsequent entries:

DR Interest expense $XXX


DR Lease liability XXX
CR Cash/lease payable $XXX

DR Amortization expense $XXX


CR Accumulated amortization—ROU asset $XXX

Unlike with operating (capital) leases, the amortization of the ROU asset for a finance lease will
be expensed based on how the entity recognizes amortization expense on similar assets.

© Becker Professional Education Corporation. All rights reserved. Module 8 A.2.


1–159Special Valuation Con
8 A.2. Special Valuation Considerations: Taxes and Leases PART 1 UNIT 1

Example 7 Reporting a Finance Lease: Lessee's Books

Facts: On January 1, Year 2, a lessee enters into a three-year asset lease with annual
payments of $18,000 per year. The first payment will be made December 31 and the
interest rate implicit in the lease is 5.75 percent. The lease qualifies as a finance lease.
Required: Assuming straight-line amortization, prepare the journal entries for the lessee at
the commencement date, the end of Year 2, the end of Year 3, and the end of Year 4.
Solution:
January 1, Year 2, journal entry:
The present value of $18,000 for three years (first payment made at the end of Year 1) at a
rate of 5.75 percent per year is equal to $48,338.

DR ROU asset $48,338


CR Lease liability $48,338

Lease Total Lease Interest Amortization Carrying Value


Date Liability Expense Expense Expense of ROU Asset
$48,338 $48,338
12/31/Year 2 $33,117 $18,892 $2,779 $16,113 $32,225
12/31/Year 3 $17,021 $18,017 $1,904 $16,113 $16,112
12/31/Year 4 – $17,091 $ 979 $16,112 –

December 31, Year 2, journal entries:


The lease payment of $18,000 comprises interest expense and the reduction of the lease
liability as calculated in the table above. The ROU asset will be amortized at $48,338 over
three years, or $16,113 per year.

DR Interest expense $ 2,779


DR Lease liability 15,221
CR Cash $18,000

DR Amortization expense 16,113


CR Accumulated amortization—ROU asset 16,113

December 31, Year 3, journal Entries:

DR Interest expense $ 1,904


DR Lease liability 16,096
CR Cash $18,000

DR Amortization expense 16,113


CR Accumulated amortization—ROU asset 16,113

(continued)

1–160 Module 8 A.2.


© Becker Special Valuation
Professional Education Considerations: Taxes
Corporation. All rights and Leases
reserved.
PART 1 UNIT
8 1 A.2. Special Valuation Considerations: Taxes and Leases

(continued)

December 31, Year 4, journal entries:

DR Interest expense $ 979


DR Lease liability 17,021
CR Cash $18,000

DR Amortization expense 16,112


CR Accumulated amortization—ROU asset 16,112

Once the final entry has been recorded, the ROU asset is fully amortized.

Pass Key

Although GAAP requires measuring the ROU asset using the one methodology described
above, IFRS allows for alternative measurement bases based on other standards, such
as the Investment Property fair value model (from IAS 40) and the Property, Plant, and
Equipment model (from IAS 16).

2.3 Financial Statement Presentation LOS 1A2u

2.3.1 Balance Sheet


ROU assets and associated lease liabilities may either be recognized as separate line items
on the balance sheet (in their respective sections) or included with other assets/liabilities and
disclosed separately in the notes to the financial statements (indicating which line items in the
balance sheet include them). The portion of lease liabilities due within a year or the operating
cycle, whichever is longer, should be reported in the current section and the remainder in
the long-term section. Finance and operating lease ROU assets and lease liabilities cannot be
presented together. The ROU asset will be amortized, and the lease liability will be paid down
over the life of the lease.
Finance Lease

ROU Asset Lease Liability

Amortize based Principal pay down


on criteria below over life of lease

The ROU asset will be amortized beginning on the commencement date using a straight-line
basis (unless another methodology better reflects usage and consumption).

© Becker Professional Education Corporation. All rights reserved. Module 8 A.2.


1–161Special Valuation Con
8 A.2. Special Valuation Considerations: Taxes and Leases PART 1 UNIT 1

Criteria for determining amortization:


yy Amortize over the underlying asset's useful life if ownership or written option criteria
are met.
yy Amortize over shorter of the lease term or the useful life of the asset if net present
value, economic life, or specialized asset criteria are met.

Illustration 2 Finance Lease: Amortization

Cooper Industries leases a boat for seven years from Kirkland Inc. There is no ownership
transfer at the end of the lease, and Cooper has not been given an option to purchase the
boat. The present value of the lease payments is equivalent to the boat's fair value, and the
boat has an accounting useful life of five years. On Cooper's books, this will qualify as a finance
lease because of the equivalence of the present value of the lease payments to the boat's fair
value (the N criteria is met). The ROU asset will be amortized over five years because this is the
lesser of the useful life of the asset (five years) and the lease term (seven years).

2.3.2 Income Statement


For operating leases, lease expense will be included in income from continuing operations
on the lessee's income statement. For finance leases, the income statement will include the
amortization of the ROU asset and the portion of the lease expense related to interest on the
lease liability.

2.3.3 Cash Flow Statement


For operating leases, lease payments (which include all variable lease payments) are classified as
cash flow from operations. Payments for short-term leases are also included in cash flow from
operations. Any payments needed to bring the asset to a condition and location in preparation
for its intended use are considered investing activities.
For finance leases, the principal portion of the lease payment is a cash flow from financing, the
interest portion of the lease payment is a cash flow from operations, and any variable lease
payments and short-term lease payments not included in the lease liability are classified as cash
flows from operations.

Cash Flow—Operations Cash Flow—Investing Cash Flow—Financing


Operating Leases
Lease payments Preparing asset for intended use

Variable lease payments

Short-term lease payments


Finance Leases
Interest payments Principal payments

Variable and short-term lease


payments not included in the
lease liability

1–162 Module 8 A.2.


© Becker Special Valuation
Professional Education Considerations: Taxes
Corporation. All rights and Leases
reserved.
PART 1 UNIT
8 1 A.2. Special Valuation Considerations: Taxes and Leases

3 Lease Differences Between U.S. GAAP and IFRS LOS 1A2ff

The following table summarizes four differences in the treatment of leases for financial
reporting purposes between U.S. GAAP and IFRS:

Subject U.S. GAAP Requirements IFRS Requirements


Classification Under U.S. GAAP, lessees classify All leases, other than short-term
leases as either finance leases or leases, are accounted for as
operating leases. The lessee is finance leases by recognizing a
required to recognize a right-of- right-of-use asset and a lease
use (ROU) asset and a lease liability liability. Operating leases are not
for all leases. applicable to lessees.
Amortization Finance lease: The lessee The lessee recognizes amortization
recognizes amortization of the of the right-of-use asset and
right-of-use asset and separately separately recognizes interest
recognizes interest expense expense and repayments of the
and repayments of the principal principal portion of the lease
portion of the lease liability. liability.
Operating leases: The lessee
recognizes a single lease cost
(lease expense) calculated on a
straight-line basis over the term of
the lease; the cost includes both
the amortization of the right-of-use
asset and the interest expense on
the lease liability.
Short-term leases The lessee can elect not to A lessee may elect not to apply the
recognize right-of-use assets or requirements to recognize a right-
lease liabilities for short-term of-use asset and lease liability for
leases of 12 months or less and short-term leases and immaterial
instead to recognize the lease leases and may instead recognize
payments as expenses on a the lease payments as expenses
generally straight-line basis over either on a straight-line basis or
the lease term. on any other systematic basis as
management finds reasonable
under the circumstances.
Measurement of The right-of-use asset at the Revaluation is permitted.
the value of the inception of the lease is measured
right-of-use as the lessee's historical cost of
the right-of-use. The historical cost
does not change during the term
of the lease, although that cost is
amortized.

© Becker Professional Education Corporation. All rights reserved. Module 8 A.2.


1–163Special Valuation Con
8 A.2. Special Valuation Considerations: Taxes and Leases PART 1 UNIT 1

Question 1 MCQ-12409

The Benedit Corp. is the lessee on a seven-year lease for a new excavator. The lease
begins immediately after the end of a two-year lease with the same lessor (C-Site Inc.)
for a different excavator. The economic life of the new excavator is eight years, but C-Site
realistically expects the equipment to be useful for 10 years. C-Site will not transfer the
excavator to Benedit when the lease is over, and there is no written purchase option in
the lease.
The ROU asset for the lease of the new excavator on Benedit's books will be amortized over:
a. 7 years.
b. 8 years.
c. 9 years.
d. 10 years.

Question 2 MCQ-12410

A lessee enters a two-year lease for a truck with an economic life of 10 years. The lessee
intends to return the truck at the end of the lease. The lessor and lessee have partnered on
several leases together and the lessor has ascertained that the collection of lease payments
to be highly probable.
Based on the facts described above, the lessor will classify the lease as:
a. Finance.
b. Operating.
c. Sales-type.
d. Direct financing.

Question 3 MCQ-12411

In a company's first year of operations, income tax expense exceeded income tax payable.
Which of the following situations most likely created the difference?
a. Annual rent received from a tenant on a leased building was paid in full at the start
of the year.
b. The company recorded higher bad debt expense on its receivables than the
amount the company deducted on the income tax return during the year.
c. An asset was purchased and at year-end had a tax basis that was below the value
on the balance sheet by $5,000.
d. A $10,000 premium payment was made on a life insurance policy where the
company is designated as the beneficiary.

1–164 Module 8 A.2.


© Becker Special Valuation
Professional Education Considerations: Taxes
Corporation. All rights and Leases
reserved.
PART 1 UNIT
8 1 A.2. Special Valuation Considerations: Taxes and Leases

Question 4 MCQ-12412

Ridgeland Co., a cash basis taxpayer, incurred the following transactions during Year 1:
yyReceived municipal bond interest payments totaling $5,000
yyAccrued warranty expenses $12,000, with no warranty expense payouts during the year
yyPaid $26,000 at the end of the year for rented space beginning in Year 2
If the income tax rate is 21 percent for the current year and for all future years, the
transactions described above will result in a net deferred tax liability of:
a. $1,890.
b. $2,940.
c. $9,000.
d. $14,000.

© Becker Professional Education Corporation. All rights reserved. Module 8 A.2.


1–165Special Valuation Con
8 A.2. Special Valuation Considerations: Taxes and Leases PART 1 UNIT 1

NOTES

1–166 Module 8 A.2.


© Becker Special Valuation
Professional Education Considerations: Taxes
Corporation. All rights and Leases
reserved.
A.2. Revenue

9
MODULE
PART 1 UNIT 1

Recognition and
Income Measurement
Part 1
Unit 1

This module covers the following content from the IMA Learning Outcome Statements.

CMA LOS Reference: Part 1—Section A.2. Revenue Recognition and Income
Measurement

The candidate should be able to:


x. apply revenue recognition principles to various types of transactions
y. demonstrate an understanding of revenue recognition for contracts with customers
using the five steps required to recognize revenue
z. demonstrate an understanding of the matching principle with respect to revenues and
expenses and be able to apply it to a specific situation
aa. define gains and losses and indicate the proper financial statement presentation
bb. demonstrate an understanding of the treatment of gain or loss on the disposal of
fixed assets
cc. demonstrate an understanding of expense recognition practices
dd. define and calculate comprehensive income
ee. identify the correct treatment of discontinued operations
ff. identify and describe the following differences between U.S. GAAP and IFRS: (i) expense
recognition, with respect to share-based payments and employee benefits

1 Revenue Recognition LOS 1A2x

LOS 1A2y
Revenue recognition occurs when an entity satisfies a performance obligation by transferring
either a good or a service to a customer. Revenue should be recognized at an amount that
reflects the expected consideration the entity is entitled to receive in exchange for the good or
service provided.
All entities (public, private, not-for-profit) that enter into contracts with customers to transfer
goods, services, or nonfinancial assets (unless governed by other standards) are subject to the
revenue recognition standard. Certain contracts, such as those covering leases, insurance, non-
warranty guarantees, and financial instruments, are covered under other standards.

© Becker Professional Education Corporation. All rights reserved. Module 9 1–167


A.2. Revenue Recogniti
9 A.2. Revenue Recognition and Income Measurement PART 1 UNIT 1

In order to properly apply the revenue recognition standard, an entity should implement the
five-step approach described below:
Step 1: Identify the contract with the customer
Step 2: Identify the separate performance obligations in the contract
Step 3: Determine the transaction price
Step 4: Allocate the transaction price to the separate performance obligations
Step 5: Recognize revenue when or as the entity satisfies each performance obligation

LOS 1A2z 1.1 Step 1: Identify the Contract(s) With the Customer
1.1.1 Definitions
A contract is defined as an agreement between two or more parties that creates enforceable
rights and obligations. Depending on an entity's typical business practices, contracts can be
verbal, written, or implied.
A customer is a party that has contracted with an entity to exchange consideration in order to
obtain goods or services that are an output of the entity's ordinary activities.

1.1.2 Criteria for Revenue Recognition


Revenue is only recognized when a contract meets all of the criteria listed below. The criteria
assessment is performed at contract inception and if all criteria are met, reassessments should
only be needed if significant changes occur. If all of the criteria are not met at inception, regular
reassessments should follow.
All parties have approved the contract and have committed to perform their obligations.
The rights of each party regarding contracted goods or services are identified.
Payment terms can be identified.
The contract has commercial substance, meaning future cash flows (amount, risk, and
timing) are expected to change as a result of the contract.
It is probable (based on the customer's intent and ability to pay when due) that the entity
will collect substantially all of the consideration due under the contract.
If the criteria above are not met but consideration has been paid by the customer, an entity
can recognize revenue if the consideration is nonrefundable and either there are no remaining
obligations to transfer goods/services or the contract has terminated. If not recognized as
revenue, the consideration received is booked as a liability.

1–168 Module 9 A.2. Revenue


© Becker Professional Recognition
Education and Income
Corporation. All rights Measurement
reserved.
PART 1 UNIT
9 1 A.2. Revenue Recognition and Income Measurement

Example 1 Identifying the Contract

Facts: On March 1, Year 1, Bulldog Inc. entered into a contract to transfer a product to
Kitty Inc. on September 1, Year 1. Kitty will pay the full contract price of $15,000 to Bulldog
by August 1, Year 1. Bulldog transferred the product to Kitty on September 1, Year 1. The
cost of the product totaled $9,000.
Required: Determine the journal entries that Bulldog will book to account for this transaction.
Solution:
March 1, Year 1, Journal Entry: No entry is required because neither party has performed
according to the contract.
August 1, Year 1, Journal Entry: A contract liability (e.g. unearned sales revenue) is recognized
when the cash is received in advance.

DR Cash $15,000
CR Unearned sales revenue $15,000

September 1, Year 1, Journal Entry: Revenue is recorded when the product is transferred
from the seller to the buyer.

DR Unearned sales revenue $15,000


CR Sales revenue $15,000
DR Cost of goods sold 9,000
CR Inventory 9,000

1.1.3 Combination of Contracts


When two or more contracts are entered into with the same customer or with related parties
of the customer at or near the same time, the contracts should be combined and accounted for
as a single contract if either the contracts are negotiated as a package with a single commercial
objective, consideration for one contract is tied to the performance or price of another contract,
or the goods/services promised represent a single performance obligation.

1.1.4 Contract Modification


A contract modification represents a change in the price or scope (or both) of a contract approved by
both parties. When a modification occurs, it is either treated as a new contract or as a modification
of the existing contract. The modification is treated as a new contract if both the scope increases
due to the addition of distinct goods or services and the price increase appropriately reflects the
stand-alone selling prices of the additional goods/services. If not accounted for as a new contract, the
modification is treated as part of the existing contract (for non-distinct goods and services) with an
adjustment to revenue to reflect the change in the transaction price.

© Becker Professional Education Corporation. All rights reserved. Module 9 1–169


A.2. Revenue Recogniti
9 A.2. Revenue Recognition and Income Measurement PART 1 UNIT 1

1.2 Step 2: Identify the Separate Performance Obligations


in the Contract
1.2.1 Performance Obligation Defined
A performance obligation is a promise to transfer a good or a service to a customer. The transfer
can be either an individual good or service (or a bundle of goods or services) that is distinct, or
a series of goods or services that are substantially the same and are thereby transferred in the
same manner. If the promise to transfer a good or a service is not distinct from other goods or
services, they will all be combined into a single performance obligation.

1.2.2 A Distinct Good or Service


In order to be distinct, both criteria below must be met:
1. The promise to transfer the good or service is separately identifiable from other goods or
services in the contract; and
2. The customer can benefit either from the good or service independently or when combined
with the customer's available resources.

1.2.3 Separately Identifiable


A transfer of a good or service is separately identifiable if the entity does not integrate the good
or service with other goods or services in the contract; the good or service does not customize
or modify another good or service in the contract; or the good or service does not depend on or
relate to other goods or services promised in the contract.
Factors that indicate two or more promises to transfer a good or a service to a customer are not
separately identifiable include (but are not limited to) the following:
The goods or services are highly interrelated or interdependent.
The entity provides a significant service of integrating the good or service with other goods
or services promised in the contract into a bundle of goods or services that represent the
combined output contracted for by the customer.

Example 2 Single Performance Obligation

Facts: Tanner Co. is building a multi-unit residential complex. The entity enters into a
contract with a customer for a specific unit that is under construction. The goods and
services to be provided in the contract include procurement, construction, piping, wiring,
installation of equipment, and finishing.
Required: Identify the performance obligation(s) in this contract.
Solution: Although the goods and services provided by the contractor are capable of being
distinct, they are not distinct in this contract because the goods and services cannot be
separately identified from the promise to construct the unit. The contractor will integrate
the goods and services into the unit, so all the goods and services are accounted for as a
single performance obligation.

1–170 Module 9 A.2. Revenue


© Becker Professional Recognition
Education and Income
Corporation. All rights Measurement
reserved.
PART 1 UNIT
9 1 A.2. Revenue Recognition and Income Measurement

Example 3 Separately Identifiable Performance Obligations

Facts: A software developer enters into a contract with a customer to transfer a software
license, perform installation, and provide software updates and technical support for
five years. The developer sells the license, installation, updates, and technical support
separately. The entity determines that each good or service is separately identifiable
because the installation does not modify the software and the software is functional
without the updates and technical support.
Required: Identify the performance obligation(s) in this contract.
Solution: The software is delivered before the installation, updates, and technical support
and is functional without the updates and technical support, so the customer can benefit
from each good or service on its own. The developer has also determined that the software
license, installation, updates, and technical support are separately identifiable. On this
basis, there are four performance obligations in this contract:
1. Software license
2. Installation service
3. Software updates
4. Technical support

1.3 Step 3: Determine the Transaction Price


1.3.1 Transaction Price Defined
The transaction price represents the amount of consideration that an entity can expect to be
entitled to receive in exchange for transferring promised goods or services to a customer.
The transaction price should be determined based on considering the effects of: variable
consideration (and any constraining estimates), significant financing if applicable, noncash
considerations, and any consideration payable to the customer (if applicable).

1.3.2 Variable Consideration


The amount of variable consideration should be estimated by taking a range of possible
amounts and using either the expected value (which sums probability-weighted amounts) or the
most likely amount—whichever is assumed to be the better predictor. Variable consideration
should only be included in the price if it is probable that a significant revenue reversal will not be
required once any uncertainty tied to the consideration is resolved.

1.3.3 Significant Financing


Time value of money should be an adjustment to the transaction price if the timing of the
payments per the contract provides either the customer or the entity with a significant benefit
in regard to financing the transfer of goods or services. Revenue should be recognized based on
the price that would have been paid in cash by the customer at the time of transfer. If the time
between the transfer of goods/services and the payment by the customer is anticipated to be
less than one year, discounting the transaction price is unnecessary.

1.3.4 Noncash Consideration


Noncash consideration should be measured at fair value at contract inception.

© Becker Professional Education Corporation. All rights reserved. Module 9 1–171


A.2. Revenue Recogniti
9 A.2. Revenue Recognition and Income Measurement PART 1 UNIT 1

1.3.5 Consideration Payable to a Customer


Any consideration (cash, credits, vouchers, etc.) that is payable to a customer should be treated
as a reduction in the transaction price and revenue recognized by the entity unless the entity is
receiving goods or services transferred by the customer.

Example 4 Time Value of Money

Facts: On January 1, Year 5, SDF sold furniture to a customer for $4,000 with three years'
interest‑free credit. The customer took delivery of the furniture on that day. The $4,000
is payable to SDF on December 31, Year 7. The applicable discount rate based on the
customer's credit profile is 8 percent.
Required: Determine the transaction price for the sale of furniture.
Solution: The transaction price is $3,175 ($4,000 × 1/(1.08)3) because the time value of
money must be considered when determining the transaction price.
Note that interest income will also be recognized each year as follows:
——Year 5: $3,175 × 8% = $254
——Year 6: ($3,175 + $254) × 8% = $274
——Year 7: ($3,175 + $254 + $274) × 8% = $296

1.4 Step 4: Allocate the Transaction Price to the Performance


Obligations in the Contract
1.4.1 Allocation Defined
If there is more than one performance obligation within a contract, the transaction price should
be allocated to each separate performance obligation based on the amount of consideration
that would be expected for satisfying each unique obligation. The stand-alone selling price (and
any applicable discount or variable consideration) of each distinct good or service underlying
each performance obligation should be determined at contract inception.

1.4.2 Stand-alone Selling Price


The price an entity would sell the promised good or service to a customer on a stand-alone
basis. Once this price is determined for each obligation within the contract, the total transaction
price should be allocated in proportion to the stand-alone selling prices.

1.4.3 Discounts
A discount exists when the sum of the stand-alone prices for each obligation within a contract
exceeds the total consideration for the contract. A discount should be allocated proportionally to
all obligations within the contact.

1–172 Module 9 A.2. Revenue


© Becker Professional Recognition
Education and Income
Corporation. All rights Measurement
reserved.
PART 1 UNIT
9 1 A.2. Revenue Recognition and Income Measurement

1.4.4 Variable Consideration


If applicable, variable consideration may be attributable to the entire contract, individual
performance obligations within a contract, or distinct goods or services within a single
performance obligation.

1.4.5 Transaction Price Changes


If the transaction price changes after contract inception, the change should be allocated to the
performance obligations in the contract on the same basis that was used at inception. Changes
in stand-alone selling prices after inception should not be reallocated.

Example 5 Allocating the Transaction Price

Facts: A software company enters into a $250,000 contract with a customer to transfer
a software license, perform installation service, and provide technical support for a
three-year period. The entity sells the license, installation service, and technical support
separately. The installation service and technical support could be performed by other
entities and the software remains functional in the absence of these services. The contract
price must be paid on installation of the software, which is planned for March 1, Year 1.
Required: Determine how the software company should recognize the revenue for these
transactions.
Solution: The entity identifies three performance obligations in the contract for the
following goods and services:
1. Software license
2. Installation service
3. Technical support
The stand-alone selling price can be determined for each performance obligation. The
license is usually sold for $160,000, the installation service is $20,000, and technical support
runs $30,000 per year. The fair value of the contract is determined to be $270,000. Based
on the relative fair values, the allocation of revenue is as follows:
——Software license [($160,000/$270,000) × $250,000] = $148,148
——Installation service [($20,000/$270,000) × $250,000] = $18,519
——Technical support [($90,000/$270,000) × $250,000] = $83,333

(continued)

© Becker Professional Education Corporation. All rights reserved. Module 9 1–173


A.2. Revenue Recogniti
9 A.2. Revenue Recognition and Income Measurement PART 1 UNIT 1

(continued)

The journal entry to record the $250,000 payment made on March 1 appears below.
March 1, Year 1:

DR Cash $250,000
CR License revenue $148,148
CR Service revenue 18,519
CR Unearned service revenue 83,333

Revenue is recorded for the sale of the license and the installation at the time of sale. The
technical support will be recognized on a monthly basis as the support is provided.
December 31, Year 1:

DR Unearned service revenue $23,148


CR Service revenue $23,148

At year-end, an adjusting entry is made to record 10 months of technical support


($83,333/36 = $2,314.80; $2,314.80 × 10 months = $23,148) through the end of Year 1.
The remaining technical support will be recorded in Years 2 and 3.

1.5 Step 5: Recognize Revenue When (or as) the Entity Satisfies
a Performance Obligation
1.5.1 Transfer of Control
An entity should recognize revenue when the entity satisfies a performance obligation by
transferring the good or service to the customer, who thereby obtains control of the asset.
Control implies the ability to obtain the benefits from and direct the usage of the asset while also
preventing other entities from obtaining benefits and directing usage. Performance obligations
may be satisfied either over time or at a point in time.

1.5.2 Satisfied Over Time


Revenue is recognized over time if any one of the criteria below is met:
The entity's performance creates or enhances an asset that the customer controls.
The customer receives and consumes the benefits of the entity's performance as the
entity performs it (e.g., service contracts, such as a cleaning service or a monthly payroll
processing service).
The entity's performance does not create an asset with alternative use to the entity
(assessed at inception) and the entity has an enforceable right to receive payment for
performance completed to date.
In order to recognize revenue, the entity must be able to reasonably measure progress toward
completion. Progress can be measured using output and input methods.

1–174 Module 9 A.2. Revenue


© Becker Professional Recognition
Education and Income
Corporation. All rights Measurement
reserved.
PART 1 UNIT
9 1 A.2. Revenue Recognition and Income Measurement

1. Output Methods
By using output methods, revenue is recognized based on the value to the customer of the
goods or services transferred to date relative to the remaining goods or services promised.
Examples of output methods include: units produced or delivered, time elapsed, milestones
achieved, surveys of performance completed to date, and appraisals of results achieved.
These methods should only be used when the output selected represents the entity's
performance toward complete satisfaction of the performance obligation. When the outputs
used to measure progress are not available or directly observable, an input method may
be necessary.

2. Input Methods
By using input methods, revenue is recognized based on the entity's efforts or inputs to the
satisfaction of the performance obligation relative to the total expected inputs. Examples of
input methods include: costs incurred relative to total expected costs, resources consumed,
labor-hours expended, and time elapsed. A disadvantage of input methods is that there may
not be a direct relationship between an entity's inputs and the transfer of control of goods
and services to a customer. If inputs are used evenly throughout the performance period,
revenue can be recognized on a straight-line basis.

Illustration 1 Straight-Line Basis

A health club enters into a contract with a customer for one year of unlimited health club
access for $75 per month. The health club determines that the customer simultaneously
receives and consumes the benefits of the club's performance, so the contract is a
performance obligation satisfied over time. Because the customer benefits from the
club's services evenly throughout the year, the best measure of progress toward complete
satisfaction of the performance obligation is a time-based measure. Revenue will be
recognized on a straight-line basis throughout the year at $75 per month.

In the absence of reliable information used to measure progress, if an entity expects to recover
its costs, revenue may be recognized to the extent that costs are recovered until the point at
which it can reasonably measure the outcome of the performance obligation.

1.5.3 Satisfied at a Point in Time


If the performance obligation is not satisfied over time, then it is satisfied at a point in time.
Revenue should be recognized at the point in time when the customer obtains control of
the asset.
Control would generally require the following:
The entity has a right to payment and the customer has an obligation to pay for an asset.
The customer has legal title to the asset.
The entity has transferred physical possession of the asset.
The customer has the significant rewards and risks of ownership.
The customer has accepted the asset.

© Becker Professional Education Corporation. All rights reserved. Module 9 1–175


A.2. Revenue Recogniti
9 A.2. Revenue Recognition and Income Measurement PART 1 UNIT 1

Example 6 Performance Obligation Satisfied Over Time

Facts: Tanner Co. is building a multi-unit residential complex. The entity enters into a
contract with a customer for a specific unit that is under construction. The contract has the
following terms:
——The customer pays a nonrefundable security deposit upon entering the contract.
——The customer agrees to make progress payments during construction.
——If the customer fails to make the progress payments, the entity has the right to all of the
consideration in the contract if it completes the unit.
——The terms of the contract prevent the entity from directing the unit to another customer.
Required: Determine whether this performance obligation is satisfied over time or at a
point in time.
Solution: This performance obligation is satisfied over time because:
——The unit does not have an alternative future use to the entity because it cannot be
directed to another customer.
——The entity has a right to payment for performance to date because the entity has a right
to all of the consideration in the contract if it completes the unit.

Example 7 Performance Obligation Satisfied at a Point in Time

Facts: Tanner Co. is building a multi-unit residential complex. The entity enters into a
contract with a customer for a specific unit that is under construction. The contract has the
following terms:
——The customer pays a deposit upon entering the contract that is refundable if the entity
fails to complete the unit in accordance with the contract.
——The remainder of the purchase price is due on completion of the unit.
——If the customer defaults on the contract before completion, the entity only has the right
to retain the deposit.
Required: Determine whether this performance obligation is satisfied over time or at a
point in time.
Solution: This is a performance obligation satisfied at a point in time because it is not a
service contract, the customer does not control the unit as it is created, and the entity does
not have an enforceable right to payment for performance completed to date (i.e., the
entity only has a right to the deposit until the unit is completed).

1–176 Module 9 A.2. Revenue


© Becker Professional Recognition
Education and Income
Corporation. All rights Measurement
reserved.
PART 1 UNIT
9 1 A.2. Revenue Recognition and Income Measurement

2 Income Measurement LOS 1A2z

2.1 The Matching Principle


Expenses are outflows, uses of assets, or the incurrence of liabilities from delivering goods or
services as part of normal operations. Expense recognition within the proper accounting period
ensures that users of financial information can assess the costs of revenues earned by the
company during the period and gives a more accurate analysis of the company's profitability.
Expenses are necessarily incurred to generate revenue. Expenses incurred to generate a specific
amount of revenue in a period are matched against that revenue (for example, matching
cost of goods sold to the related sales revenue). Expenses that do not have a direct link with
revenue may be recognized when cash is spent or when liabilities are incurred (such as selling,
general, and administrative expenses). Other expenses are allocated by systematic and rational
procedures to the period in which the assets provide benefit (such as depreciation expense).
Losses may result when future economic benefits of an asset are reduced or eliminated.

Example 8 Revenue and Expense Recognition

Facts: England Enterprises had the following transactions during Year 1:


— Earned revenues of $225,000 with $22,500 uncollected at year-end.
— Purchased merchandise for $110,000, of which $36,200 is owed at year-end. All
inventory purchased was sold during the year.
— Paid $3,500 for a two-year insurance policy on the first day of business.
— Employees earned a total of $9,100 in salaries during the year. $1,000 of this amount is
owed at year-end and will be paid on January 2, Year 2.
— Owners invested $30,000 in the business; on January 2, Year 1, the business borrowed
$45,000; and the owners signed a five-year, 6 percent note.
Required: Calculate the Year 1 net income for England.
Solution:

Revenue $ 225,000

Cost of goods sold (110,000)


Gross profit $ 115,000
Other Expenses
Insurance $(1,750)
Salaries (9,100)
Interest (2,700)
Total other expenses (13,550)
Net income $ 101,450

(continued)

© Becker Professional Education Corporation. All rights reserved. Module 9 1–177


A.2. Revenue Recogniti
9 A.2. Revenue Recognition and Income Measurement PART 1 UNIT 1

(continued)

Revenues earned by England during the year are recognized, even if all cash is not collected
at year-end.
Cost of goods sold consists of the purchases of $110,000 of inventory. No inventory
remains on hand; the entire cost of items purchased will be matched against revenues.
Insurance expense is $3,500 for the two-year period, beginning January 1, Year 1; for one
year the expense is $1,750: $3,500 payment ÷ 2 years' coverage = $1,750 expense each
year.
Salary expense consists of all salaries and wages earned during the period by employees,
regardless of the amount paid to employees.
Interest expense consists of interest costs incurred during the first year of operations. The
calculation is the $45,000 principal amount borrowed times the 6 percent interest rate
$45,000 loan × 6% interest rate = $2,700.

LOS 1A2aa 2.2 Gains and Losses


Gains are increases in equity, and losses are decreases in equity resulting from transactions that
are incidental to the ordinary course of business. Transactions resulting in gains/losses affect a
company's financial position. Examples of transactions that give rise to gains and losses include
the following:
Investments sold often result in gains and losses, as the selling price frequently differs from
the cost basis.
Property, plant, and equipment is replaced or sold during the ordinary course of business;
the replacement or sale will also result in gains/losses when the proceeds received from the
sale differ from the net book value.
Inventory losses due to decreases in the value of inventory on account of either changing
markets or obsolescence.
Impairments of assets

LOS 1A2bb 2.2.1 Calculating Gains/Losses on Disposals of Fixed Assets


To calculate a gain or loss on the disposal of a fixed asset, compare the selling price and net
book value of the asset. The net book value of a fixed asset is the difference between the initial
cost and accumulated depreciation on the asset (if the asset is depreciable. Land is never
depreciable). The selling price is the price received in exchange for the asset. When the selling
price of an asset exceeds the net book value, the company has a gain on the sale. When the
selling price of an asset is less than the net book value, the company has a loss on the sale.

1–178 Module 9 A.2. Revenue


© Becker Professional Recognition
Education and Income
Corporation. All rights Measurement
reserved.
PART 1 UNIT
9 1 A.2. Revenue Recognition and Income Measurement

Calculation of Gain/(Loss)

Step 1: Determine the net book value


Cost $ XXX
Less: accumulated depreciation, if any (XX)
Net book value $ XXX
Step 2: Calculate the gain/(loss)
Selling price $ XXX
Net book value (XXX)
Gain/(loss) on sale $XXX

Example 9 Calculating Gain on Sale

Facts: Ricky's Auto Shop purchased a piece of equipment for the shop at a cost of $50,000
on January 1, Year 1. The equipment had an estimated five-year life and an estimated
salvage value of $5,000. Ricky uses the straight-line method of depreciation. On December
31, Year 3, Ricky sold the equipment for $32,000.
Required: Calculate the gain or loss on the sale.
Solution:

Step 1: Determine the net book value


Cost $50,000
Less: accumulated depreciation (27,000)*
Net book value $23,000
* Annual straight-line depreciation =
($50,000 cost − $5,000 estimated salvage value) / 5-year life = $9,000 yearly
depreciation expense.
Accumulated depreciation =
3 years × $9,000 yearly depreciation expense = $27,000.

Step 2: Calculate the gain/(loss)


Selling price $32,000
Net book value (23,000)
Gain on sale $ 9,000

© Becker Professional Education Corporation. All rights reserved. Module 9 1–179


A.2. Revenue Recogniti
9 A.2. Revenue Recognition and Income Measurement PART 1 UNIT 1

Example 10 Calculating Loss on Sale

Facts: Ricky's Auto Shop purchased a piece of equipment for the shop at a cost of $50,000
on January 1, Year 1. The equipment had an estimated five-year life and an estimated
salvage value of $5,000. On December 31, Year 3, Ricky sold the equipment for $19,000.
Required: Calculate the gain or loss on the sale.
Solution:

Step 1: Determine the net book value


Cost $50,000
Less: accumulated depreciation (27,000)*
Net book value $23,000
Step 2: Calculate the gain/(loss)
Selling price $19,000
Net book value (23,000)
Loss on sale $ (4,000)
*See Example 9.

2.3 Operating Income vs. Nonoperating Income


Operating income consists of revenues, expenses, gains, and losses related to the primary
revenue-generating activities of the company.
Nonoperating income consists of revenues, expenses, gains, and losses related to peripheral
activities or transactions incidental to the primary activities of the company.
To determine where on the income statement the gain or the loss is reported, determine the use
of the asset or liability with which the gain or loss is associated. For example, sales of property,
plant, and equipment are used in the primary revenue-generating activities of a company. So,
gains and losses associated with the sale of these assets are generally considered a component
of operating income.
Investments are often held for purposes outside of the primary revenue-generating activities
of a company. So, gains and losses associated with the sale of these assets are generally
considered a component of nonoperating income.

1–180 Module 9 A.2. Revenue


© Becker Professional Recognition
Education and Income
Corporation. All rights Measurement
reserved.
PART 1 UNIT
9 1 A.2. Revenue Recognition and Income Measurement

Example 11 Classification of Gains/Losses

Facts: The following information is related to Ricky's Auto Shop as of December 31, Year 1.
Account Balance
Sales revenue $345,000
Interest revenue 12,000
Gain on sale of equipment 5,500
Cost of goods sold 156,000
Administrative expenses 35,000
Interest expense 15,000
Selling expense 40,000
Income tax expense 22,000
Loss on sale of investments 6,000
Required: Prepare an income statement with gains and losses appropriately presented.
Solution:

Ricky's Auto Shop


Income Statement
For the Year Ended December 31, Year 1
Sales revenue $345,000
Cost of goods sold (156,000)
Gross profit $189,000
Operating expenses:
Selling expense (40,000)
Administrative expenses (35,000)
Gain on sale of equipment 5,500
Total operating expenses (69,500)
Operating income $119,500
Other income/(expense)
Interest revenue 12,000
Interest expense (15,000)
Loss on sale of investments (6,000)
Total other income/expense (9,000)
Income before income taxes $110,500
Income tax expense (22,000)
Net income $ 88,500

The gain on the sale of equipment is included in the calculation of operating income because
that gain is associated with the sale of an asset used in the operations of the business.
The loss on the sale of investments is included in the calculation of other income/(expense)
as investments held by the company are not used in the operations of the business.

© Becker Professional Education Corporation. All rights reserved. Module 9 1–181


A.2. Revenue Recogniti
9 A.2. Revenue Recognition and Income Measurement PART 1 UNIT 1

Pass Key

When determining whether a gain or loss is operating or nonoperating, refer to the asset
or liability that resulted in the gain or loss. If the asset/liability is used in operations,
the gain/loss will be classified as an operating income item. If the asset/liability is not
used in the operations of the business, the gain/loss will be classified as a nonoperating
income item.

LOS 1A2cc 2.4 Expense Recognition


The timing of expense recognition should be based on related revenues. In addition to being
related to revenues, expense recognition affects the timing of asset and liability recognition.
The recording of an expense will result in either the outflow of cash (or another asset) or the
recognition of a liability for the payment of cash in the future.

LOS 1A2ff 2.4.1 Cost of Goods Sold (COGS)


Cost of goods sold represents the costs associated with sales of inventory. Under a perpetual
inventory system, the determination of cost of goods sold occurs throughout the year as each
sale occurs. Alternatively, under a periodic inventory system the determination of cost of goods
sold occurs only at the end of the year after a physical count of inventory is completed.
The journal entry to record costs of goods sold under either system is recorded as follows:

DR Cost of goods sold $XXX


CR Inventory $XXX

When revaluation of inventory under GAAP or IFRS is considered immaterial, the cost of goods
sold account will be used to record any necessary adjustment.
Recognize Loss in Current Period
Under U.S. GAAP, the write-down of inventory is usually reflected in cost of goods sold,
unless the amount is material, in which case the loss should be identified separately in the
income statement. IFRSs do not specify where an inventory write-down should be reported
on the income statement.
Reversal of Inventory Write-downs
Under U.S. GAAP, reversals of inventory write-downs are prohibited. IFRSs allow the reversal
of inventory write-downs for subsequent recoveries of inventory value. The reversal is
limited to the amount of the original write-down and is recorded as a reduction of total
inventory costs on the income statement (COGS) in the period of reversal.

2.4.2 Operating Expenses


Operating expenses are incurred in the ordinary course of business and are related to the costs
that are necessary to conduct the day-to-day operations of a business.

1–182 Module 9 A.2. Revenue


© Becker Professional Recognition
Education and Income
Corporation. All rights Measurement
reserved.
PART 1 UNIT
9 1 A.2. Revenue Recognition and Income Measurement

2.4.3 Selling Expenses


Selling expenses represent costs incurred in the generation of sales for a company. These costs
include the marketing efforts associated with advertising the product or service, commissions on
sales, and freight-out. Freight-out represents the transportation costs paid by the company to
deliver goods to customers (the cost of freight-in, which represents the transportation costs paid
by the company to receive inventory from supplies, is part of the cost of inventory purchased).
The timing of selling expense recognition is based on when the costs are incurred.

2.4.4 General and Administrative Expenses


General and administrative expenses represent costs incurred in support of a company's
nonmanufacturing operations. These nonmanufacturing costs include the following:
Utilities (water, waste, heating, and cooling expenses)
Supplies
Salaries and wages of general and administrative personnel
Rent (recognized ratably over the lease period)
Repairs and maintenance (amount necessary to maintain or keep an asset in operational
condition)
Insurance (recognized ratably over the policy period)
These costs are necessary to run the company but are related to the manufacture of a product
or a service related to manufacturing. These costs indirectly benefit the company as the
company generates revenue during the period, and they are expensed in the period incurred to
match properly against revenues.

2.4.5 Bad Debt Expense


Bad debt expense is necessary when businesses allow sales on a credit basis. An analysis of
receivables is necessary to estimate future credit losses associated with customers who will
ultimately not pay amounts owed.
Expense recognition will be determined based on a review of accounts receivable and
management's assessment of cash collection based on the analysis performed.

2.4.6 Depreciation Expense and Amortization Expense


Depreciation expense and amortization expense represent the allocation of the cost of long-
term assets over the estimated period of benefit from the asset. Tangible long-term assets are
depreciated and intangible assets are amortized. Both processes are intended to recognize the
benefit that extends across multiple accounting periods. To match those costs properly with
associated revenues, management must estimate the benefit period.

2.4.7 Interest Expense


Interest expense represents the cost of borrowing money. The interest on borrowings (the
principal) should be recorded in the period incurred to match properly against current period
revenues. Interest expense will be recorded at the end of each reporting period and is based
on the principal balance of the note times the interest rate charged and then adjusted for the
length of time the note is outstanding during the period.

© Becker Professional Education Corporation. All rights reserved. Module 9 1–183


A.2. Revenue Recogniti
9 A.2. Revenue Recognition and Income Measurement PART 1 UNIT 1

Example 12 Year-End Interest Accrual

Facts: Harris Hats borrows $100,000 from Emmons Corp. on February 1, Year 1. The note
bears interest of 12 percent, and interest and principal are due February 1, Year 3.
Required: Determine the amount of interest expense Harris Hats must accrue at the end
of Year 1 and record appropriate entry at the end of Year 1.
Solution:
Calculation: $100,000 principal × 12 percent annual interest rate × 11/12 adjustment for
the length of time the note is outstanding during the first year = $11,000
The journal entry to record interest expense is recorded as follows:

DR Interest expense $11,000


CR Interest payable $11,000

2.4.8 Income Tax Expense


Income tax expense reported for the year includes current income tax payable plus any future
tax due on current year income statement items minus tax currently due on current year income
statement items that will be deductible in future years. Differences between the timing of
recognition of certain items on the tax return and financial statements create either permanent
differences or temporary differences.

LOS 1A2ff 2.4.9 Employee Benefits


Employees receive compensation through hourly wages, salaries, bonuses, commissions, and
eligibility for participation in stock compensation plans and pensions. In addition to these forms
of compensation, employees may be eligible for future compensated absences in the form of
sick leave, vacation, and holiday pay.
Employers are required to accrue liabilities associated with future absences when the following
conditions exist:
1. The employer's obligation to compensate employees for future absences is attributable to
services already rendered by employees.
2. The obligation relates to rights that vest (are not contingent on an employee's future service)
or accumulate (may be carried forward to one or more accounting periods subsequent to
that in which earned).
3. Payment of the compensation is probable.
4. The amount can be reasonably estimated.
Under U.S. GAAP, companies should recognize expenses and related liabilities for compensated
absences in the year earned by employees.
Under IFRS, short-term benefits expected to be settled within 12 months from the annual
reporting period must be accrued. The amount of future paid absences must be accumulated
and carried forward if the current period's entitlement is not used in full. A liability results from
employees' services rendered, and those rendered services increase entitlement to future
paid absences.

1–184 Module 9 A.2. Revenue


© Becker Professional Recognition
Education and Income
Corporation. All rights Measurement
reserved.
PART 1 UNIT
9 1 A.2. Revenue Recognition and Income Measurement

2.4.10 Sick Pay Benefits


Some companies only pay sick pay if an illness occurs. Other companies may allow employees to
take time off related to accumulated sick pay, even if an employee is not ill. Under U.S. GAAP, an
employer is not required to accrue a liability for non-vesting accumulating rights to receive sick
pay benefits. Estimates of future sick pay are not reliable, and the cost of making and evaluating
those estimates do not justify making an accrual. However, the employer should accrue sick pay
benefits if the four criteria are met and the estimate is reliable.
IFRS requires the accrual of sick pay benefits as services are rendered by employees.

2.4.11 Noncompensatory Stock Purchase Plans


Under U.S. GAAP, an employee stock purchase plan is noncompensatory if it meets all of the
following requirements:
Substantially all full-time employees meeting limited employee qualifications may
participate. Excluded are officers and employees owning a specific amount of the
outstanding stock in the corporation.
Stock is offered to eligible employees equally, but the plan may limit the total amount of
shares that can be purchased.
The time permitted to exercise the rights is limited to a reasonable period.
Any discount from the market price is no greater than would be a reasonable offer of stock
to shareholders or others.
Stock option plans that meet the requirements of a noncompensatory plan do not require the
recognition of compensation expense by the sponsoring company. Plans that do not contain
these characteristics are usually classified as compensatory plans and require the recognition of
compensation expense.
Under IFRS, employee stock purchase plans and stock options are generally considered to be
compensatory.

2.4.12 Compensatory Stock Option and Stock Purchase Plans


Compensatory stock options and stock purchase plans require the recognition of compensation
expense. Under the fair value method, total compensation expense is measured by applying
an acceptable fair value pricing model such as the Black-Scholes Option Pricing Model. This
compensation expense, calculated on the grant date of the options, is allocated over the service
period in accordance with the matching principle. The service period is the vesting period which
is the time between the grant date and the vesting date.
Definitions
yy Exercise Price: The price at which the underlying stock can be purchased pursuant to
the option contract.
yy Exercise Date: The date by which the option holder must use the option to purchase
the underlying shares of stock (and typically the date at which the stock options
outstanding account is reduced).
yy Fair Value of the Option: Determined by an economic pricing model such as the Black-
Scholes method.
yy Grant Date: The date the option is issued.

© Becker Professional Education Corporation. All rights reserved. Module 9 1–185


A.2. Revenue Recogniti
9 A.2. Revenue Recognition and Income Measurement PART 1 UNIT 1

yy Vesting Period: The period over which the employee has to perform services in order to
earn the right to exercise the options (i.e., the time from the grant date to the vesting date).
yy Service Period: The period over which compensation expense is recognized (i.e., the
period in which the employee performs the service). The service period is generally the
vesting period.

Example 13 Accounting for Stock Options

Facts: On January 1, Year 1, ABC Co. granted options exercisable after December 31, Year 2,
to purchase 10,000 shares of $5 par common stock for $25 per share. On the grant date,
the market price of the stock was $20 per share. Using an acceptable valuation model, the
options had a total fair value of $50,000. The options are to serve as compensation for
services during Year 1 and Year 2.
Required: Prepare the journal entries to account for the stock.
Solution:
January 1, Year 1:
No entry required.
December 31, Year 1 journal entry to allocate compensation cost to Year 1 operations:

DR Compensation expense $25,000


CR Additional paid-in capital—stock options $25,000

December 31, Year 2 journal entry to allocate compensation cost to Year 2 operations:

DR Compensation expense $25,000


CR Additional paid-in capital—stock options $25,000

On January 1, Year 3, all options are exercised. On the exercise date, the market price of the
stock was $35 per share.

January 1, Year 3 journal entry to record the exercise of the options:

DR Cash (10,000 × $25) $250,000


DR Additional paid-in capital—stock options 50,000
CR Common stock (10,000 × $5 par) $ 50,000
CR Additional paid-in capital in excess of par 250,000

1–186 Module 9 A.2. Revenue


© Becker Professional Recognition
Education and Income
Corporation. All rights Measurement
reserved.
PART 1 UNIT
9 1 A.2. Revenue Recognition and Income Measurement

2.4.13 Stock Appreciation Rights (SARs)


A stock appreciation right entitles an employee to receive an amount equal to the excess of the
market price of stock at the exercise date over a predetermined amount (usually market price at
grant date). This excess multiplied by the number of rights outstanding is recorded as compensation
expense and a liability. Compensation expense for stock appreciation rights outstanding must be
adjusted annually to account for changes in the market price of the stock. Unlike stock options, stock
appreciation rights do not require the employee to make a cash payment.

Example 14 Stock Appreciation Rights

Facts: On January 1, Year 1, Loud Corp. granted Mort, its president, 10,000 stock
appreciation rights expiring on January 3, Year 4. Upon exercise, Mort may receive cash
for the excess of market price of the stock on that date over the market price on the grant
date, and the service period runs for two years. Market prices were as follows:

January 1, Year 1 $30


December 31, Year 1 45
December 31, Year 2 40

Mort exercised all his stock appreciation rights on January 2, Year 3, when the market value
of Loud's stock was still $40.
Required: Prepare the journal entries that should be recorded by Loud to account for the
stock appreciation rights on December 31, Year 1; December 31, Year 2; and January 2, Year 3.
Solution:
December 31, Year 1:
Market price at December 31, Year 1 $ 45
Market price at January 1, Year 1 (30)
Appreciation in market value $ 15
Number of stock rights outstanding × 10,000
Total compensation expense $150,000

Compensation expense for Year 1 is then ½ x $150,000 because the service period is for
two years. Loud's journal entry at December 31, Year 1 follows:

DR Compensation expense $75,000


CR Liability for SAR plan $75,000

A liability account is credited because Mort will receive cash. Otherwise, paid-in capital
would be credited if stock is to be issued.

(continued)

© Becker Professional Education Corporation. All rights reserved. Module 9 1–187


A.2. Revenue Recogniti
9 A.2. Revenue Recognition and Income Measurement PART 1 UNIT 1

(continued)

December 31, Year 2:


Market price at December 31, Year 2 $ 40
Market price at January 1, Year 1 (30)
Appreciation in market value $ 10
Number of rights outstanding × 10,000
Required balance in liability, Year 2 $100,000

Compensation expense is $100,000 less the $75,000 previously recognized, or $25,000.


The required journal entry for Year 2 follows:

DR Compensation expense $25,000


CR Liability for SAR plan $25,000

January 2, Year 3:
When the stock appreciation rights are exercised, Loud would make the following entry:

DR Liability for SAR plan $100,000


CR Cash $100,000

LOS 1A2dd 3 Comprehensive Income

Comprehensive income is the change in equity (net assets) of a business enterprise during
a period from transactions and other events and circumstances from nonowner sources. It
includes all changes in equity during a period except those resulting from investments by
owners and distributions to owners.
Nonowner transactions include revenues, expenses, gains, and losses for the period. Gains and
losses are not always reported directly on the income statement of a company. Certain gains
and losses are excluded from the calculation of net income but are included in the calculation of
comprehensive income.

Net income

+ Other comprehensive income

Comprehensive income

Each entity can report comprehensive income (loss) with either a single statement of
comprehensive income or in two separate, consecutively presented statements.

1–188 Module 9 A.2. Revenue


© Becker Professional Recognition
Education and Income
Corporation. All rights Measurement
reserved.
PART 1 UNIT
9 1 A.2. Revenue Recognition and Income Measurement

3.1 Net Income (Loss)


Net income includes the following items:
1. Income from continuing operations
2. Discontinued operations

3.2 Other Comprehensive Income (Loss)


Other comprehensive income items are revenues, expenses, gains, and losses that are included
in comprehensive income but excluded from net income under U.S. GAAP and/or IFRS. An entity
must classify the specific items by their nature, such as:
Pension Adjustments
Under U.S. GAAP, changes in the funded status of a pension plan due to gains or losses,
prior service costs, and net transition assets or obligations must be recognized in other
comprehensive income in the year the changes occur. All gains or losses, prior service
costs, and transition assets or obligations are included in other comprehensive income until
recognized as a component of net periodic benefit cost.
Under IFRS, certain actuarial gains and losses may be included in other comprehensive
income. These gains and losses are not reclassified to net income in subsequent periods.
Unrealized Gains and Losses (Available-for-Sale Debt Securities)
The following types of unrealized gains and losses on certain investments in debt securities
are reported as components of other comprehensive income until the securities are sold.
yy Unrealized holding gains and losses on "available-for-sale debt securities."
yy Unrealized holding gains and losses that result from a debt security being transferred
into the "available-for-sale" category from "held to maturity."
yy Subsequent decreases or increases in the fair value of "available-for-sale" debt
securities previously written down as impaired.
Foreign Currency Items
Foreign currency translation adjustments and gains and losses on foreign currency
transactions that are designated as (and are effective as) economic hedges of a net
investment in a foreign entity are reported as a component of other comprehensive income.
Foreign currency translation adjustments remain in other comprehensive income until the
sale or liquidation of the investment in the foreign entity. Also, gains and losses on intra‑entity
foreign currency transactions that are of a long-term investment nature, when the entities to
the transaction are consolidated, are combined or accounted for by the equity method.
Instrument-Specific Credit Risk
For liabilities for which the fair value option is elected, changes in fair value that are
attributable to instrument-specific credit risk are included in comprehensive income.
Effective Portion of Cash Flow Hedges
The effective portion of a cash flow hedge is reported as a component of other
comprehensive income until the cash flows associated with the hedged item are realized.
Revaluation Surplus (IFRS Only)
Under IFRS, revaluation surpluses (gains) recognized when intangible assets and fixed assets
are revalued are also included in other comprehensive income. Revaluation surpluses are
not reclassified to net income in subsequent periods, but may be transferred directly to
retained earnings when the related asset is used or derecognized.

© Becker Professional Education Corporation. All rights reserved. Module 9 1–189


A.2. Revenue Recogniti
9 A.2. Revenue Recognition and Income Measurement PART 1 UNIT 1

3.3 Reclassification Adjustments


Reclassification adjustments move other comprehensive income items from accumulated
other comprehensive income to the income statement. Reclassifications impact current
year comprehensive income. If, for example, an item of unrealized gain is reclassified from
accumulated other comprehensive income to a realized gain reported in the income statement,
then the current year comprehensive income will be reduced in an amount equal to the amount
of realized gain reported in the income statement.

3.4 Accumulated Other Comprehensive Income


Accumulated other comprehensive income (AOCI) is a component of equity that includes the
total of other comprehensive income for the current period and all previous periods.

Pass Key

At the end of each accounting period, all components of comprehensive income are closed
to the balance sheet. Net income is closed to retained earnings, and other comprehensive
income is closed to accumulated other comprehensive income.

Example 15 Calculating Other Comprehensive Income (OCI)

Facts: Evelyn Enterprises' trial balance included the following items.

Foreign currency translation adjustment gain $6,200


Interest revenue 10,000
Cost of goods sold 85,000
Unrealized gain on trading debt securities 25,000
Sales revenue 255,000
Unrealized loss on AFS debt securities (15,000)
General and administrative expenses 40,000
Selling expenses 22,000
Actuarial loss on pension (8,000)

(continued)

1–190 Module 9 A.2. Revenue


© Becker Professional Recognition
Education and Income
Corporation. All rights Measurement
reserved.
PART 1 UNIT
9 1 A.2. Revenue Recognition and Income Measurement

(continued)

Required: If net income is $107,250, indicate whether each of the items listed is a
component of net income (NI) or other comprehensive income (OCI). Calculate other
comprehensive income and total comprehensive income on an after-tax basis. Assume a
tax rate of 25 percent.
Solution:

Foreign currency translation adjustment gain $6,200 OCI


Interest revenue 10,000 NI
Cost of goods sold 85,000 NI
Unrealized gain on trading debt securities 25,000 NI
Sales revenue 255,000 NI
Unrealized loss on AFS debt securities (15,000) OCI
General and administrative expenses 40,000 NI
Selling expenses 22,000 NI
Actuarial loss on pension (8,000) OCI

Foreign currency translation adjustment = $6,200 × (1 − 25%) = $ 4,650


Unrealized loss on AFS debt securities = $(15,000) × (1 − 25%) = (11,250)
Actuarial loss on pension = $(8,000) × (1 − 25%) = (6,000)
Total OCI $(12,600)

Comprehensive income = Net income + Other comprehensive income

Net income $107,250


Other comprehensive income (12,600)
Comprehensive income $ 94,650

3.4.1 Reclassification Adjustments


Reclassification adjustments move other comprehensive income items from accumulated
other comprehensive income to the income statement. Reclassifications impact current
year comprehensive income. If, for example, an item of unrealized gain is reclassified from
accumulated other comprehensive income to a realized gain reported in the income statement,
then the current year comprehensive income will be reduced in an amount equal to the amount
of realized gain reported in the income statement. Other comprehensive income is closed to
accumulated other comprehensive income just as net income is closed to retained earnings.

© Becker Professional Education Corporation. All rights reserved. Module 9 1–191


A.2. Revenue Recogniti
9 A.2. Revenue Recognition and Income Measurement PART 1 UNIT 1

LOS 1A2ee 4 Discontinued Operations

Discontinued operations are reported separately from continuing operations in the income
statement, net of tax. A discontinued operation may include a component of an entity, a
group of components of an entity, or a business or nonprofit activity. Items reported within
discontinued operations can consist of an impairment loss, a gain or loss from actual operations,
and a gain or loss on disposal. All of these amounts are included in discontinued operations in
the period in which they occur.

4.1 Definitions
4.1.1 Component of an Entity
A component of an entity is a part of an entity (the lowest level) for which operations and cash
flows can be clearly distinguished, both operationally and for financial reporting purposes, from
the rest of the entity.
According to U.S. GAAP, a component can refer to:
an operating segment (as defined in segment reporting);
a reportable segment (as defined in segment reporting);
a reporting unit (as defined in goodwill impairment testing);
a subsidiary; or
an asset group (a collection of assets to be disposed of together as a group in a single
transaction and the liabilities directly associated with those assets that will be transferred in
that same transaction).

4.1.2 Business
A business is an integrated set of activities and assets that is capable of being conducted and
managed for the purpose of providing a return in the form of dividends, lower costs, or other
economic benefits directly to investors or other owners, members, or participants.

4.1.3 Nonprofit Activity


A nonprofit activity is an integrated set of activities and assets that is conducted and managed
for the purpose of providing benefits, other than goods or services at a profit, to fulfill an entity's
purpose or mission.

4.1.4 Held for Sale


A component of a business is classified as held for sale in the period in which all of the following
criteria are met:
1. Management commits to a plan to sell the component.
2. The component is available for immediate sale in its present condition.
3. An active program to locate a buyer has been initiated.
4. The sale of the component is probable and the sale is expected to be complete within
one year. (There are limited exceptions to this one-year rule, when certain events and
circumstances occur that are beyond the control of the entity.)

1–192 Module 9 A.2. Revenue


© Becker Professional Recognition
Education and Income
Corporation. All rights Measurement
reserved.
PART 1 UNIT
9 1 A.2. Revenue Recognition and Income Measurement

5. The sale of the component is being actively marketed.


6. Actions required to complete the sale make it unlikely that significant changes to the plan
will be made or that the plan will be withdrawn.
When a component is classified as held for sale, an impairment analysis of the component must
be conducted.

4.2 Accounting Rules


4.2.1 Types of Entities to Be Considered
The results of operations of a component of an entity or a group of components of an entity, or
a business or nonprofit activity, will be reported in discontinued operations if it:
has been disposed of; or
is classified as held for sale.

4.2.2 Conditions That Must Be Present


All related costs shall be recognized when the obligations to others exist, not necessarily in
the period of commitment to a plan. A disposal of a component or group of components is
reported in discontinued operations if the disposal represents a strategic shift that has or will
have a major effect on an entity's operations and financial results.
Examples of a strategic shift that could have a major effect on operations and financial results
may include, among others:
Disposal of a major geographical area
Disposal of a major equity method investment
Disposal of a major line of business
A business or nonprofit activity that, on acquisition, meets the criteria to be classified as held for
sale is a discontinued operation.

Illustration 2 Discontinued Operation

Am-Serv Inc. is a food service company that delivers frozen food products to food service
providers. Its clients include fast-food restaurants, high-end steak houses, home-delivery
diet food companies, and institutions such as schools and hospitals. Historically, the
fast‑food restaurants have been the largest segment of Am-Serv's business in terms of
revenue and operating profit. However, that division is now forecast to begin to decline in
revenues because the public is looking for healthier options. Am-Serv has decided to sell its
fast-food operation and, instead, focus on selling to locally operated restaurants offering
a healthier fare. Such restaurants will generally show lower revenue per unit, but higher
operating profit per unit. Because the fast-food division is the largest component in terms
of revenue and operating profit, the disposal represents a major strategic shift, and will be
reported as a discontinued operation.

© Becker Professional Education Corporation. All rights reserved. Module 9 1–193


A.2. Revenue Recogniti
9 A.2. Revenue Recognition and Income Measurement PART 1 UNIT 1

4.3 Discontinued Operations Calculation


4.3.1 Types of Items Included in Results of Discontinued Operations
Results of operations of the component.
Gain or loss on disposal of the component.
Impairment loss (and subsequent increases in fair value) of the component.
yy Initial and Subsequent Impairment Losses
A loss is recognized for recording the impairment of the component (i.e., any initial or
subsequent write-down to fair value less costs to sell).
yy Subsequent Increases in Fair Value
A gain is recognized for any subsequent increase in fair value minus the costs to sell
(but not in excess of the previously recognized cumulative loss).

4.3.2 Report in the Period Disposed of or Held for Sale


The results of discontinued operations of a component are reported in discontinued operations
(for the current period and for all prior periods presented) in the period the component is either
disposed of or is held for sale. The results of subsequent operations of a component classified
as held for sale are reported in discontinued operations in the period in which they occur.

4.3.3 Depreciation and Amortization


Once management decides to dispose of the component, assets within the component are no
longer depreciated or amortized.

4.4 Anticipated Future Gains or Losses


Losses anticipated to occur in future periods are considered when impairment analysis is
conducted on the component. Gains anticipated to occur in future periods are not recognized
until they occur. A gain or loss not previously recognized that results from the sale of the
component is recognized at the date of sale and not before.

4.5 Measurement and Valuation


A component classified as held for sale is measured at the lower of its carrying amount or fair
value less costs to sell. Costs to sell are the incremental direct costs to transact the sale.

4.6 Presentation and Disclosure


Present as a Separate Component of Income: The results of discontinued operations,
net of tax, are reported as a separate component of income, below income from
continuing operations.
Disclose in Face or in Notes: A gain or loss recognized on the disposal shall be disclosed
either on the face of the income statement or in the notes to the financial statements.

1–194 Module 9 A.2. Revenue


© Becker Professional Recognition
Education and Income
Corporation. All rights Measurement
reserved.
PART 1 UNIT
9 1 A.2. Revenue Recognition and Income Measurement

Discontinued Operations Calculations


Example 16
and Income Statement Presentation

Facts: The trial balance below presents the income statement accounts for Year 1 from
All Sports Company's trial balance. The golf division of All Sports has been losing money on
a monthly basis. The golf division's income statement accounts are also presented below.
The board of directors decides on April 30, Year 1, to dispose of the golf division. The
carrying value of the golf division on April 30, Year 1, is $4,000,000, and its fair value less
costs to sell is $2,200,000. After months of negotiations, the division's net assets are sold on
June 30, Year 2, for $2,000,000. The golf division has continuing losses in Year 2 of $200,000
per month. All Sports' income tax rate is 40 percent for Years 1 and 2. Assume that All
Sports' income from continuing operations is $4,875,000 in Year 1 and $5,200,000 in Year 2.

All Sports Company Trial Balance


Year 1

All Components Golf Division


4000 Sales revenue $21,000,000 $2,500,000
5000 Cost of goods sold $9,500,000 1,850,000
5300 Freight out 155,000 135,000
5400 Commissions 900,000 220,000
5500 Advertising 1,200,000 600,000
5600 Insurance expense 1,400,000 750,000
5700 Salaries expense 2,500,000 850,000
5800 Depreciation expense 950,000 495,000
6000 Interest revenue 750,000
6200 Other revenue 300,000
6500 Gain on sale of assets 400,000
7000 Interest expense 120,000
7500 Impairment loss 1,800,000 1,800,000

Required: How should the disposal of the golf division be reported on All Sports
Company's Year 1 and Year 2 financial statements?

(continued)

© Becker Professional Education Corporation. All rights reserved. Module 9 1–195


A.2. Revenue Recogniti
9 A.2. Revenue Recognition and Income Measurement PART 1 UNIT 1

(continued)

Solution:
Reporting for Year 1:
The golf division was not disposed of until Year 2 and would be reported as held for sale in
the Year 1 financial statements.
The continuing loss from the golf division would be included in discontinued operations in
Year 1.
Loss from operations = $2,500,000 − ($1,850,000 + $135,000 + $220,000 + $600,000
+ $750,000 + $850,000 + $495,000) = ($2,400,000).
Loss from operations, net of tax = ($2,400,000) × (1 − 40%) = ($1,440,000)
Impairment loss = $2,200,000 − $4,000,000 = ($1,800,000)
Impairment loss, net of tax = ($1,800,000) × (1 − 40%) = ($1,080,000)
Income statement presentation Year 1:
Income from continuing operations $4,875,000
Discontinued operations
Loss from operations of discontinued component, net of tax (1,440,000)
Loss from impairment of discontinued operations, net of tax (1,080,000)
Net income $2,355,000

Reporting for Year 2:


In Year 2, discontinued operations would include the continuing losses incurred before the
sale and the loss on disposal.
Loss from operations (January 1 − June 30) = ($200,000) × 6 = ($1,200,000)
Loss from operations, net of tax = ($1,200,000) × (1 − 40%) = ($720,000)
Loss on disposal = $2,000,000 − $2,200,000 = ($200,000)
Loss on disposal, net of tax = ($200,000) × (1 − 40%) = ($120,000)
Income statement presentation Year 2:
Income from continuing operations $5,200,000
Discontinued operations
Loss from operations of discontinued component, net of tax (720,000)
Loss from sale (disposal) of discontinued operations, net of tax (120,000)
Net income $4,360,000

1–196 Module 9 A.2. Revenue


© Becker Professional Recognition
Education and Income
Corporation. All rights Measurement
reserved.
PART 1 UNIT
9 1 A.2. Revenue Recognition and Income Measurement

Question 1 MCQ-12413

The Thompson Toy Co. manufactures toys and has a division that creates custom doll
houses for clients. As the result of a change in its strategic focus, the company sold the
custom doll house division at year-end. The sale of the division will have a major effect on
the company's operations and financial results. How should Thompson Toy Co. report the
sale in its income statement?
a. Report it as part of other comprehensive income.
b. Report it as a discontinued operation, reported below income from continuing
operations.
c. Report the income or loss from operations of the division in discontinued
operations below continuing operations and the gain or loss from disposal in
continuing operations.
d. Report as a component of income from continuing operations.

Question 2 MCQ-12414

Katie has signed up and paid $960 for a three-month introduction to cake decorating
class at Bakers R Us. The class begins August 1. Bakers R Us has a September 30 year-end.
Indicate the amounts reflected in Bakers' financial statements associated with the class.
a. $960 of service revenue, $960 cash
b. $640 service revenue, $320 deferred revenue
c. $320 of service revenue, $960 cash, and $640 deferred revenue
d. $0 service revenue, $960 deferred revenue

Question 3 MCQ-12415

Jones and Hill Consulting enters into a contract for consulting with BGSE Inc. The
contract indicates payments of $2,000 per month for one year of consulting services with
approximately eight hours per month spent with BGSE Inc. Included in the contract is a
$5,000 bonus depending on specified objectives. Jones and Hill estimates that there is an
80 percent probability that the company will receive the bonus.
Assume that Jones and Hill uses the expected value to determine variable consideration in
the contract. What amount of revenue will Jones and Hill recognize in the first six months of
the contract?
a. $12,000
b. $14,000
c. $14,500
d. $28,000

© Becker Professional Education Corporation. All rights reserved. Module 9 1–197


A.2. Revenue Recogniti
9 A.2. Revenue Recognition and Income Measurement PART 1 UNIT 1

NOTES

1–198 Module 9 A.2. Revenue


© Becker Professional Recognition
Education and Income
Corporation. All rights Measurement
reserved.
Class Question Explanations Part 1

UNIT 1

Unit 1, Module 2

1. MCQ-12325
Choice "c" is correct. The presentation of significant subtotals on an income statement can
facilitate a more meaningful analysis of income reported by a company. Operating income is
a measure of profitability based on a company's core, or normal operations. This line item is
beneficial in assessing future income from a company because it focuses exclusively on the
company's earnings from normal operations.
Lewis Industries operating income would be $250,000: $800,000 service revenues less $310,000
cost of services less $240,000 administrative expenses.
Choice "a" is incorrect. The $235,000 incorrectly includes the $15,000 loss on sale of investment
in the calculation of operating income. The loss would be included in the other income/(expense)
section of the income statement and of the calculation of income from continuing operations.
Choice "b" is incorrect. The $246,000 incorrectly includes the $15,000 loss on sale of investment,
$8,000 interest income, and $7,000 interest expense in the calculation of operating income.
The loss on sale of investment, interest revenue, and interest expense would be included in
the other income/(expense) section of the income statement and included in the calculation of
income from continuing operations.
Choice "d" is incorrect. The $500,000 represents gross profit for Lewis Industries. Gross profit is
the difference between service revenue and cost of sales. The question specifically asks for the
calculation of operating income so administrative expenses must be included.

2. MCQ-12328
Choice "d" is correct. The primary revenue-generating activities are reported as operating
items on the income statement. The primary revenue-generating activities represent the core
mission of the organization and the resulting revenues and expense items associated with
these activities.
The classification of primary revenue and expense items as operating revenues and operating
expenses allows users of the income statement to analyze profitability from the perspective of
core business operations.
Choice "a" is incorrect. Primary revenue-generating activities are operational revenues and
expenses. Classifying these types of activities as nonoperating would be incorrect.
Choice "b" is incorrect. Primary revenue-generating activities are included and reported on
the income statement. These activities represent utilization of assets in the operations of the
company and the resulting revenues and expenses.
Choice "c" is incorrect. Primary revenue-generating activities are included and reported on
the income statement, not the balance sheet. Although current assets are often utilized in the
generation of revenues and expenses, the changes in those assets are articulated through
revenue and expense reporting.

© Becker Professional Education Corporation. All rights reserved. CQ–3


Part 1 Class Question Explanations

3. MCQ-12333
Choice "b" is correct. The total current assets are those that are expected to be converted to
cash or to be consumed within one year of the balance sheet date or within the company's
operating cycle, whichever is longer. Assets consumed outside of one year or the operating cycle
would not be classified as current on the balance sheet.
Cash, inventory, and accounts receivable are all current assets held by a corporation. These
assets are typically utilized during the operating cycle.
Total current assets = $24,500: $4,500 cash + $12,000 inventory + $8,000 accounts receivable.
Choice "a" is incorrect. The amount indicated in the calculation of total current assets includes
dividends of $12,000. Dividends declared reduce retained earnings when declared by the
company's board of directors. Dividends declared are not included with current assets on the
balance sheet.
Choice "c" is incorrect. The amount indicated includes equipment in the calculation of total
current assets. On the balance sheet, equipment would be reported with property, plant, and
equipment, and not as a current asset.
Choice "d" is incorrect. The amount indicated includes equipment and buildings in the
calculation of total current assets. On the balance sheet, equipment and buildings would be
reported with property, plant, and equipment, and not as a current asset.

4. MCQ-12340
Choice "c" is correct. Changes in stockholders' equity have two major components: contributed
capital and retained earnings. Contributed capital represents contributions made by owners
to the company. Retained earnings represent cumulative net income retained by the company
since inception. The retained earnings account is increased by each year's net income and
decreased by dividends and/or net losses during the period.
The total increase in stockholders' equity can be calculated as the difference between the
beginning and ending stockholders' equity balance or $57,000 at the end of the period less
$25,000 at the beginning of the period, or $32,000. The increase in equity attributable to
retained earnings would be $22,000. Stock issuances of $10,000 during the period would not
be associated with retained earnings. The net increase in retained earnings is the difference
between net income and dividends.
Net income less dividends = Increase in retained earnings
X – $7,000 = $22,000
Therefore, net income = $29,000.
Choice "a" is incorrect. The change in total stockholders' equity does not equal net income for
the company. Stock issuances and dividends affect the stockholders' equity balance and must be
factored into the calculation to determine net income.
Choice "b" is incorrect. Retained earnings net increase is affected by both net income and
dividends. The 22,000 does not factor in the dividends when analyzing the net increase to
retained earnings to determine net income.
Choice "d" is incorrect. Sufficient information is provided in order to calculate net income for the
company. The total change in equity is provided along with changes in contributed capital and
information related to dividends declared during the period.

CQ–4 © Becker Professional Education Corporation. All rights reserved.


Class Question Explanations Part 1

5. MCQ-12338
Choice "b" is correct. The financial statements prepared by an entity describe relationships
between the financial statements. The beginning balances on the balance sheet and resulting
ending balances are described through the income statement, statement of stockholders'
equity, and cash flow statement. The change in retained earnings due to income/loss reported is
determined on the income statement of an entity.
The equity accounts reflected on the balance sheet and the increases and decreases during the
period are reconciled on the statement of stockholders' equity to determine causes of changes
in ending balances. The statement of cash flows dissects the increase or decrease in cash
reflected on the balance sheet.
Choice "a" is incorrect. The financial statements are not unrelated to each other. Each financial
statement serves to provide insights into the changes experienced on the balance sheet of an
entity each period.
Choice "c" is incorrect. The balance sheet, income statement, statement of stockholders' equity,
and cash flow statement are each connected.
Choice "d" is incorrect. The changes in the balance sheet are explained on each of the other
financial statements presented.

Unit 1, Module 3

1. MCQ-12349
Choice "d" is correct. The statement of cash flows reflects all cash inflows and outflows for a
given period. There are three categories of cash flows: operating, investing, and financing. Cash
flow from operations reflects the amount of cash generated from, and used for, core operating
activities for an entity. All else equal, having positive cash flows from operations reflects more
positively on the entity.
Equity trading securities are current assets and selling them will generate cash. Proceeds from
the sale subsequently used to purchase held-to-maturity debt securities (which are non-current
assets) are treated as cash outflows from investing. The overall effect on cash will net to zero,
but the transaction will produce positive cash flows from operations and negative cash flows
from investing.
Choice "a" is incorrect. Stock issuances are cash inflows from financing (not operations).
Choice "b" is incorrect. The allowance for doubtful accounts is a contra-asset that serves to
reduce net accounts receivable. Reducing this allowance will increase net accounts receivable,
which is a current asset. Increases in current assets are treated as reductions in cash flows from
operations. Furthermore, decreasing the allowance account (a debit to the allowance account)
will require a credit to the bad debt expense account, which will then increase net income. So,
the dollar increase to net income (the "starting point" for determining the change in cash from
operations) will be equal to the dollar reduction in cash flow attributable to the increase in net
accounts receivable; the net effect on the statement of cash flows from operations is 0.
Choice "c" is incorrect. Long-term debt transactions are categorized as cash flows from financing
rather than from operations.

© Becker Professional Education Corporation. All rights reserved. CQ–5


Part 1 Class Question Explanations

2. MCQ-12352
Choice "a" is correct. The statement of cash flows can be prepared using information from
the balance sheet and income statement. Cash flow from operations is calculated using
either the direct or indirect method. The indirect method calculation begins with net income,
adds depreciation, adds any losses, subtracts any gains, and then incorporates changes in
current assets and liabilities. Increases in current assets and decreases in current liabilities are
reductions in cash, whereas decreases in current assets and increases in current liabilities are
increases to cash.
The calculation for cash flow from operations is shown below:

Line Item Amount Notes


Net income 220,000
Depreciation 35,000 Added back as a noncash expense
Gain on the sale of fixed assets –18,000 Subtracted as noncash revenue
Accounts receivable (change) –4,000 Increase from 35,000 to 39,000
Accounts payable (change) –3,000 Decrease from 22,000 to 19,000
Inventory (change) 4,800 Decrease from 84,500 to 79,700
Interest payment 0 65,000 was paid from a par value bond, but
this will exactly equal interest expense (which is
already factored into net income)
Principal payment 0 125,000 is a cash outflow from financing
Long-term liability (change) 0 Increase from 45,000 to 50,000 is not counted
because these are long-term (non-current) liabilities
Trading securities (change) 4,500 Decrease from 29,400 to 24,900
Total 239,300

Choice "b" is incorrect. The gain on the sale of fixed assets should be subtracted, not added.
Choice "c" is incorrect. The interest payment of 65,000 does not need to be added back to the
total, as it appropriately represents a cash outflow and is already accounted for in net income.
Choice "d" is incorrect. This answer choice incorrectly incorporates both the interest payment as
an add-back and the change in the long-term liability (which is non-current).

3. MCQ-12354
Choice "c" is correct. An integrated report is a concise communication to stakeholders about
how an organization's strategy, governance, performance and prospects, in the context of its
external environment, lead to the creation of value over the short-, medium- and long-term.
All of the items listed above are potentially reported in an integrated report except for the
variance analysis reports used by management for internal decision making. IR has an external
reporting focus.
Choice "a" is incorrect. Annual financial statements are included in integrated reports.
Choice "b" is incorrect. Management discussion and analysis are included in integrated reports.
Choice "d" is incorrect. A description of the external environment and its effect on strategy is
included in integrated reports.

CQ–6 © Becker Professional Education Corporation. All rights reserved.


Class Question Explanations Part 1

4. MCQ-12356
Choice "d" is correct. Outputs are the goods and services produced by the organization.
Outcomes are the internal consequences and the external consequences (whether positive or
negative) with respect to the six capitals (inputs); outputs and outcomes result from the various
business activities.
By definition, outcomes are the achievements that occurred because of what the company produces.
Choice "a" is incorrect. It is true that it is not easy to measure outcomes, but it is possible to
identify a quantitative measure.
Choice "b" is incorrect. The measurement methods for outputs and outcomes are different, and
the terms are not the same.
Choice "c" is incorrect. Outcomes do not represent quantities produced.

5. MCQ-12347
Choice "b" is correct. Outputs are the goods and services produced by the organization.
Outcomes are the internal consequences and the external consequences (whether positive or
negative) for the six capitals (inputs) resulting from the various business activities.
The number of students served reflects an output (a quantifiable number) but not an outcome.
Choice "a" is incorrect. The employees' learning of new processes is an outcome.
Choice "c" is incorrect. The improvement of students' health is an outcome.
Choice "d" is incorrect. Students achieving better in school is an outcome.

© Becker Professional Education Corporation. All rights reserved. CQ–7


Part 1 Class Question Explanations

Unit 1, Module 4

1. MCQ-12358
Choice "d" is correct. Infrequently, a receivable that has been written off is later collected. The
subsequent recovery of an account previously written off requires a reversal of the initial entry
recorded to reinstate the receivable and allowance. Another entry is necessary to show the
collection of cash and reduce the accounts receivable.
Examples can be a powerful way to visualize the effects of these changes. Assume, a subsequent
cash collection of $10,000 for an account previously written off.
Reverse write-off
Accounts receivable $10,000
Allowance for uncollectible accounts $10,000
Re-establish the accounts receivable and the corresponding allowance.
Collection of cash
Cash $10,000
Accounts receivable $10,000
Receive cash and reduce amount owed by customer.
The net effect is accounts receivable is increased and decreased by $10,000, resulting in no
effect on the accounts receivable balance. The allowance account increases when the reversal of
the previously recorded write-off was recorded.
Choice "a" is incorrect. The effect of subsequent collections of accounts receivable results in
both an increase and decrease of equal amounts resulting in no change in the account balance.
The allowance account is increased as a result of subsequent cash collections for accounts
previously written off.
Choice "b" is incorrect. The effect of subsequent collections of accounts receivable results in
both an increase and decrease of equal amounts resulting in no change in the account balance.
The allowance account is increased as a result of subsequent cash collections for accounts
previously written off not decreased.
Choice "c" is incorrect. The effect of subsequent collections of accounts receivable results in
both an increase and decrease of equal amounts resulting in no change in the account balance.
However, the allowance account is increased because of subsequent cash collections for
accounts previously written off.

CQ–8 © Becker Professional Education Corporation. All rights reserved.


Class Question Explanations Part 1

2. MCQ-12361
Choice "a" is correct. The accounts receivable account is affected by sales made on credit to
customers, cash collections, write-offs and the conversion of an account receivable into a note
receivable. At times questions will require candidates to back into an amount using knowledge
of increases and decreases within the account.

Accounts Receivable
Date Debit Credit
Beg. balance X
Credit sales 1,200,000
1,150,000 Cash collections
25,000 Write-offs
Change in AR 25,000
End. balance 35,000

The balance in the accounts receivable account based on activity provided is a $25,000 debit or
increase to the receivable. If the ending balance of accounts receivable is $35,000 (given), the
beginning balance can be calculated as $10,000 (X + $25,000 = $35,000).
Choice "b" is incorrect. The $25,000 indicated reflects the net increase to the receivable account
based on activity during the period.
Choice "c" is incorrect. Because accounts receivable has a normal debit balance, this answer
cannot be correct unless the beginning balance was a $15,000 credit or if write-offs from the
analysis of accounts receivable were incorrectly excluded.
Choice "d" is incorrect. This amount, $35,000, is the ending balance given. The beginning balance
must be derived by working backwards through the information given.

3. MCQ-12364
Choice "d" is correct. The gross method records a sale without regard to the available discount.
If payment is received within the discount period, a sales discount (contra-revenue) account
is debited to reflect the sales discount with a corresponding credit to accounts receivable to
reduce the value to the amount collected.
The cash payment received from the customer would be the initial sale of $9,500 less the return
of goods worth $2,000 or accounts receivable balance of $7,500 less the 2 percent discount or
98 percent of the receivable (1 – 0.02).
$7,500 × 98% = $7,350 cash collected
Choice "a" is incorrect. The $9,500 amount ignores both the return of $2,000 worth of goods by
the customer and does not factor the 2 percent discount offered for payment within 10 days.
Choice "b" is incorrect. The $9,310 amount ignores both the return of $2,000 worth of goods by
the customer and applies the discount to the initial sales amount of $9,500. The customer would
not pay for returned goods when remitting payment back to Sowell.
Choice "c" is incorrect. The $7,500 amount factors in the return of $2,000 worth of goods by the
customer but does not factor the 2 percent discount offered for payment within 10 days.

© Becker Professional Education Corporation. All rights reserved. CQ–9


Part 1 Class Question Explanations

4. MCQ-12366
Choice "d" is correct. In order for a company to account for the transfer of receivables as a
sale, the most critical element is the extent to which the company has transferred control of
the assets to the factor. In order to demonstrate the surrender of control, all of the following
conditions must be present:
yyTransferred assets are beyond the reach of the transferor and its creditors.
yyThe transferor surrenders control of the future economic benefits of the receivables to the
factor (the buyer).
yyThe transferor cannot be required to repurchase the receivables but may be required to
replace the receivables with other similar receivables.
The transferor must surrender control of the assets to account for the transaction as a sale. This
element is crucial to determine whether a sale has taken place or a secured borrowing.
Choice "a" is incorrect. The transferee, not the transferor, has the right to pledge or exchange
the receivables received.
Choice "b" is incorrect. The critical element in accounting for the financing of receivables as a
sale is the transfer of control.
Choice "d" is incorrect. A surrender of control is demonstrated by the lack of access to
transferred assets by the transferor and its creditors.

Unit 1, Module 5

1. MCQ-12381
Choice "c" is correct. The FIFO inventory valuation method assumes the first goods purchased
are the first goods sold. With respect to the determination of cost of goods sold, the oldest
inventory layers are always fully depleted before moving to the next layer of inventory. Because
the oldest inventory layer is always the first layer to transfer to, and constitute, cost of goods
sold, the perpetual inventory system and periodic inventory system will always produce the
same cost of goods sold and the same ending inventory.
Choice "a" is incorrect. Periodic average cost will calculate the cost of goods sold and ending
inventory by calculating an average cost per unit: total purchases of inventory divided by total
number of units. This average is used to allocate cost between cost of goods sold and ending
inventory. This method is only used under a periodic inventory system.
Choice "b" is incorrect. The LIFO inventory method assumes the most recently acquired goods
are the first ones sold. The periodic inventory system and the perpetual LIFO inventory system
will not produce the same cost of goods sold; the periodic inventory system and the perpetual
LIFO inventory system will not produce the same cost of ending inventory. LIFO periodic will
assume that the cost of ending inventory is from the cost of oldest inventory layers held by the
company, whereas LIFO perpetual might result in a different cost of ending inventory.
Choice "d" is incorrect. The perpetual weighted average will calculate cost of goods sold
using a weighted average cost per unit of inventory held at the point of a sale. The weighted
average cost per unit is the inventory costs divided by units purchased. After each purchase of
inventory, a new average cost is calculated and used to allocate costs between cost of goods
sold and ending inventory for sales of goods. This method is only used under a perpetual
inventory system.

CQ–10 © Becker Professional Education Corporation. All rights reserved.


Class Question Explanations Part 1

2. MCQ-12382
Choice "d" is correct. During a period of declining prices, the method of inventory valuation
will produce varying effects on the financial statements. When prices are declining, the FIFO
inventory method will produce the highest cost of goods sold; LIFO will produce the lowest cost
of goods sold; and average cost will remain in the middle of the two.
Period of Falling Prices
FIFO LIFO Weighted Average
Income statement: Highest Lowest Middle
Cost of goods sold
Income statement: Lowest Highest Middle
Net income
Balance sheet: Lowest Highest Middle
Inventory

Cost of goods sold will be lowest under the LIFO method of inventory when prices are declining.
Choice "a" is incorrect. During a period of declining prices, LIFO will produce the lowest cost of
goods sold, and FIFO will produce the highest cost of goods sold.
Choice "b" is incorrect. During a period of declining prices, average cost will be lower than FIFO
but higher than LIFO.
Choice "c" is incorrect. During a period of declining prices, LIFO will produce the lowest cost of
goods sold, and FIFO will produce the highest cost of goods sold.

3. MCQ-12383
Choice "c" is correct. Under FIFO, the first costs inventoried are the first costs transferred to cost
of goods sold. Ending inventory includes the most recently incurred costs; therefore, the ending
balance approximates replacement cost. Ending inventory and cost of goods sold are the same
regardless of the inventory system (periodic or perpetual) used.
Dawson's sales of 65 units would be attributable to the oldest inventory layers first.

Sale
February 1 25 units × $110/unit = $2,750
Total COGS—Feb. 1 sale $2,750

March 15 15 units × $110/unit = $1,650


25 units × $115/unit = $2,875
Total COGS—March 15 sale $4,525

Total COGS for all sales $7,275

Choice "a" is incorrect. The $7,350 amount was calculated using the LIFO perpetual inventory
method: [25 units × $110/unit] + [40 units × $115/unit]. The question asks for cost of goods sold
using FIFO perpetual.
Choice "b" is incorrect. The $8,125 amount was calculated using the LIFO periodic inventory
method: 65 units × $125/unit. The question asks for cost of goods sold using FIFO perpetual.
Choice "d" is incorrect. The $7,679 amount was calculated using the periodic average cost
inventory method: {[(40 units × $110/unit) + (60 units × $115/unit) + (75 units × $125/unit)]/175
units} × 65 units sold. The question asks for cost of goods sold using FIFO perpetual.

© Becker Professional Education Corporation. All rights reserved. CQ–11


Part 1 Class Question Explanations

4. MCQ-12384
Choice "a" is correct. Under LIFO, the most recent purchases are the first costs transferred to
cost of goods sold. Ending inventory, therefore, is comprised of the oldest inventory costs. As
a result, the ending balance of inventory on the balance sheet likely does not approximate
replacement cost.
Dawson's sales of inventory would be attributable to the most recently acquired inventory.

Sale
February 1 25 units × $110/unit = $2,750
Total COGS—Feb. 1 sale $2,750

March 15 40 units × $115/unit = $4,600


Total COGS—March 15 sale $4,600

Total COGS for all sales $7,350

Choice "b" is incorrect. The $8,125 amount was calculated using the LIFO periodic inventory
method: 65 units × $125/unit. The question asks for cost of goods sold using LIFO perpetual.
Choice "c" is incorrect. The $7,275 amount was calculated using the FIFO perpetual inventory
method: (25 units × $110/unit) + (15 units × $110/unit) + (25 units × $115/unit). The question asks
for cost of goods sold using LIFO perpetual.
Choice "d" is incorrect. The $7,679 was calculated using the periodic average cost inventory
method: {[(40 units × $110/unit) + (60 units × $115/unit) + (75 units × $125/unit)]/175 units} × 65
units sold. The question asks for cost of goods sold using LIFO perpetual.

5. MCQ-12386
Choice "a" is correct. Inventory errors include the overstatement of ending inventory or the
understatement of ending inventory due to a mistake in the physical count and/or a mistake in
pricing inventory. If management discovers the error during the year of the error, management
simply corrects inventory with the appropriate journal entry (debit or credit inventory, and credit
or debit cost of goods sold). When the inventory error is discovered and corrected in the same
reporting period, there are no financial statement errors. If an error related to inventory goes
undetected, the error will correct itself by the end of the second year.

Year 1 Year 2
Beginning inventory No error Overstated by $6,000
Ending inventory Overstated by $6,000 No error
Cost of goods sold Understated by $6,000 Overstated by $6,000
Gross profit Overstated by $6,000 Understated by $6,000
Net income Overstated by $6,000 Understated by $6,000

If the ending inventory for BGSE is overstated at the end of Year 1, the cost of goods sold on
the income statement during Year 1 is understated, and gross profit and net income are both
overstated. In Year 2, the inventory error will reverse. If ending inventory is overstated at the end
of Year 1, the beginning inventory for Year 2 will be overstated as well. If beginning inventory is
overstated, cost of goods sold for that year is overstated, and gross profit and net income would
both be understated for that year.

CQ–12 © Becker Professional Education Corporation. All rights reserved.


Class Question Explanations Part 1

Choice "b" is incorrect. If beginning inventory is overstated by $6,000 in Year 2, then the cost
of goods sold is overstated for that year. However, gross profit and net income would both be
understated for that year.
Choice "c" is incorrect. If ending inventory is overstated at the end of Year 1, the beginning
inventory in Year 2 is also overstated by $6,000. The Year 2 cost of goods sold will be overstated,
and the Year 2 gross profit and net income will both be understated.
Choice "d" is incorrect. If ending inventory is overstated at the end of Year 1, the beginning
inventory in Year 2 is also overstated by $6,000. The Year 2 cost of goods sold will be overstated,
and the Year 2 gross profit and net income will both be understated.

Unit 1, Module 6

1. MCQ-12392
Choice “d” is correct. Investment reporting on the balance sheet is determined by the type of
investment purchased by the business. Available-for-sale (AFS) investments are reported at
fair value at the end of the reporting period with unrealized gains and losses recorded in other
comprehensive income. Trading securities are reported at fair value at the end of the reporting
period with unrealized gains and losses recorded on the income statement. Held-to-maturity
(HTM) investments are reported at amortized cost at the end of each reporting period. No
adjustments are made to these investments for changes in fair value.

Fair Value
Available-for-sale securities
XYZ bonds $225,000
BGSE bonds $75,000

Trading securities
ESGB bonds $107,000
Dawson investments $118,000

Amortized Cost
Held-to-maturity securities
BK bonds $415,000
GHHKG bonds $314,000
Total investments $1,254,000

Choice “a” is incorrect. The $1,237,000 records the AFS securities at initial cost of $217,000
and $65,000 and does not adjust for changes in fair value at the end of Year 2. Trading
securities were appropriately valued at fair value at the end of year or $107,000 and $118,000,
respectively. HTM investments were recorded at fair value instead of amortized cost.
Choice “b” is incorrect. The $1,255,000 amount records all debt investments at fair value at the
end of Year 2. The only investments that would be recorded at fair value are AFS and trading
securities. HTM investments would be reported at amortized cost.
Choice “c” is incorrect. The $1,220,000 amount records both the AFS and trading securities
at initial cost and the HTM securities are appropriately recorded at amortized cost. AFS and
trading securities would be recorded at fair value at the end of Year 2, not the initial cost of
the securities.

© Becker Professional Education Corporation. All rights reserved. CQ–13


Part 1 Class Question Explanations

2. MCQ-12394
Choice “b” is correct. Debt investments classified as trading securities are reported at fair
value at the end of each reporting period. Any fluctuations in value are reported as unrealized
gains and losses on the income statement. These investments are adjusted to fair value on
the balance sheet, and unrealized holding gains and losses are included in the net income
calculation on the income statement.

Fair Value Cost


GWD investments $ 75,000 $ 67,000
HCD investments 79,000 55,000
Total $154,000 $122,000
Change in fair value $32,000 ($154,000 less $122,000)
Previously recorded gain in Year 1 18,500
Year 2 gain $13,500

Choice “a” is incorrect. The $32,000 gain represents the total change in fair value of the
investments since purchase. The gain related to Year 1 was recorded and reported in the
determination of the Year 1 net income. The increase in value from Year 1 to Year 2 is the only
amount to be recorded in Year 2.
Choice “c” is incorrect. The $18,500 gain represents the unrealized gain associated with Year 1.
The question specifically asks for unrealized gain associated with Year 2.
Choice “d” is incorrect. Changes in value of trading securities are reported on the
income statement.

3. MCQ-12396
Choice “b” is correct. Book value of an asset is determined by taking cost less accumulated
depreciation. Depreciation on tangible fixed assets is recorded by utilizing a contra-asset
account called accumulated depreciation instead of reducing the cost basis of the asset itself.
Each year an entry is made to record depreciation expenses associated with the current period
based on estimated depreciation appropriate during the period, with a corresponding credit to
accumulated depreciation. The balance in the accumulated depreciation account increases each
year as additional depreciation is recorded on the asset.
Karrington Corp.’s book value of the delivery van would be calculated as follows:

Calculation Depreciation Accumulated Book Value


(Cost – Salvage value) / Useful life Expense Depreciation (Cost – Acc. depr.)
($40,000 – $4,000) / 8 years × 12/12 months $4,500 $4,500 $35,500
($40,000 – $4,000) / 8 years × 12/12 months $4,500 $9,000 $31,000

Choice “a” is incorrect. The $40,000 indicated represents the cost of the delivery van purchased
by Karrington. The question asks for book value of the van at the end of Year 2, which would be
calculated as the difference between cost and accumulated depreciation.
Choice “c” is incorrect. The $35,500 indicates the book value of the delivery van at the end of
Year 1. The question asks for the book value at the end of Year 2.
Choice “d” is incorrect. The $31,500 was calculated using the depreciable base of the
asset of $36,000 less depreciation expense of $4,500. Book value is calculated as cost less
accumulated depreciation.

CQ–14 © Becker Professional Education Corporation. All rights reserved.


Class Question Explanations Part 1

4. MCQ-12397
Choice “d” is correct. GAAP requires an investigation into events that indicate the book value
(carrying value) of an asset may not be recoverable.
Indicators of impairment for finite life tangible/intangible assets:
yyA decline in market value of asset which is substantial and not likely to reverse.
yyA substantial adverse change in how the asset is being used or in its physical condition.
yyA significant adverse change in legal factors or in the business climate.
yyLosses during the current period with projections of losses continuing in the future.
yyDecision to dispose of an asset prior to end of its estimated useful life.
Choice “a” is incorrect. A significant change in the market value of an asset is an indicator of
impairment, but the other answer choices also represent indicators of impairment.
Choice “b” is incorrect. A significant change in the legal environment associated with an
asset is an indicator of impairment, but the other answer choices also represent indicators
of impairment.
Choice “c” is incorrect. A significant change in the estimated utilization of an asset prior to the
end of the asset's useful life is an indicator of impairment, but the other answer choices also
represent indicators of impairment.

Unit 1, Module 7

1. MCQ-12403
Choice "b" is correct. Assurance-type warranties provide the customer a guarantee that the
product/service will work properly for the time period covered. If the product/service does not
work properly, then the seller will correct the situation by repairing the problem, substituting
a new product/service, or possibly reimbursing the customer. With this type of warranty, the
liability and the related expense are recognized in the year of sale to "match" the cost with the
corresponding revenue from selling the product.
Emmons Industries will record warranty expense and an increase to the warranty liability
of $40,000 during the year: $2 million sales revenue × 2% warranty expense rate. Actual
expenditures incurred for warranty work reduce the warranty liability. So, the $40,000 increase
to warranty liability will be decreased during the year by $15,000 actual warranty repairs during
the year. The end-of-year warranty liability balance will be $25,000: $40,000 increase during the
year – $15,000 actual warranty repairs during the year.
Choice "a" is incorrect. Emmons Industries will record warranty expense and an increase
to the warranty liability of $40,000 during the year: $2 million sales revenue × 2% warranty
expense rate. Warranty repairs during the year reduce the liability. The $15,000 will reduce the
warranty liability.
Choice "c" is incorrect. Actual warranty repair costs incurred during the year reduce the overall
liability for warranties of the product.
Choice "d" is incorrect. Assurance-type warranties are contingent losses that are probable and
can be reasonably estimated. Liabilities associated with these types of warranties must be
accrued and accounted for at year-end.

© Becker Professional Education Corporation. All rights reserved. CQ–15


Part 1 Class Question Explanations

2. MCQ-12406
Choice "c" is correct. Treasury shares are recorded and carried at their reacquisition cost. A gain
(or loss) occurs when treasury stock is reissued (resold) and the selling price on the subsequent
reissue (resale) of the treasury stock is greater than (or less than) the cost of the previously
reacquired stock. The account "additional paid-in capital—treasury stock" is increased for gains
and decreased for losses (if there is an existing balance in the account) when treasury stock is
reissued (resold) at prices that differ from the previous reacquisition cost. If the loss from the
resale is greater than the amount in the "additional paid-in capital—treasury stock" account, the
amount of that loss in excess of the amount in the "additional paid-in capital—treasury stock"
account decreases retained earnings.
On November 15, Year 2, Midge Enterprises reissued (resold) 75,000 shares for $35 per share.
That $35-per-share selling price of the stock reissued exceeds the reacquisition cost by $3 per
share: $35-per-share resale price – $32 previous reacquisition cost. The $3 "gain" is recorded
in the additional paid-in capital—treasury stock account at $3 gain per share × 75,000 shares
= $225,000.
On January 15, Year 3, 10,000 shares are reissued (resold) for $30 per share, which is less than
the previous reacquisition cost of $32, resulting in a $2-per-share "loss:" $30-per-share resale
price – $32 previous reacquisition cost. The total "loss" is $20,000: $2-per-share "loss" × 10,000
shares resold. Because the additional paid-in capital—treasury stock account has a balance
of at least $20,000, the $20,000 "loss" decreases the additional paid-in capital—treasury stock
account; so the new balance in additional paid-in capital—capital stock is $205,000: $225,000
previous balance – $20,000 on account of the "loss."

Nov. 15, Year 2, Selling price > Reacquisition price Increase APIC—treasury stock = $225,000
Jan. 15, Year 3, Selling price < Reacquisition price Decrease APIC—treasury stock = –20,000
Balance $205,000

Choice "a" is incorrect. The $20,000 loss on the reissuance of shares on January 15 reduces the
additional paid-in capital—treasury stock account. This answer choice treats the loss as a gain
and increases the additional paid-in capital—treasury stock balance from $225,000 to $245,000.
Choice "b" is incorrect. The $225,000 represents the balance in the additional paid-in capital—
treasury stock account at the end of Year 2, but the question asks for the balance for the
following year. The balance is decreased by the reissuance of 10,000 shares at a reissuance price
that is $2 less than the previous reacquisition price.
Choice "d" is incorrect. The $20,000 represents the loss in Year 3 based on the January 15
transaction. However, from the previous year, an existing balance exists in the additional paid-in
capital—treasury stock account. This answer choice does not take into consideration previous
activity affecting the balance in the additional paid-in capital—treasury stock account.

CQ–16 © Becker Professional Education Corporation. All rights reserved.


Class Question Explanations Part 1

3. MCQ-12408
Choice "b" is correct. When less than 20 to 25 percent of the shares previously outstanding are
declared as a stock dividend, the dividend is treated as a small stock. The fair market value of the
stock dividend at the date of the declaration of the small stock dividend—not at the date of the
distribution of the small stock dividend—is transferred from retained earnings to capital stock
and additional paid-in capital.
The fair market value of the stock dividend at the date of declaration is $9,375,000: 5% multiplier
× 7,500,000 shares outstanding on dividend declaration date × $25-per-share market price on
date of declaration = $9,375,000 value of small stock dividend (and amount to be transferred
from retained earnings).
Choice "a" is incorrect. This answer choice incorrectly used the stock's $3 par value, rather than
the stock's market value ($25) on the date of the declaration of the small stock dividend, to
compute—incorrectly—the value of the small stock dividend: 5% multiplier × 7,500,000 shares
outstanding on dividend declaration date × $3 par value = $1,125,000 incorrect answer choice.
Choice "c" in incorrect. This answer choice incorrectly uses the number of shares of stock
authorized to be issued (10,000,000 shares), rather than the number of shares outstanding
(7,500,000), to compute—incorrectly—the value of the small stock dividend: 5% multiplier ×
10,000,000 shares authorized to be issued on dividend declaration date × $25-per-share market
price on date of declaration = $12,500,000 incorrect answer choice.
Choice "d" is incorrect. This answer choice incorrectly uses the stock's market value on the date
of the small stock dividend distribution, rather than on the date of the small stock dividend
declaration, to compute—incorrectly—the value of the small stock dividend: 5% multiplier ×
7,500,000 shares outstanding on dividend declaration date × $35 per share market price on date
of distribution = $13,125,000 incorrect answer choice. The stock's $25 market value on the date
of the small stock dividend declaration should have been used.

Unit 1, Module 8

1. MCQ-12409
Choice "a" is correct. A lease that meets at least one of the "OWNES" criteria is recorded as a
finance lease on the books of the lessee. For a finance lease, the lessee records an ROU (right of
use) asset and a lease liability. Generally, both the ROU and the lease liability will be recorded
at the present value of the lease payments. The ROU asset is amortized over the lesser of the
life of the lease (seven years) or the economic life of the asset (eight years); the lease liability is
amortized, paid down along with interest, over the life of the lease.
The lease is a seven-year lease on equipment that has an economic life of eight years. Seven out
of eight years is 87.5 percent, which represents the major portion of the economic life of the
asset (typically, a threshold of 75 percent meets the criteria). This lease is therefore a finance
lease, and Benedit will record an ROU asset. Because there is no ownership transfer or written
purchase option, the ROU asset is amortized over the lesser of the life of the lease (seven years)
or the economic life of the asset (eight years).
Choice "b" is incorrect. The economic life of the asset is the appropriate amortization period if
the lease had either an ownership transfer provision or a written option to purchase that the
lessee was reasonably certain to exercise.
Choice "c" is incorrect. Although the seven-year lease comes after the two-year lease, the
cumulative total of nine years is irrelevant for this determination.
Choice "d" is incorrect. The expected life span of 10 years is irrelevant to the determination of
the amortization period for the ROU asset.

© Becker Professional Education Corporation. All rights reserved. CQ–17


Part 1 Class Question Explanations

2. MCQ-12410
Choice "b" is correct. A lease that meets at least one of the "OWNES" criteria is recorded as a
finance lease on the books of the lessee and a sales-type lease on the books of the lessor. If
none of the OWNES criteria is met, the lessee will classify the lease as an operating lease, and
the lessor will classify it as either an operating lease or a direct financing lease.
The lease described in this question fails to meet at least one of the OWNES criteria because the
lessee intends to return the truck to the lessor, the lease term of two years is only 20 percent of
the economic life of 10 years, the present value of the lease payments for only two years of a 10-
year life is unlikely to be close to the fair value of the truck, and a truck is not a specialized item.
So, the lessee will classify the lease as an operating lease.
With respect to the lessor, the determination of either an operating lease or direct financing
lease depends on both "PC" criteria being present: present value (P) of the lease payments and
residual values relative to the fair value of the underlying asset and collection (C) of the lease
payments is probable. If both PC criteria are met, a lease is a direct financing lease; otherwise,
the lease is an operating lease. In this question the collection of payments for the lessor is highly
probable, but because the present value of the sum of the lease payments is likely well below
the fair value of the asset and because there is no mention of residual value obligations, both PC
criteria are not present. So, the lessor will classify the lease as an operating lease.
Choice "a" is incorrect. A "finance" lease is a lessee, not lessor, classification.
Choice "c" is incorrect. The lessor cannot classify the lease as a sales-type lease because not
even one of the OWNES criteria is met.
Choice "d" is incorrect. The direct financing classification only applies when both of the PC
criteria are met: the present value (P) of the lease payments and any residual values (guaranteed
by lessees and third parties) are essentially equal to the fair value of the underlying asset, and
collection (C) of the lease payments is probable.

CQ–18 © Becker Professional Education Corporation. All rights reserved.


Class Question Explanations Part 1

3. MCQ-12411
Choice "c" is correct. When there is a difference between income tax expense and income tax
payable, the difference relates to a change in the deferred tax asset or the deferred tax liability.
Income tax expense that is greater than income tax currently payable generates a deferred tax
liability. That is, a deferred tax liability exists when there is a difference between the income
tax currently due for tax return purposes and the income tax expense recorded for financial
statement purposes. This difference is due to a timing difference, which will reverse in the
future. When a company has a deferred tax liability, the company benefits because the company
will pay more taxes later, rather than now, when the timing difference reverses.
If after one year of operations an asset's tax basis is lower than its financial statement basis,
this difference implies that more depreciation expense was deducted on the tax return versus
the amount of depreciation expense recorded for financial statement (GAAP) purposes. This
difference creates a deferred tax liability and will cause income tax expense to exceed income
taxes currently payable. The deferred tax liability will reverse over time, because in later years
the depreciation expense recorded for the financial statements will exceed the depreciation
expense deducted on the tax return. For most depreciable tangible property at the end of
the asset's useful life, the asset's financial statement basis and the asset's tax basis will be
the same: 0.
Choice "a" is incorrect. Unearned rent revenue, a liability account, represents cash collected
and is reported as income on the corporate income tax return in the year of receipt rather than
in the later year when the cash is earned. As such, the income is recognized on the income
tax return before that income is recognized as revenue on the income statement. This timing
difference creates a deferred tax asset.
Choice "b" is incorrect. If a company records more bad debt expense on its income statement
than the amount the company deducts on its income tax return, this timing difference creates a
deferred tax asset.
Choice "d" is incorrect. Premium payments made on life insurance policies where the
corporation is the beneficiary are expenses on the income statement but are not deductible as
expenses on the income tax return. This difference is a permanent difference that will not give
rise to deferred taxes; so, income tax expense and income tax payable do not differ with respect
to permanent differences.

© Becker Professional Education Corporation. All rights reserved. CQ–19


Part 1 Class Question Explanations

4. MCQ-12412
Choice "b" is correct. Deferred taxes arise from temporary differences (often called timing
differences) between revenues and/or expenses reported on the income tax return versus
revenues and/or expense reported on the financial statements in the same year. However,
permanent differences will not create deferred taxes, as permanent differences represent
tax return versus financial statement revenues and/or expenses differences, which will
never reverse.
The net temporary difference multiplied by the currently enacted future income tax rate (if
the same currently enacted future income tax rate applies to each difference) will produce
either the net deferred tax asset or the net deferred tax liability reported on the balance sheet
(even though deferred tax assets and deferred tax liabilities are separately maintained on the
business' books and records).
Municipal interest is nontaxable on the income tax return but is recorded as income on a
company's income statement. This difference in treatment creates a permanent difference and
will not give rise to deferred taxes.
Accrued warranty expenses of $12,000 versus payments of 0 give rise to a $2,520 deferred tax
asset (21% × $12,000) because warranty expenses are not deductible on the income tax return
until payment of those expenses occurs.
Prepaid expenses of $26,000 for rental costs paid by a cash basis taxpayer give rise to a deferred
tax liability because this expense is deductible in the year paid. That is, a cash basis taxpayer
can deduct the payment in the year paid, but the payment is not recorded as an expense on
the "GAAP-based" income statement until Year 2, the year in which the company benefits from
the rented space (note that many businesses are "cash basis" for income tax purposes but
are "GAAP basis" for financial statement purposes). The net deferred tax liability is $2,940: the
$5,460 deferred tax liability reduced by the $2,520 deferred tax asset.
Choice "a" is incorrect. This answer choice counts the municipal interest of $5,000 as a
temporary difference giving rise to a deferred tax asset. However, this interest is a permanent
difference that will not lead to deferred taxes.
Choice "c" is incorrect. This answer choice treats the municipal interest of $5,000 as a temporary
difference giving rise to a deferred tax asset. However, this interest is a permanent difference
that will not lead to deferred taxes. In addition, $9,000, which is the sum of $26,000 – $12,000 –
$5,000, is the net of the differences themselves. However, the net differences multiplied by the
currently enacted future tax rate produces the net of all deferred tax assets and all deferred tax
liabilities (if the same currently enacted future income tax rate applies to each difference).
Choice "d" is incorrect. This answer choice reflects the net temporary difference of $14,000. The
difference must be multiplied by the currently enacted future tax rate to derive the net deferred
tax amount liability (if the same currently enacted future income tax rate applies to each of the
differences, which net to $14,000).

CQ–20 © Becker Professional Education Corporation. All rights reserved.


Class Question Explanations Part 1

Unit 1, Module 9

1. MCQ-12413
Choice "b" is correct. Discontinued operations are reported net of tax on the income statement
separately from continuing operations. A discontinued operation may include a component of
an entity, a group of components of an entity, or a business or nonprofit activity. Discontinued
operations are classified as either disposed of or held for sale.
Thompson Toy Co. will report the custom doll house division as a component of discontinued
operations, net of tax. Discontinued operations are shown below income from continuing
operations in the income statement.
Choice "a" is incorrect. The division will not be included in other comprehensive income. Because
the division qualifies as a component of the entity according to U.S. GAAP, the division will be
reported in the income statement as discontinued operations.
Choice "c" is incorrect. Both the income or loss from operations and the gain or loss from
disposal are categorized as discontinued operations and shown net of tax on the income
statement below income from continuing operations.
Choice "d" is incorrect. The custom doll house division is appropriately classified as a
discontinued operation, as the sale of the doll house division represents a strategic shift and
qualifies as a component of the entity. As such, the financial activity of that division will not be
included as a component of income from continuing operations.

© Becker Professional Education Corporation. All rights reserved. CQ–21


Part 1 Class Question Explanations

2. MCQ-12414
Choice "b" is correct. Bakers R Us must recognize revenue over time if one of the following
conditions is met:
yyThe customer controls an asset being created or enhanced by the seller; or
yyThe customer receives and consumes the benefits of the entity's performance as the entity
performs; or
yyThe entity's performance does not create an asset with an alternative use to the entity, and
the entity has the enforceable right to receive payment for performance completed to date.
When revenue is recognized over time, the seller must be able to reasonably measure progress
toward completion.
At the end of the fiscal year, the cake decorating class has been underway for two out of three
months. The company has earned $640 in service revenue for classes held, and there is still one
month of classes remaining for students who enrolled in classes that began August 1. As such,
the deferred revenue (a liability account) is $320: $940 received prior to the end of the fiscal year
in which the business received the payment times one third of the classes to be presented after
the last day of the fiscal year in which the business received the payment.
Choice "a" is incorrect. Revenue associated with the arrangement should be recognized over
time, not all at once. The class duration is three months, and only two months have passed at
the end of the business' fiscal year. Revenue eligible to be recognized associated with the baking
class is $960 × 2/3 = $640.
Choice "c" is incorrect. Revenue associated with the arrangement should be recognized over
time. The class duration is three months, and only two months have passed at the end of the
business' fiscal year. Revenue eligible to be recognized associated with the baking class is
$960 × 2/3 = $640. The deferred revenue (a liability account) at September 30 is equal to one
month's revenue, $320.
Choice "d" is incorrect. Revenue associated with the arrangement should be recognized over
time. The class duration is three months, and only two months have passed at the end of the
fiscal year. Bakers R Us is able to recognize two months of revenue and does not have to delay
recognition of revenue until the course is completed, as the client is consuming the benefit of
the classes over the duration of the class.

CQ–22 © Becker Professional Education Corporation. All rights reserved.


Class Question Explanations Part 1

3. MCQ-12415
Choice "b" is correct. The amount of variable consideration is estimated by taking a range
of possible amounts and using either the expected value (which sums probability-weighted
amounts) or the most likely amount—whichever is assumed to be the better predictor. Variable
consideration is included in the price only if it is probable that a significant revenue reversal will
not be required once any uncertainty tied to the consideration is resolved.
The expected value is utilized by Jones and Hill, resulting in a total expected value of the contract
at inception of $28,000:

Possible Amounts Probabilities Expected Amounts


$29,000 (includes the $5,000 bonus) × 80% = $23,200
$24,000 (excludes the $5,000 bonus) × 20% = $ 4,800
Expected value of contract at inception $28,000

Each month Jones and Hill will recognize $2,333.33 of revenue: $28,000 expected value ÷
12-month service period = $2,333.33 per month.
After six months, Jones and Hill will have recognized revenue of $14,000: $2,333.33 monthly
revenue × 6 months' passage of time.
Choice "a" is incorrect. The $12,000 amount is computed as $24,000 total revenue without the
bonus ÷ 12-month service period × 6 months' passage of time. However, the $12,000 amount
does not consider the potential $5,000 bonus. Under the expected value method, the probability
of potential payouts under the contract are considered in the computation of revenue to be
recorded over the duration of the contract.
Choice "c" is incorrect. The $14,500 amount is computed as $29,000 total revenue with the
bonus ÷ 12-month service period × 6 months' passage of time. However, this amount does
not consider the 20 percent probability that the firm will not receive the bonus. Because the
business uses the expected value method, the use of the $29,000 "most likely" amount is not
correct.
Choice "d" is incorrect. This amount represents the expected value of the 12-month service
contact at inception of the arrangement between Jones and Hill Consulting with BGSE Inc. This
value is used to determine monthly revenue to be realized over the entire one-year contract.
This value is not the realized amount for the end of the first six months' revenue.

© Becker Professional Education Corporation. All rights reserved. CQ–23


Part 1 Class Question Explanations

NOTES

CQ–24 © Becker Professional Education Corporation. All rights reserved.

You might also like